Sie sind auf Seite 1von 249

.

~
,.

/'

Publis'Md .t Distribut.tJ by:

~O~-t~~
si:si. '

. 8~ ~k:anol' Reye
Tel. Nos. 741-4916 74;_.49:20>1.
1977 C.M. R.cto Awnue
'
Tel. Not. 741-4956 741-4967

M1nifa. Pt.irlppillff "r ",:

Joj;"BilJOna [oriu~daip_
':

..

.' '

RBSra ~ ~ Asi!oOa~s~: Ar'ociaf:iqaot


P,lll1iJIPin.e.~~ <APBt; BOok .Dev !liipnent A~aticin
0C. die Phqip~'lBDAPJ;Philippine F.duca~ Publishers ,

A#~tioci (J>EPA>
'

PEi>A's lnt.ernational Book'.-Assepations 11.embership: Asian


. Pao&c.Pubmd-s ,AsSiatim tAPPA>;.Associatioa aL South
' EMt~Asian Publisher. CASEAP>; ~tiooal Publishers
"'! . ~at.ion <IPA>

P~ted by i

AEX

~OJ11MY.ic. 84-8fi p_ Florentino St_

Sta. ~C!S11 He.ights. Que.ma.City. Tels.: nUl--08, 712-U-01;

Fu no_ TII-54-lZ

"

'/'

,,

'

.,

"' .':

"
'' ,

:, .

....

Pre.face

"

'

. ' .~;', ~
: . .
.
};
.
'The book : entltled Learning , Gulde In Engineering
Jfe chanlcs was written as text / reviewer for. fhe purpose
o .f .-present'Jn(. fhe prin~iplEia
and
concepts
of
'E'n gine"ring Mechanics iii a ver1. basic and .$YSte.aatic
!lPProach to help the . atud~nta .
underata!id and lear .n
the ' subJect matters that wJ,11 - develop .the 'J r .. 'o rderly '
~ pfoc~'ssti's ::ot th'tnk,t'n{

. .. :

..

1,

I
,o

. .

lU th .. the . pr e se~ted t~pJ cs. 1 , ~.he u,s.~rs .will 1''a"e... a_.. ....
very comprehen~ive, a :t~df" Of the Auhdamental pr:tnC'iplt['s
of Engineering- 11ec11anica wblch are' appl I cable.: ~o wide
varJety of prac~fcal . s~ tuatii>ns norally encoun~ered by
.,r. :/
" them iin \ heir' day to :.d ay acttvJtle,s ~ .
.. . " . '.
..
;

'

Extra effort was exerted so that everyt~lng will


b'e presented 1 n .!l 'v.ery ,p~dectly ~ unde.rslood, .but ,in . a
clear and concts*' language. This l'itll eilminate' or
reduce ~o a liainimua the !'tudent's attM~de o.r " b~~it of
inemp~i. z~ng tbe con~ept with o~t u.n~e~:standJ~g. , '..:
_ The authors wish to acknowledge ~h~J r inde b'fedriess
M,ichael
Siongco, .Jef'frey " Bo,rlongan,. , Jl.onaldo ,
C~tin~lg, . . and Rqm~o
Adrlano fo'r
tlletr
valuable
contributions ln preparing and .;rJti~g tbe manuscript.
to

N.

c-:

'l,"

De l.a~.ama

A.G. Mendoza

"

"

. {

...
\

-.

,, . .

'

...

:'

ACKNOWLEDGEMENT

..

'.

The a'uthors arc parti~ularly indeb~.in the p~aration ofthjs book to .

Mr: Romco Adriano, Mr. Ronaldo Catindig. ~.


Mr..Jcfilcy BOrlongan.

' ,.

GeiardosiimSon, and

, They further wish fo speeia.lly ri;eogniT.c the exerted effort of


Mr. AmoldQuetiU! in wtitilig the mS.nuscript
I

~-

S~ial mention fo those who briitg tb,e greatcstjoy into Nick's life,
r,,ii~, Nikki.Md' NikkJl:,
.

'

'

-.::.,

. '

' ,..

'1

'

'.

'

,':

CONTENTS
Chapter 2: R.aoultonfc; cf force~ t - H
Choptor 3 : fquil1bnum ci" forco ~ 2S - sa
Choptor

+:

hno~

of S trud~

OqJtcr 5 : frlction
Chapter 6 : Fbroo Sydcnw. in Spcx:o
Chopicr 7 : Controidco "- Ce-tlC'Jl"'C of

'

Gr<Mty

Chqplor 8 : Momc;ntG of lnorfio


O)optor10; Roctiliooot'- Tron&btiorl
Otoptc;r 11 : Curviliooor T'l:)nl61ofion
Chapter 12: Rolotton
Oooplol +.: Worf\ 'Enorgy

Choptor 15:

.s.-86
fJl -m
110-BO
B1 --145

146--tSQ
t60- P4

ns-t!H
1a2-1l11

lmpulGo ~ Mornontum

1!38-Anr
3Jr2t3

PROBl.EMS

2K-2'0

.,

..

'

'

<-

''

Choptor 2

Resultonk of Fon::e f:y~tem~

ft>f.) Determine lhe. X 'Y conwponGOf6 of ooch of the forces ahown in Figure P -2o+ .

consider ftle

3QOJb

lood:

f -300 ( J,~ &* )

-t.5012!_b

F.!:I 3GO ( Iii' t~

.960 ""'.

eonaM:.ler

r,. --too

eoe -t0

I'.~ 300b.

r--tOOl!L

fhe. ..f<>D lb lood:

eons:Jer the

R> 1ood :

- aoq :fil lbs.

T!4 - .f<XJ

'1in -t<t

.. -'}~]. 12 lbs.
205-) Compute the '1
. c;hown

in f iqure

"> Y

~nts of eodl of the forocs

P - '1.0!!J .

conraider tho 7<?2 lb food:


1,

Ja\a*
)
I

T...

-111. (
.:.i.Jvi ,

-;ix.x-41911:)..-.

"

,"

------'E-.E.----

Tu 122 ( J}. .~1 ~-d 1i1oo. 7-+ lbs.


r'
con$;1dor-

p,. ..

fhe 20C>lb lood =

~00 COQ 60. ,. 100

I.

P9 ~ sin60 i 173. 21 lbs,

~ ihe +f8 Lb. load:


2
f-.. - +'f6
.J ~.f~ J] - ..f(J0. 7 lbF-

Fg

-tff(J 2}~ 1*

~ - ~.!J!J II.

200.) Tile triangular blod\ shown io fiq- P-a16 is subjCGfcd


to the loads P woo~ t., F 600 lb. ff 1\5"' Bi"l ~ l3C 6~. ~

resofvo eoGh )ood into COrf\pont:tJts namal ~ Jongentiol


~.

eonliider the fooo lb lood :


sine--~

C:OS&. ':

At ..L(t600) f2Mlbs.
10

Pt 1~

(1600) 960 lbl>.

AC./e6i
"' fOin .

." '

c.cnsider the 6<P I~. lood :


~in e- ...fL C.OS e- _En_

. " ;
ft

F
(~oo) ~ -+80 lbs. .

10
I

fn 6
10

i . ..

(600) .... 360 Zbs.


'

20'1) P.eworl< p(()b.

al6 if -9 60
ft f.sio&' 600 Stn6t:i .. ~tQ.6~ lbq.

fn
Pn
Pl
.206)

fcose- :- (J(Jb coe 60 ~ 1~Pein& 1600 sinao J!J8~- 6+ JIN.


p C.05&, 1600 00560. 800 lb6'
The horixonfol ~ ver-ticol compgnenfs

or .-c:YCrol

fbr

ccs ore : (o~ Ph~2tp./~. ~ Pv.toplb. i~1'.) m,~300/1) ...,fv -wo


Jb. i (c) TIJ .. -!!JOlb ~ 7Y -go/b. Deter.mine eacn force6
o .)

~~
. , Pv
~

Ph

ti.)

Fh

..

P ./R.,* .. Pn1 J<uxJf.t 100 \. 2:1s.61 lb .


ton-e- - Pvfai 100Aooie26.~1

fc:~f~1
~

Fv1

{'SOO~ 1(-u:;,'f'

.. ton - 1_:2!!)_ ... -

360-56 lb.

33.6g ,.. i!l.60

300

c)

Th

1(

pv

T./Trf
- l,<-150) 4 (~~)I 102.!16 Jb.
~tan-~ 609+
tTv 0

&

-<T

Vfh... .. Fv 1
ton- 1

./(-2'40)f + ::i60~
s60ft~ - s6 .-,1

+32.6.7

rb.

'

'

../(-G<>O)~ ~ (-:aoo)c

T v'rh t llrs

.!1&!1.1 lb .

-e- ton"' m/.,oo - so.gg.


z.ao) Tn Fig. P - !l.10, tfJe x oomponent of the force P iw
to tile Jen . Determine P tf} jf6 Y componeni . .>

1-t0 lb.

p. 8.06(1+o)h - 161. 25 /b.

P1/p,c
Py -+h (1+o) = so lb

Ton-&-

111 ~ The body on the

so 1'noline in Fig. P-.2.11 i~ oded upan

o" forcp P inclined ot f20" ~/ ihe hOriwntal .If, P i~ resolv~d 'into componerrfe par<allel .~ pwp~iculcir t? fhe inby

componenf

cline:~ the.value. bf the parolld


!>.'

of

~'(()~Ive
fue !:~~: :-m~::~ ~o; of P
Sin&

00

is ' +oq1b,-carnPote

PJ. =

P "'~ ,. 6~2 . !Z9 lb$.


s1n10

-f<X>

+16.1 tb

ton id

j'n Fig P - ~HJ .

J!1.. :.

...s- .IOO.,
"

~r>c

-~ cos<IO

~.

aoocos+s-,oocosao

-161 . 07 lb .

~Fy ~ ~60 t 300 6'1-ts" - 200 C>Jn ao


~ Fy

.. -f-5& .5+ JO .

R /~tv + $.Fy 2

"'

/(r----1-6-1.0~7),....,2_+_(~+
-~
-e-.~~
+)~
:i

.~~~86 . 01 lb

-e-" ton"'

"t6S.5-f

up to lhe leri .

70.65

161.07

a:1+.) Dcferrnne: thr::. reeulton~ or lh~ conou~n~ s!1sfe.~


forces 4'hown in f(9 P - 21+

<,.~ ~ -~(;Of. .:ta' -

S000 (

- -1-fJ66 .0S lb .

~) t

<JCX>0 GOSSO'

or

"4 ~

.- I

,,-

..
I

"

..

...

\,

\ ,'

'. ,.

.~ . .: , \ . ': ' ::f ' , , ,-_.

I. '"

.,.

'

'

',

~'k/

. _;,
. ~fy .; ; :..f~~Jb'. .
~ I
-;.=---'~"'1r...i~-:+
R J~F,.~ t ~Ff'" "'_J(_-+a66.o~y~
~ +

'J

'

'

:.

~,

',._'. '

::

"" . l;

'! '

'

'

(-1~00)2

'

. <:' ;.' .. , ~x ,- , t~~'. fsoo/:1 066,03

. ,

' '."

: R'." -"09j.9.7 lb d~'.,.;hdo tl1P 'lef'f

. .

:,,

~Fy~ woosiriatit~ -~lliina0-1oob -s~'--J..\

."
~

':6 .

'

.17, 1::i.

, : '

)":" ;_ . : .e1.S:) . Fi're. the.,.~ul~qr:it: of . Jl:ie. .. ~nCu~t rqrce ~y6tem s~I) In

.:~~\;.;: ~- .
,,,)?' ' h,>,

, .

:/~~~/..
'r:,i,~.. '\

,'

:--

. ~lb;

'

.z..'. "
t.

7-

'

<' ' 1,

': ,. ,,

. . '"

'

. .
''

'o

'

'

x . ~Fy. ~:qap s1ri 3o - aoo .sin 6:0..- a~o sin.po. , ,


. . . ~Fy. 0 -419, 6~ lb.
'

"
3ai:>tb,..

...,

-,

:.,00- !loo~
ab - aob ;;x;;6~,"t 3ao boa 6o
,

' '

. J:

: ~f~ .;;,. :1~6.79.. lb .

: '

'

. ,

: .

"".

~~~~1

>.,.:.1.~.'.~.:,.,
_:~ ..,f.
.1

,'

fig.:P-ti,-16 :
.
.
.<- .

~,. =', fon~1

'aootb ,_
I

-t19,62 "' s1..6,: ,

. .2~6.;79

.. '

;,

R"'.feFi<'<-t ~Fy.2 ~\/(;~6,7g).z. ' (-"19.~~) 2 ! '


..- , , R~ 476~ee 1b'.. .. ci~..... i~ +he f.{<:J!:+
~.) ~ ~eur~t': .fo;::_~, ~cte;:r> ,~'d-Qw"-' h t=Jg-. 'P--2.16 . Oetei;<-

1>

frltne . the r~vitatit.

... ,

.. , .v'..::~io '.'

')"

'

.- ~F,. ~ -fQO ~;.~ ~ ;2:oo(i-)"'" 3~ cos 60~;

..

'

. ' .,.

= .-316.+Hb . ., .

~--~-JF--"--c,..,___ _
. x
,~PY."
.

+oo.sin 30 - !Zi:>;(i-)
-278.81 /b.

~ooo~fn 60.

~Fy ' ~

, ,

F<.#~Fi" t~Fy .=J:(a16.-11)2 -1(-!Z1981)


. , R.c ~B5.27ib. do~ fo :fhe r-(9ht '

:"""-,-

'the

-~l<

217) O,mpufe
vqlue
oe"s ~fx>w'"1 ;n fig. 21 7.

for:1"1 ~1~,SY.316+1 . ~ 31-,79.

6(/be ~lto~f

=iEF,.

fhe~ c~~r'renl

'of

.-,

100~)
H.00COGo<,.-3COCD~0-2Qoer,16
~

for- ;. .
)>

,.

' ~Fx b 144. 43 lbs.

. .

' .

-;l!IC....,,.--,---~ ~Fy 1~~) f .\Oa?SW-60' ~ too"!-~~ - ~~ Giri+G.

~r~

~~

_' ' ,.. .:


I'-'

. .

......

~F7 ~ -~~.9 lb ,

R. =./""
-~..,.F~,.!t......-tii.:=.=17..,,:z- {(1+-t-.f:;5)'! t- ,(-,.a6.89)'"-
R,. 1.,.~_...07 1ti "down to thf;. ri.9ht
.
~ = ton_, s~.69/i.tt;f3 - 1+. 3.3 . : , .
body sl-owo f" Fig. p.- 21e i" ooted 6ri ~ thr'ee fbrce~ .
4001b

z10.) The
Deteirmine the res.ultal"")f.

'

'

'l

,,.:,

' f

I/

, "'.',

.' ..~.
\'

'-

'1

~~~~.~ ~(+l'.s).:t.;9~(~,~~~) "';2t (14i)


... , '::rf.,;. 2~,'8~ ~b. .

. ' . ' .. .

'I

t--:--Ml'~r-::..,,,.x__, .. :E'l=y_~;;3QO<~~? ~:~~~;c1., ?(~) .:.:,2p~_(:i~) .


.. _, -38C,3:5 Ip.

>

, . .

.)~..-:/.~,~ .t.~~t~-) l(~ei!.9'~)~..~ ,(-,,sa:a~J,1,


;
1

'

B... ~7e ,1::i d,owrf tp t~ -Nghf

':" ,, ,

',:,

r_',.'

'

I ...~

'I

.'

'rt '{{'-.:': ':- ... ...........r:e-.lf.


.

~i.0.2 Deter-mine

~~j

~,

..

1,"

'

'w

..

'

."

~
'1 .

'. /

.-.;i;tf '' ./

,I

.!

'

' ;

,,

'

,...

Jhe re.sultant of fhe"-foul' ,foicee ootrng ~ the.


~.......

'

~~lb~ 1 : ~....

.'

.~'. lan -i '~~O.j!,S'':z8 9,5.,2 .. .!ll2.p~ 1 ..

bOdy"<sti?wf'l:II:\:: F1g'.1P- 21g ..


.

'. '

"

lc,

c' . .

,, .

..

.. ~~600(~)t~1(~) ~ +1:fl(Ws) .: ?60(1i3)


""'"' " .,.,..;;.
' .. ,.,
'J.
'
'<'Q.91b\
'
'
~,..
o:>-rv'0"fQ
...
'
''
'

.~

,,.
.

:.Fr,.6o0(%) ~. ~ft/a)-s6d'/ti!t)-::i{'J{~)
t ~Fy ;. +.60.'-4.6. lb. . .
'

.;~R .)~,_,_. t:~fi, .2/(3 t,<?:~~)~. ~(+&<:>.-+<s) 2

.36110

a~ :.S7:1.a9 'Jb! up 'h~ fkrl9h't


-e-,. -~ io~'~! -t6'~,+.'4+<iO(l, .- .sa.s.s~ .
~~Tho resulfonf Q" a '"certoi" ,~stem cf. forces" hos the 'X ~ Y
c.Jmp:i~nfo, ehdw~)'ff\ fi9 1 P~ 220,,. P~t~rrriine H-e ~r\Or\t~ ,
.'<: tN~ ~sv/_lo~f,i vj/ rupooi to the ff~ T oxeG rotated . 30
'.

..

'

"'

6ount0Pcl~l<:w1seu-elalive tp
R
y
,
'.,p

fne x.\s y. oxes. .

~T- 11 I~it' , R .. .;3oo~ t +so<


..:;~_-1-/-x

_ _ _ _..,,.o::.J..

R,.c3ci0fb.

=,!566.0+.!b

, -&x ton_, +sopao = 57. 00


~ -.s7.9~?/-a9 ,. ~
Rtt

~ .566.04-

up kit~e r.1ght
.

oos !l7; 99 .+99.01 lb-

Hr ..566:0.t '9ih ~ ;.99 265, 6.911b.


4
.2zi) The: 're-svlfont of the GOl"GUrren~ forces .stiawn rn Fig .P-.e~
is ::JOO/b p01ntlng up along ihe. '( QlClS . e;om"pute th~ vck.ies of F
."5.fr reqvired ,tq _give thi s ;~esuHqnt .
-

~lb..
,/ I

Fcos fr t 40 c.oi; ao .; ~oO


Fcose- fl02, t5
fi!_fy a Fs1ne - a40 einaO @

- ~

6Jbs t,'fufe

. .z:ry

.C:fi< =O '

= ' 11!~2. n; ' tan& - 120

F<.

=.

o + ~.Fr"tl.

soo~ ( ,zg,z ,15fon &- -f.2D)1.Z

00000

(-'9f.t$)2 taf4>- - ~(!Zs2.1.s)fane-

8535'-G to""-&

-70116-9t

~~io Formc1la :

ton-e,, 1.+t

t 1....+00

tone- -766()0 "'o

-f'"t "' 5~!>.18

fOl\-&i -0.~16 . ; ~. - -a1.63

c1-

-e; ~ .-c.1er

>

frorn1

,,

I'.' ..

2g~.15 . '

- 511, 61 lb

<XH>56-~e

~) ~~t ~- ~1ll -.r ffe J'C$Jlt01'f iio "fOo lb o~n fo ft-e r~ht
of 60

W,ff\ jhe.' XO)ltG.

ft)I ~f)c
<fCO

'

oos 6'J

f). t ~08 -600 , ..., :F,..

...

"t92 lb .

~y 1Fy

Fy"' -~6."fil lb.


- ..s-t1 . 6 1b.

- "t006"'60" Fy-120 -

F/~r7'c~F1 a

::/(J9!l)a+{" !l~6. "'t1)z

,,.

-e-... = ton""' q!26AY~


12+.n
.:
.
~.). The block Gl10Nn "'- fig. P - !l."!13 iG ool-e.d UPo'\ by '1$,wei9ht
W; 200lb7 o hort~fol l'brce "fV 6Co lb, ~ ihe. pr-oscure' P
~ t:rr 1tle ~liru::cl paof'\e. . "The resvlto~t R.
the&e
fOrcei; is up~ paroalel lo ftle ln.oline thereby sliding ffic::.b .
up if - Oeiormin~ P ~ R . tlint ; Tal<e Olle Q')(i s porotle-1 10 the.

or

W~ll>.

MJ()lb ,,

to. obf~I"- 0-~()I'\~ paratleJ to fhe i"dir-ed plone. . e.,,~ ~o

.t::f=y' 0
PG06tfi -~ sinao - a.co 00680" . -o ~ P- +ee,0 it>.
6 7.' 600 CJ:/6'3(J - P 51r11r/ - 200 6~ao
R J~F,.. ,_ f' ~F7 " - /(w~.aiy -to -

~F)'.' 2 02. 62 lb.


~ <f~ . e~ lb
oana l pull a bcirge.

1.ff.) 1wo hcn;eG on oppa;ito bani<.' of a


rno.rlng poroll&l to 1he banks by meoni> of iwo hor1z.otol rnf6
ihe ten510n "'- ~ roi>e' are- 200 lb~ !Z-40lb wtile -Me- ongle
he;tween fhem .~ (I:) rird the. mwltont pull on tro_bor9~ ~-the

,-

. ~~ betwee:,i Jd\

\ . , aoo

cf 1he

~ ~ lhe 6ideii Of ihe CXll'\ol .


In order to have o .resultani para.lie? to
)~

~FyO

bon'k&

~Fy 200 sin e-

- 240 sin"'<
O 200.si~-&- - 2'+0 si~(60--&}
bv), s11'1 (60-&) "' sin 60 cose - sin<:>-c.os6o
O 200 6~ -24-0

ainoo COS9-+ ~40.9~COS60

zoosin~ - ~07 . 8S COS& - 120 &ine.3:t0

s inr:r

~T- 85

5m C'"/cos~

oose-

201.9~i10

Ton<t o.6-.95
~

o(

.. 33

60

-e-

60:-3!i ~ 27

"'=FX :: - ~c.o533-2"fa cas:l7' .. -~01 . .sa fb.


.
R - l~Fx* + ~Fy!f a
397,59 lb.
-2.26.) 1n fig . P'-'216 assuming b'1oyhw;se moments
posi tivo.
c:ampvte IM momenl of fo,..ce F +oo-1b '>of force P !361 lb.
ot>out poinl~ I\. B, C,~ D .
. Consider force F
vP f
M,.. - - Fy ( 1) - Fic (3)
,.v
.. - -t~(/:i}(1)-+.so(4/e)(3)
I/
c
MA
.. - 13.!So n - 1b
\
'
Me resolve ol pl. 1
p"\
Me fx(6) .. +so.(-+A;) (6) .. z160fi-/b
i':.
D o>t't
6
Mc resolv~ ol pl. 2
Mo re&olvo ol pl. 2
Mc fy(s) = +-'0 ( .3/s)(.!5)
Mo-fy (1) tf" (3)
Mc-13.so st - /b .
= <tS0( /.!5)(3)- ~(~AJ)(1) Me reSdve ol
.-+
Mo = 010 0 - 1b .
Me Py(1).: - 300.31 ff - lb .

os

consicie,.. for'Ce P

Mc resolve o ) p} .3

Mo. rusolvo oi pf. 3


p.,, 3d1 (4/ffi ) 200. zs tb .

M c - Py(4) - '3()().37(41-)

Mc :..1wt.-t6 0-Jb .

Py = 361 (:V.(13) 300.'37 lb . Mo resolve of p~ . 4 Py (z) - p,. (~)


Mo - Py(+) 3o0.37 (+)

M;..

"' soo.a7(2) - 200 . 2s(a)

Mo"'

1~01. 41-6

ft "' lb.

..

'

'<\.,.., ' ...

- ....~

:\:

......__ _.;.;,---.-,-,..'.. - . ,...,,..-.-.....--.~....,,.-..'".'"--.;""."'""'"""~-.,...,-__,...,...,..,-;;,,..-r


.' ,....,
. ' ~J: . :
zv.) Two fd-ces P- ~~ pass lh~gh ~ poin) l\ - ~/Y.'"f.s4"f.i .: '.
to }he .r1f:it,o~~~ .3 ft,. p'e_ove a -mom~f: ceri.'J~r e,: -Fbrdei.P.ds,'
20010. d ifecteef ~p lo >he r ight al 30 wf. the .hori~n lal.:' 'ii)" ~- ,..
1

force ~ is i<1' lb . dir.-eote'd 'up totbe . J-ec.t at 6(f.~/ }00,~!'lof-'i


zon.lol . Oelermine lhe m0men.i .ol lhe .resuJ.tnril of'tl>reoo , : .,,
: two .fd~ w,< ri'qjJecl ' lQ Q,;, "
1

'>

" 100. . .

'

'

'

'

'

I'

'

-M~. ~-:-P,,.(:+)+P;1.,(p)-(\)y(-+1-Q.~ (r.)

p~wo_,. ,,'(7'2o0sina0)(+) i (:z0ocosao~)(3)-

',

.J

. (100 sin 60;)(+)-(1oocos 60)(a)

w
'

'

\I

'

..:

-376:79 rr~Jb (m~ning

ccw)

<
, z.zs.) Wi}hou:{ camput1()g }fie magnitude . of the resul lon1, del . , , ~,
vv:here lhe roouHan) of lbe forces shown :;.- Fig. P-228/ inl,cr- '.
~els }Ae ?C ~ Y.a'Xes.
.
, ..:Et.10 "' - 361 ( ~3 !6111(s) t :2(1.!500 .ros.~)--.S~(srn :+s'?(~)
.
.. 1001.23 11'\-lb.
'
' '
''
'.;; , -. ~l"x ' ~ .900qo.9"'t'5 1- .;3'61(%.61) :;. 6Ss;g~.fb.
' .
~Ff =: _.500 .sin +5 6 - 361 ( 2/~.61) = 153.31 ,lb:
.
,
lx ., .Mo/~t=y "'1oot: 2il/f~3:31.. .; .6.s:a in' to the 1en oi'O
ly =~~F,. .,. 1001-2?/6.53.9!2 " 1.sa fn ob(Ne 0 ,
,, -, !
I

'
,,
"

r.

.S

,, v v

,~

..

/.

' ,....

'

'
..........

~f'

361 lb
'

'

~9,) In F-9 P-~~9, Find lhe.y coordiiiate . c)f pOin' A .so lho1 ...
lhe 361 lb force will hove aclock wise mol'l'!4'nfo r400 f}-11:)
oboul 0. /\ lsa delermine lhe x ~ Y in lerc;.e.pts of the ocf ioo
line. of }he rq,,.ce. ~Mo" a61..(:a~.61)(Y,.,)-:J6t(~/a.61)(a) ... 40ofl-lb
Y
.
~lb Mo -.. -400R -lb 300~A-'f00 /b ,
~'.'.:~!!~,ll
J'\ l1

' ' !JA"" ~.67ff.


' , ,
t,,.
..
-400:a61(%,01)i,.

Moz:~f,.J
..

:YI\
tK "' 11.Q ff Jefl (>[ (}
-+---''--~
)(~Mo
- ~F. L.!i = J361 (~, 61}(iy) ..
0
L!f' 1. 33 n: Clbove o

"fOO

...J

/ '~ 'r

~~i

.jf~;:;.,~-;;~:~~~~r ,.I~;

,iff1"t.,f,~"!'.=-i1 .h}rr.,r:~-~1o:, t ~
.f...t-,il
)<,~ ~~~ ~J

--;~:

t.-1

..:.

~~: ~.

. r:

...

''\:t

1 ,

. : . . . '. ~ .. .,. ''"

.:

-~

\ ..

.,.

i.

'

...
'

,i,

f,

:f1 .

'.

~:~l~ ~- ~j~::) .~ +we ' r:u~~l-<s~~~) t~:f.ig : p~ . ~~Q ,. i::;ornp'ute":~herpor'pen~


~f~:~.";/~~ifi.~.l ~~: ~~0,.0.~~:~P.'fi.~/~':6tq,1~e. )io,e1 f30. H!nt: [ma- .
: '.

g;r;ie a for:ce ./:directed olgpg ~D~ comf'!!ie,:.. its moments

'111

~:t. :i ' l'e~f'tt?l~of :j)9p9fiTIPQnen~s'.o_oout,; t5 ~ o~~f 6r1T~n ~qu9t~


~,,; .: ~.;,1 :ltJ~se~rF:'lf.s,: lq . tt,'l~. d6;f '.ni t.1~n ~f
M = fd fo.~pu le

rr:omen

;-',... : ~ lhe required perp_enq1~uJor . d1.sfcir/~', ..

~.~"' ~'":-;'",:p;

"., ,

;_;,.~

1J~~j

1,

._...

{:'~

'

I .
J~

-..

..: :~ ~ -''
c!e\ )6'

' :

12!

' c! j;f'~-E .
G
') ~~lye:', o) 0
~M .. 'f<.(3/-r;:o)(16) ; ':ft'cde)

de ..

. I

~otve

'al

1ir16 Q
D .~ '

~Me r'ca/~)(16) t F-( Vw; )(12) Fdg


. . ~~= 1~; g, ft . ..

,.

A force P posslr:i9 lhrougt\ ' p,dinls " ' lf., B ii\, Fig. P-2a1
I ot .300
-lb obo~ i' O .
p,ute lhe .
.., val ue of P .' ' ~/:''/ ," . " :,
.
23.1-)

~~'
~'"~'f

. 'T! ,his.

.r

a ~~wioo morfen t

l :. -.,vhen resolved

- . "'.
"

f-1'
I

".

ot I\

Mo" P><(3). ,,, .3oofi-lb

Px. "" 300/3 :=;,100 lh. ~o ~he rii3hi.


al 13

.. , f, ,;>(
0

q:,r'n

fl

when. reso)vea

Mo Py(6) ' = 30ofl-lb


Py= 300/6 - .so lb.

'

-P=/Px 2 +Py2

~32.) In . fiq P -~~t, . }he'

rnomen) of o

.. /1001 ... soll

certo'n

111.Slb

tcwce' F

is.18<? rt -

.lb o(ock..v.vi.ee .'.abou} 0 ~ 90 rt - lb ooun ~erclocl<..wi.se obou I 13 .

" .If 'its rnomeht,, about II is z.ero, delermine lhe fore.e .


refer./ io the figure' .231
When .w e resolveq ot. /\
Mo== 100 fl< (3)

fx 7. 60 lbs.
when we resolved of 6

-F;(ah fy(6)
fy"' ;+.s lbs .

Mo<= 90 =

~ .r f'> 2

"

Fy 2

..

.;.-6-o-~-.-.,.,-a""""'

f'" ?sib.

tone-

= ..;.7'60
-& "' f-on- +.s/6q

-fr s -.36, SZ

'

'

fig. P-2~, a force. P inlerseds 1he IC .o-x1s q) +ft. to ltle


righl' of o. If it6' incmenl obout /\ ie 110 fi-lb counlercl09k.y..I
_.., ils ~.menl ~l J3-~. 4<l fi -lb. clocl<.""flse, derermine il,G y "

. 233 ,)' In

"

inlereepl".

"" Ta' .

.,

when ~esolved ol pl. C


.
. Me---P_y(:l) .... Py Me/
P.t t<>'fl..194R ~o lb.
-M"'" P:r.(+) t P.;.(.a) "170fHb
. 201b(+fl) 1- P1'(3) ., 11on-1b

B_

f;o(" 30)b-.

.,
.

so Ly .. -t(w) ~ Lx'"' 2.67 ff belovv 0

~ poroUel. ro:.ce S~G".lem o,d G. Ol1 U1e lever shoWn '1n Ff9.
}',-236. Del,e_rrnine, lhe mogn'1l~do ~ poBHion of }he reeullonl.

*'

236.)

,' .

6Q)b. :alb.

!!!Olb.

40lb.

1.-1 .... I

, \ ---"- -

I .

.
'
.
'p,- -3-0-60tW-'1o -ilQ)tJB_, .
M,-. .. 30(~) t 60(9)- 2.0(7) -t (11) +o
.

=;

66o lb-(t .

'

,;_,,;. w'A(dfr.A)
'

dfr.I\ .. 6 ,6 0/110 .,~from A

'
I

'

,,

resultar1 ot t~ . four porolfel forces o~~


ling . on lhe rocker orm of f1'g. P - 237 '.

237) Determine lhe

!'11-+M~, i' '


11

+ 40 t 20 -601 "' -:-.so lb: . '


~Mo"' -.so(6) { -fO(~) - 00(3) t _6 0(8)
..,.,200 ft - 1b (cw)
~Mo c R ( d fr.
~fr.O .. ~oo~ 0 .~ 4/i. -righf of() .

R ..

.C i;o

o) .

z3a.) The beom /\13 in fig. P- 238 .supports o food w_h ich
varies from on lnferi.s-ity of .so lb. per n lo 200 lb per fl.
Colculate the mogn'dude ~ position of l he resuilonl 1009rt int : Reploce )he given loodinq by o uniforn:'ly d istr:ib.uled :
lood of ~lb per rf. plul> a _lr1ongulor lood vory;ng from

..

' ~.

'

6'

p.-1~(!SO) t Y2(12)(1i:>C>)

B 1soo lb (downward)

Me - goo(+) - 600(6) " - 7200 fl -lb.


... +.e Fl Jefl of 8 .

d 1200,1(900

ia~.) The 16 fl ._ wi~ of on oirplone. ;c;r ~ubjeded too 1irt w/c


.
vorieso from
of tho t ip to 360 lb per ft of the f\Jsew;~. ' J lone occor0ina to w 90 )C t'.z lb 11er ft where x is;- r-neosured
fl"""' 1 t O:::J
~
r~
.
~: . ."rrorri the tip . .Cornpvl,e fhe ~ultorf 11ij its l6salion from the

zero

:.~'

'i(Y'.. . wing tip

tt:~~( ..

t/'.li

'

\f 1 ' f '

' .

" r'

. 'fb(M)a/I - go
.iv,

Mtip

J J wd-;
(

'h
3686~
3&-t<>

3/2

ae-10 lb .

aoxa.t'g

go(~
~
o.tSo n .

: 90(16)a.tt _

d ..

-- '

16

16

go,,.df]:.

Rfwdx':;;1 Jo"''9ox'1t1dx
I

' ,

d~ ~

go

x.!V.q]"

!:>/'2

3686+

l>

n - tb .

The shocled oreo in f ;9 . P - !l<fO represen l~ o efed plolo


ol unirorm fhickne66 . /\ hole of-+ in . diornefer hoc been c u l
il Iha p fol.e . Locote the center a gravity of the plote - Hint :
The weight or the plofe i~ equ1vo len~ lo fhe weight or Hie
ori91nol plate minu~ the weight or the moter1o l cut owoy Represent the ori91nol weight of the ptole by o downward
force octinq ot fhe cenler of the 1ox1+ in. r.ectongle . P.epre
sent lhew61'ght of the moler-lol cut owoy by on upward foroe
2<t0)

or

'oot1'ng ot !he c.enter or the 01role . Locote the position


the
re.sulfonf of lhe.r;e two forces with re.speot to the lef'I- edg9

.,

'

~'\i .)

...
'

'

'

'

.:, '

'

'

'

' '

I\

weights are proportional to-breas 'ir

m'777.7:'7)'7777:>'7''77nj

i- thfcKness' '*' dstii;/t..,


~

_l

.A.Pr

; 16 ' I J

~'""""'

r;..L.<-<...<:..<..L..L.."-""".......

....

10(1+). _,; 140 in~

.AH - .ic::!12

",.

....

'

-,

'

' '

1~.. 01 in~

"

f-(fn~t 1~ -1~.~7

,.'.' '

~/;

127."f-:3.

in~ :

/\net y /\P,1 ':Ji - J\11 Y:z

/\net i< ~ ~1 - J\11_ ~2

y ' = 11'0(e)- 1:?.57(6)

X"' 1'1<>(:l)-1:z.s7(9)
127. "+-~' .

j' "'6.803 In

arle c.on'~tonJ .'

1'27....3

. Y .,; -+.go~ oboxe fhe ~orn edge.

..

rrom lhe leffedge.

: 241-H.o~fe the ;oinouril ~ P~!fion .:or: lhe res~\font or. t~e


' loa'd6' ocling on Jh'e fink. lrvss ~n~n 'in Fig . P- 2+1.
;.

400/b

. '

''

' ' '

R= 200 :Joo-t.~ 1300 +200 +. 2000


R .. .34-00 lb(direofeq QQwnwqr-Q)
tMc .. p..' (s)
I
.,
1

1_i001' "'?t'"1<'0 tSOO

t 200

~ =

1...00 lb(direote? doNnword)


:tM_c 1-t00(s) = 7000 ft - lb
Rdc .. ~Mc - ~ -= 7000/3400 ,. 2.06 ff rignt of
cor

:or;,. ..

d .. 12 .'o6"l:l . right
.
M~.) Find the values o( P ~ F so 1hal lhe four fOrces shovvrl
in fig. P-2+2 pro~uce on Uf"NOr~:i ~sul!onl of 300 lb octing
ot +.fl. from the lert encl of' the bar. .

~Mp= -F(3) + ~(s)

f =

200

-t

100(2) :;iQO(~)

lb ( dovvnvvon::J)

,:If.MF"' aoo(1) ~ P(s)- 100(!!) - ~cx>(<z)

P - -100 lb

(upward)

;;ii

... .

~~E:~~:;::,:):. >.-> '.... . . ~ ,. ~ . . . . ~~.


. =.;. - .~ . . ..
rt.+' '.r':2~S'.); .T,n~ ,_.,~ultont of three poro,1,l~'l. lo'Oqs (one load it.~P,issinq

ih Flg.P: z-+sY is .ao lb

pute

the rnognifude
WIO

10 ft .'to

9cti'19 t!p .ot

'ihe[-1ghf,0CA. Com '

~ po'iit:on of t'he JT1~nci: I~.

6011:> ,

. t-2'-+-f-z11.
'==f
--

R- +o ,

60

tF

-f - -+-O t 60-SQ - ' 70lb.


F ,. -70 ft> ( do~n'rY'~~)

.tMA
.. f\ (10)
1

~-70(x)t 41<>(2)

60 (13)-;. aoo

:x=Brl from ~.he right-or/\


!

b '"

~s.) R;..rer

...'.

;: i:.: .

or 60 lb

~.

.. (

toe ngure . /\ COl,lple

to

e,aoh ..One

ror-oo

consists

oct4 up through

or iwb '/ertico,l forCes

1
/ \

~- .the oher ocfs

..

thrt?u.g.h D . Tr;o.nsform the eovp'7 info:c;i.r'l eq;volent couple

te:,: .. vin~ (loriwn1ol f~s ooflr;'1g. th'roug~ t 'ls..,f:


"' .~.

'

:.. ' " . '

,,, ' '. ".

1 /~;::'.,,

<X?

" ,,.

_'

' .

'

,..~, ' ,

--.

.
h1 , .

'

IJ .
2'f-6.)

Deformirf,

sy~iem

or

side . ao1b

'
I

1?>

,,:>,l 3 ! '

.... .

c.

II

-...,____
~ .
~

VJ ')..'

lhe

'

')

no-

'

" , ,
CA.b- 60.. 2 -120,in
- lb ::Cer
."
Ce~. r(s) .G:129 'in:db . 'i
. os showil
f -; -+O lb act1119
.

resul!ont. mo~nt obout po~nt ~ of the

force1; s hown an Fig. P- 246 . Eooh .squore


by resolvin9

IS

1n. on 0

lhe forces inlo ils .x ">Y

cornponen ) s :

MA wo(s4;)(3) + ooo(ll/-lfa)(~) tao(a)


-(eo)(1) 'IOO(if,[4)(3) -100(1/5){-t) t 1oo(a)
(1/.f:i' ) - 1oo~k:f)(-+)-~oo(~)(~)-1oo(~X1)
' 4!M"' ~ s6C.8 f1 -lb cw

Z_.7.) The three-step pulley shown in Fig. P-2-+7 is subj~cted


to the given couples. Compute l~e vo)ue or )he resultonl cou
p)e . /\)so det errnine lhe forces ocli'ng ot }he rim of 7he mid-

die puHey
so(n:

lhol ore required }o bolonce.


ao(12)-<t<>(t6)-60(e)

~he

given sysem.

~c

-760 in-IO (-imeons Coutlb-CW)

f . 76%t 63.a lb (eouple


1

1c;

or ) }his much

required

40!0

.M e.) To close o goJe valve i} is necessary loe-,ce,..} }wo

'"-

of 60 lb of opposi )e side" ot o hondwheel 3 fl in diomepcciden~ }he whee.I ie broken~ 1he volve


mu.st be closed by lhrusting o bor }hl"0t-9h o .slol in lhe
" volve .S}em " C?1edin9 0 fOi-ce 4 ff. OU} from }he cenler.
(;,t!!I'.;

fer-. Thro1.J<3h on

Dolermlne lhe force required ~ draw a fr-ee body d1~rom


the bor.

-or

601b

r-- ~...'__________,J"""--hond

r;

i601b

whee.I .3ft. in diome-b-

4p 60(a)
P= 46 lb.

~.) fig . P- ~40 represen le }he top vie""' of o speed n=ducer which i<: geored fer o four lo one reduc.im in speed .
The lorquc inpul o) Ille hori:wnlol 6hon C is 100 lb-rl .T
torque ovlpul o) }he bor1zonlol 6haf} D, because of' ~he~
peed reduc)ion, is 4fOO lb - fl. Compuie lhe forque reodiOn
o) lhe mounling bo)ls /\ "'.> B holding the reducer lo lhe
noor. Hin} : The lorqus reaclion is caused by the unbo lonced lorque, which is o covple .

..

C c 30R

(-too -100) ft-lb - (30/,Q) R

!3'" 120lb

d1"reoled verlicolly
up ot /\ ~ down ot 5.

2.so.) The con! i lever truss shown in Fig. P- 2!50 corrieG o


vert1col load of Q400 lb . The truss ,s supported by bearings o) /\ 9.., f3 which. exerl }he forces Av., /\h, I/; Bh. The
four forces shown oonl 1h.ie two couples which must hove
oppos'1 le mornen~ effecls lo prevent rnove-rnent
lhe trus-s.
Determine rhe ma9n1~ude of lhe -supporting forces.

or

Sh

to constiiute. o c.ouple
Av "' 2-'tOO lb (upword)
2"'(6) - Bh (4)
Bh "'Ah "' 3600 lb

in order

~400lb

A.v.

ono}her verti~I F ot /3 ir"I


fig . P-2s1 p~uoe o resulfont ol 100 lb down ot D ~a
ce1:1F1iePcloc~ise eouple c
2o6 lb-(1 . Find }he mognifude
2-51.) I\ verf 1co)

force Pol II ~

or

~ direction of foroes P ~

,._

/".C,.!ZOOlb-fl

~~

:1~lb

=~B
;i'-

F.
(2001b-f1

lo

4'

I\

s'

R1oolb

~MB" 100(1) =-200 .t P(3)

300 lb. (downward)

.fMA -200 '"'100 ( +) ~J ( a)

F ~
l::f-): ~ 2.t't)_,p[ ~ )
p.,._

"3~

200

lb (upward)

2U.) A Wc.c ~ern oonsisls of o cloc:f\wiae couple of 1-901b41

plus o 210 J>- foroe dirccled up to lhe riqh\ lhrouqh }he "9in
of 'X ~ Y oJr.eS al -&ii: ~ 30-. ~ lhe given syslem by on
equovolen1 ~le fOrce Ii; cvmpute . lhe inlercepls d' ils
rinc of ocl ion wilh ,lhe 'X ~ y oxes.
R,. = F,. ~ 2-t0 cos ao
= 2a7. 8.S lb. (to lhe righl)

Ry= JY

= 2-fO .sin sc)

.._

-120(~)

M C ~ F)I i:,-

iy

=~ -

2.31

fl . ob<Ne 0

~85

i..x =~ 120

.... n.
.

tell of o

ae;;,.) Jn flq. p - 253 o G)!Sfero of fCJrco; reduces loo downward


verla'.cal Corces of" -f> J> lhrough A plus o counlerclooKwiee
couple of 800 IJ-fl. &Jennine }he Sn:je Jorce. Jho} will pro-

on equivolenl effect .
:5..- - - ----.,J\

<M;:e

I
I

112

R F = -too I> downward

MoMR

: .,.

..foo(-+) - 800 = -100 J(

x :0 2 n rlqht of o

if the sysfem reduces too leff word honz.onlol fOn::e of 300 Jb }~ poinl I\ plus o
docltwise couple of 750 Jb - A .
%54.) Rework Prob - 253

R=F = 300 lb

lo the lefl

MR "" Wlo
300y =.300(~) -750

y
256.)

= -

o.sff (f-> meonc; below o)

A short c.ompreGSion member carries on eccen Irle

lood P

OG .Ghown

200 lb sa1uoled 2 in. from lhe ol(is of the member.


in f~ . P - !lSS . Jn slrenglh Of mo}er-aols il is Jen-

rned }ho) lhe inlernol slre<>Ses o re detd)rmined from the


e<:tuivolen} ox iol lood ~ couple inlo which P rnoy be resolved . Delermine }his equlvolenl O'lliol lood ~ couple .
.l!-~ Pwo)b.

200

with lhe oddihon of o poir a\ opposae


oxiol loods eoch ~uol lo 200 lb ; wo get
F.. 200 lb (downward)
c - ~oo(~) 41()() lb-in cw

ii

256.) /\ verlicol shaft /\B iG srt long ~ bolted }o o rigid B"uppor l al ils lower end /\. /\ l i"ts upper end B .IG oHoched o hori:zonto1 lxlr f3C which is 2fl lonq . /\i lhe end or C is oppliod
a force P - 1801b. forc;e P is perpendiculor lo lhe plorie c:onloin;ng points A, a,~ c. Delermine }he lwislln9 ef'recl or Pon
lho ~art .AB "> }he bending erfet;} ol pain) A .
F l~fb

T""'iG}inq effecl

= 190(~)

= 360 n .-lb (I

,.

8end1"r19 efiecl -

11'?'~

100 (s)

goo n -tb

of

257.) Replace lhe sys tem


forces actinq On )h8 frorne in Fig .
P-25' by o resulton) R o) /\ ~ a couple oding hon"zonlolly

Jhrouqh /3

,.

2010~

~ C.

+'

:solb

c
601b

R=.:.r

= 3 0- 20-00

.P. ~ - .50 lb (t-l meone downword)


~Mo

"'C
60(-f) -w(1) "F(2)

F .. 110 lb llru B ~C os shown

I'

211s) Replace }he sy"lem of' forces shown in fi'g P-~a by on

oquivalenl force through 0 %o couple oding Jhrouqh A ~ 8.


Solve if thB forces of Hie couple are (a) horizonlol ond (tj.)
verlicol.
F,. =

'

F'~l--k--f-~8
j

,.

"r-........

n+tpn~h

361 fb

141.+(V.n) t 22+(.JS) -:s6t (o/Ji3)

~F.,. a 100 . ~tbOo }he righl)


.tfy = 1f1.+(Wz:)-!l2.+(1/,rs) -.361 (o/.J6)
~fx~ -300.S6{<->means Gl"'rw,.ord)
R ,. /(100.01)2 t (-3oo.'!J6)-i
3 16 .79 lb (do.vn lo the right)

= Fy/.:i:F~

e/C tori' 30t'.),s6jioo.og


e-~ = 71.se

o .)
p

C =~Mo
3F "'361(o/.JT9)(1)t361(.a.413)(1) t 141-41

(1/..iz)(2)-141.+(1/..rz)(s)'- 224(2/-19)(1)
f
0

-2~+(V..rs)(1)
, !3F c 100 .1

. F =33.37 lb
b~

4P "' 1001.1

e=

2.s.03

1b .

260-) The effecl of o cerloin non-eoncvrren 1 for<;e Gyslem

{.;de.fined by 'the following dalo: :1- +golb,eY= -60lb,ond


.iMO = 360 lb f1 COlJnlercloc'kwise. Oelermine the poin) o) w/c
'the res1,4lfonl in lerseds the Y ox is.
0

iy .
i.y =:: 360,40 "'

2Mo =~f,.

ft

below.

261.) In 0 cer1oin non-conctJrred fOrce &y!>lem i} is fovnd


lhot ~x = -60 lb, ~Y- +1601b, ~~Mo., +ao lb rt In o counlerclockw 1r;e sense . Oererm'1ne lhe paint' o) which lhe re6ultanl inlen;ec)s lhe x o'llis.
:ZMo ~ .fy t;i:
Lx == +ao/f6o ~ righl of Orig.in

26U Delermine complele)y lhe resullon) of }he forces ocling on lhe s)~p pune~ 6hown in fig.

1SO

250

~r,. 750 COS.Bo

P-'262 .

.. '250

89Q.s21b(lo

.fy -

lhe righ) )

7SO s 1nao - 1250

- - 875 lb ( - means downV'IO


~F,, t ~Fy

(899.S2) a ( - 87s)a

R 125+. sg lb down lo r ighl


-&,. }on"' 87S/B99.S2 "' ++.~1
Rd ~Mo 750 (u.s) - 1250(0.s)-280(1.2")
Rd .. o " d oj soR po~es }hro.Jgh lhc
oxle

215&.) De}er mine lhe resullonl of lhe force ~~}etn s hown in

fig . P

- 26S ~ i}~

'lf"

y inlercepl~

It~
[8GV .sinso. aet (V.Jj3)-224(o/.fi)
1+9.W(to lhe..riglll)
~fy 300 cooao +22'9(1/..f&) - 361 (/.Jii)
:: s g .61 (upward)

Tt"-~-t--t--'
"""N:: . -"\

.224 lb
.a61

lb

R c ./~F~ 0 t ~Fy ..
/(1'tQ.g)2 t (.S9 .61) ..
161 .318Jb ( up to lhe ri9h'l)
ton&,. .EPy/~1">< ... .59 .6-y1~.9
-&" fon-1 .&Q.61/1-.9.9
-0-x 21, 69

~Mo - 300 .sinao(2) - 22~(1/~)(2)-~1(o/.fill)(1)

lx =

-100.6 0-lb

(- means Counler CW)

1006/ .s0.61

1.67 ft r19lil of O
~ 100.6/1+9.9

i.y

- 0.61

below' o

Completely d ::>lermine lhe reGullonl w ilh respect


0 of lhe forc.e syslem shown in Fig. P- 26+.
26+.)

IY
I

:if,." 141.4(Y,J2) t

300 sin6Q i

ro pl.

260~%i69)

- 240sin30 1:79.Z.Q lb..

Fy.,. 1.+1.4fi/.J2) t260(.ru'3) '1"2"40 co~ao


-soocos6o .. 257 . 99 lb,

R - ./:.F,. 1 + .:EFy<t
-1(-+79-79)" .,. (2.S785)~

,.

B
'ton Eh
-&,.

s++ .68 lb (up lo lhe r19hl)

~Fy/~Fx

= lon-

2s1.03/41g.w

-&x "' 29. 2.5


~Mo"' 141.+(l/..(l')(3)i sooc,os6<i("'f) t 300 .sin60 (+) t 260 (134a~

- 260(-..raX-+) = 1779.19 ft-.lb CW


d"' ~Mo/P: "' 1779.19 .., 3.27
5+4-68

R
'

fl.

~.) Cornpule
2~& ,

lhe resullonl of lhe lhroe f0rce6 .shown in fig PLooole ils irilerseclion w'1lh lhe ')( '5 Y a;tes ..

1'

,. '
- -

lor,&,. Efy/~Fx
-Ehc = }on-1 810.3/810:1'7

= 3i:q

.ific"ago(1 ~si) t mz(~)-(300)


(&in.30) 810.~7 lo the righl
Uy= aoo(-5,fa)- 722(2/..[13)-300COS!IO
-510..s(-moons
downward).
lL
.

n2Jb

R ..

./~F,..a + ~Fy2

/(810..,.1)2 -t(-.s10.a) 2
R 957. 97 down lo righT

g_
~Mo =- aoo(12.A3)(2)-.3.90(&A:1)(s)+ 7!Z2(2/.Ji3)(4) -300sinao(3)
Mo= 1121. 97 fl-lb cw
-&11

l.;.c = rnzt.97 - 2.2

1.

s 10.a

ty ::
R

11~1 . g7

910.47

n.

.. 1.38

righ) o) 0

n. above 0

Deter.mine l he resullont of lhe lhree fOrces oclinq on lhe


dom shown in fig .. P-11.66, ~ locale '1ls inte.rsed1on w'1lh lhe
bose 118 . for- good 'design. 1his 1n1erseo~ion shou ld occur '("/i'n
lhe middle third oflhe base. Does '1 l ?

26<5.)

~F>t = 10.000 - 6000

cos 30.

= 4903. 95 lb(lo

lhe

right.) ,

-ify
P'<10,000/b.

-24000 - 6000 sin so

; -2?000(-meons dONn-

6'

R
l\ i---

word)
<I

(l

= ~F,. ~ ,fy

II

=/(-+003.95)4 ~c-21000Yz

-e'J( = ton-1 . Fy/:F-x


~ >n"1 27000/480.3.85
-e-,. ~

= _2_i'4~~;.,Q.gj12,;_(down

io the

r!ghl)

7!1.91

~~ = -24-000(11)

10000 (6) - MOO(+)

= -229, 000 fl-lb (- meons ccw)


~Me= ~Fy "1-b

Xb

229000~7000 - s .-+-t ft .(from the left of 8 ~


hence w1lhin the mi~le third of lhe bose)
'

267.) . The Howe roof truss shown i11 fig . P- 207 carries the
given loods . The wind loads ore ~rpendiculor to the inclined mernberS . Determine lhe mognilude of lhe resul ton l its 'sndi'no-lton w'ilh ~he hor1zo11tol. ~ where , t inler.Geds AB
.Zf,. 2000 t 44so(V..r&)

S:Fy

- 4003. .5_2 lb
~ - 3000- :2000 - 1DQO -44-a.i~A!f)

....:_.:.,10007 .03.!,b.(- meons down -

word)
sooalb. 200011>. 1ooolb

R;i.

~Fl<;'L

+ ~F/'-

...J(400a.s2)>J. 1- ( - 10007.03)>1

.. 1077~_.17 ll:>(d""n lo }he

r ight)

ton_, fy/1'Sc - ion- 1 10007.09/-t00a:s2

-9->C

-e-? .. fJB.~..
LMA .. +f-80 (1/~)(s) t +'-90 (1"6)(10)

2ooo (s) t 30006<>)

, 'looO (20) t iooo (ao) .. 160081. 12 Q.-lb.

}( .. 16ooe1. 1~Aooo1. 03

16.0

n.

r-iQht

or"

268.) The resultonl of four force.G', of which :three ore sihown.


in fig . P- 268. i<: ot o couple of 480 lb-ff : clock.wlioe in aens& .
ff eooh .squo~ i<: 1 0. on o sido .. deierm;ne . the fOur~h force
completely .

!v

1101b

~,..,.

r ..

- R"
110 ..

F'1 Rv

Fy -Hio t 1so(""t/s) .. o

"

//

1so(3/s) - o

r,. - 'JOO lb . (-meonsi to the le(\)

'
/

"'--- ' - - - + - ' _ _ L

there.lore'
I" f>C '" 'loo

1Wlb

lb to the left

C '"Mo
+SO -Mp t 110(+)+120(~)
!Ar ' ~oo Q-lb (ccw)
MF Fd

d 200/200

n . obove 0

Repeol Prob. 269 1r l he reoultont ic 390 lb dirooiod doNn


to the rig hi of o slope or .s to 1Q possinq through poin~ t\ . Al
$0 determino the x ~ y inleroop ..~ of the. rnSs-inq force F
269.)

flO

i50

,'

R.,,

r.---1--- ,+-"--'
'

/~
II

.,. "'-..
--120

~F,.

390(1'/13) - N t uo 1so ( 3/s)


~fx 169lb ( to !he ri9h~)

Ry ~fy

390

- 390(8 ...t)

.. Fy -120 +160(+/s)
- 1so (downwor-d )

RF
F~ .:r)I ~r 2 - (160) "" + (-}So)~
7
P !219, 32 lb (do.Nn to right ,)

-6!

= -S15-

MR ~h4<>

390{o/ia){3) + 3'N>(1~)(1) "'; 110(+) t 1!0(2) t Mf


Mr ..itJO
lb cw
b "" -t<)0,4,o - 3.27 fl_ righ~ of 0
ly ~ -t00_,4M> - -:a.067 fl - obO"B 0

n-

th-cc fOrcc.s '6hown i1' Fig. P-Jt70 or& required to


ho"i2()0tol reGlJltont octinq fhrou<jh point A . If F = .
316 I>. doteNnine fhe ~ues of P,, T_ Hrilf : Apply MR-~
io ~ier~ R .. then MR- 8l'IC to ftiid P. ~ f.nolly either trtR..=ato or Ry -:EY to c;ornp.ofe f .
210-)

The

(t.ll.IGe

#F
M9 - ~

;
1 - - - 4 - 1--...._........

'T

- ,,

8
/'

"' ..

~p

. ,.,_ !l10

-316(1/.Ji0)(t)t'316(3/-.1ii>)(2) r R(t)
R.,. -t9'J. 6+ lb.(io f he ric:#'t.)

"ll"' ~Mc

...,.,.ff(.3) ~ 316(1.Mo)(1 ) - 316(o/.Ji0)(1}


-t P(~)(-t)

p- 1'7'4-82 lb

:rn

t.tq ~Mo
ff9.6+(a) c -1(-Ae)(...)f 316(~)(:t)
.. 316 (1/-11o)(1)

T- - 225-18 lb
Dt} The .lhree. force6 io Fig P-270 creof& o vertiool reculionfocting fllra.gh point A _If Tie l<nOWn to be .3611>, conp>fe -the vaIU06 of F .., P .

.::E:hte - M"'-361(~){_c)t361(24;a)(_-t) - R(e)

R .., :t00--19 lb ( do.,.,.wvord )


~=~M&

-100-49 ~) = F(3/.r.a)()- r(~X1J

r- ~sa. n lb
~T =MR

p(245)(1-) .. (ua.29){-W.-0)6) - 2S3 - ~(~)(1). -f-00. +9 (+)


P - 1cH . ..st lb -

Chopier 3
fquilibrilPl of forco

Sy~fems

30fl.)

Tbe cyhrder C i"' fita p-ao2 weighs

1000

lb. Oreiw o FBD

of cyhroer C 9'j of rod /\B.

I.,,

GY

i:

+
-+-3'

wc:;~1o001b

Ah

Av

303'.'j 1he u~forrn rod ~I'\. f;9 . P-aoa we~l-w 4QO lb


hOG its
Gef'lie.r of _grovity o l 6 Drow a FBO o( '\he rod . tle.91ec~ the.
ih1cl<.ness of the rod ~ assurne oil contool ~urtoces +o besrroolh.

304~ The fl'Orne .shown i" Fi9 . P-304 Is support~ In. pivots of
A~i? . Eooh member w~ghs .so.lb per fl . Drew~ a ' FBD of~c.h
rr-ornber .

Sv

!IC5 .) A 600 lb lood is .suppor'ied by a coble which runs o ver o pulley ~ i11 fo6tened to !he bor De in f .rg. P-ao.s. Oro
o FBD of bol"s /IC ~OE ~of the pulley. AssL'.mc;. oll hinge,s to be -smooth ~ ne9leo1 the ws1gh~
eooh.

or

bor.

'._

-i-

-+ @ ~~
1

10

tAv

6'

"'

e;oc)lbt

Ch

c:

...

I!
Ev

ao6.) Drow o FBD af pulley'1 E ~ D ~ of ~he bor /\ D .shown ~ Fi


P- :a06 . /\uu me o II h (n,geg to be smoofh ~ ne;gle.ct t~ w01ght of'
~h bol"'.

309~ The coble~ boom shown in f1'g . P-aoa support o load


of 600(b. Oeterm1ne the lenstle. force T In ihe coble ~ iho

compressive {Orce C in the boom .


t MethOO I (uJ:;hg hor-izontol "'-. verticol
Axes)

~Fh -o

Tc:osao C cos +s
.Z:f.,, -0

T.sn30 t Csm+s - 600

subst. eq_ 1 io2 (eq_uoto I in terms or T)


T .i;in ao + (icosao/co.11,...,,)(.sin4s') = 600

I =-

4.39. ~.3

lb.

C "1'39,23 (cos.ao")/cos-+S

C"' 5a7 .24{4 LtL..


Meihod !(using rololiorl o~es)
T

_s C

L fv O :

Csin7~

600 .a;,..,60

...: &.oJ7. 9'):.? lb


"Q.or,"t

,Lftico : T=60000S6t;)tCcoc;7S'
6CO

T #

t;Ct; 60' t

.sa7- 94.s; (cos 7.& )

439. 2.s lb.

Mel hod ll ( ui.ing force i l"iongle)

.909,)

J.. cylinder.

woighing

-.oo lb- i~

held 09o'1nG+ a .smooth in-

oline by rneons of the woightlesr> rod AB1nfi'g.P309.Det. P"He.t-

erfe.d on the.eyltd~

.
b

Fv 0: t1Qn6Q

lr1e.thod l (1.1Vir1CJ t\"-.'< o~)


FBO ol Cylinder

lb

~A..-o: P-t<>e>.:;os;6~+n~,o

p "'400~661'
p : .378.!!15 lb-

.~

~fh c O :

...00 &1~

ti~ . -+19 .605

"\18.60.S c;o46()'

Melhod J: ( using Foroo Tr.:.ongle)

Pc.osa . "J1co& sio


P =Ncos~~CX>G,'s

Psin'1s -t ti .sin a~ - ..00 @


.cubct-

+o

[Nco.r.s (~1n12s'.j/eo&t;. t
- "tCO -.-

N-

-HS.60S

HGin as

lb

p ~ 11 coi;as/cas2e

= ~79 . -25

lb ..

H = +is. 605 tb -

= -tte.60.S(cosasoYco&~'

p-

.378.36 lb.

a10) f. !10!' lb bo,c is held at rect on a smooth plone by a force P


1ncllned ol on angle f} w/ the plane as .shown ih fi9 . P - 310. If
e-" -ts det - the volue of P ~ the normol pros;svre H eicl"'ted by

!hop/a~

Mettoc::I I ( Us;;ng ~Y axec:)

FBO of the bloc.K

.f.,0

300 H sin 60' -

Ps ;n.+s'

Fi.o
PW' 10'

Hc.oc; 60.
P H co~60/cos t1j

~tx;t

Me thod [ (U6ing Rololed

"1.ttr.)

tioos60/co~
ti 1<1"807 lb

300 tfamcso' -

211.132 It>

M&ihod I ( us""9

-~

aoolb

fv"'0

.n

-::.. :..,oS. ::..r

ri6in4S' 900a1n75
H 409,907 lb

fOf'Ce Triangle)

ri 409.B07 lb .
p . 212.132 lb.

1'.Fh 0 0 : P

s rlcoce' -300c:O!: 1s
P "I09,eo7(eoc<Ki- aooc.ocnc

P 212.1s: lb .

;m.)

rr the V'Olue or p in rig P-310 j1; 180 Jb, dct()NT'l~ tho ongfe

e- ot w/c -,~

mu'*

be 1r.c.1ined witll t"e. Gmooth plane to hold

t~ 300 lb b0 in equilib-ium .

lJeing ~;no Low


000

Sn:(

1SO

:. -&- 33,s;.s'

or

lhe mognituclos
p ~ f nece~ry to keep
force '"r"em , shoV<n 1n fog. P - :a111 in equilibrium .

lllli.). Dolermi ne

the concurronl

-~lb....
p

.aoo ~n 60 t P .sin ao :zoo .s111 105


p -133. 2.+~ It>

: ._
cotb

~fh O

>

f aooCQ1,6a

Pcos .,o- 2oococo1os

f aooco56o" - 1 33, H.S (cosao~- 200 co& 1 05


F 86 .s7 lb .

Ja'la.) fig . a1a repr-e4;en\s ttc c.oncu l'rent force ~e.m ooHng ot o
j~nl of o bridg e t rvss . De lenr>;ne the voluOb ot P ~ E to mo"1ntain C<1u"1lib.-'1um

or he

{'o n:;os .
U&ing Rotoled A-,.os; Melhod
~fvO

ooinis t fsin75 " "tOOSn" t


F c 4 12. 4 as 10

200 s in1011

.:trFh O

Pt

4-00COG+s

2ooco>71!"-1.aoo~is"t Fcos~

p - 16'5.4tt lb.
314-.) 1he r ;ve IOl"ce& s hown ii\ Fig . P-31-1 ore In equilibrium . <hn
pule ille v oluec; of P '-. F .
aool>

~fh O

t-400 COS75 t QOOC066<>' - fc,os30' -1300~80

Qlbcofitule the volue. of F


p ,100cos11S t aooax;60" 126.33 (C1>ao) +ao0 oos3o
p .. - ..sa.1!2.S lb .
0

The :aoo lb force ~ the ~lb forc;c s~ in fig . P-3\5 ore


to I: h~ld in ~ui l ibriurn by o third force Footing ot on unknown orete~ w1lh the ~zonlol . Oet~ne the volues off ~&.
aia.)

..,.. ('-~
-t001b

~-o

300" <+<la a:>Gao- t- F C()IO-&-

1io1Jb5t . (!) to
300 -400

eq.

cos; .ao +(~ GJnao ) (cos&-)


.5tn~

-e- - 76. 935q:r

76 - 935 bolo-

f ., ...ooainao' .sine-

F
!316.) Oeterm"1ne the values

ces shown

1n

4DD

"'n ac

sin 76.936

~os.:ns

or the on_gleS

x-oxilO

0(

lb
~ e- so that toe for-

fig. P - 316 w ill be in e.qvilibrium.

!lOlb

-~

force Margie

by c=ine low
30 .

QO

CO& "(7-

40" -

2 ( 30)(40)Cc6&-

O. 875

-tr cos_,

0. 97~

f r - 2896 ' ... 29

by

~1n

low

_E2_ .2Q_
510~.,

sin<><.

.Sino\

[Bo (sin 29)]/20

sinoe. 0.1212
o(

4-6.65

The ~y.sfom of !W*fed ~ dwwn in


the 1ndicoted wolghts. C()mpute ~ ienGile

f'.'-4 P-317

/ :a17.)

support

fon:e io eodl<XWd -

- _c__ -_o__
~

..-.715"

6'11117!1'

c-~lb

o-

~b

ZFh'=-0
,A " C C:OS60 BGOG-15

e-1n-+6 aoo~:c 6fn60

- 400(<DSJ') t 9f+.162(cos+s")

8 300 1- 100(s1n60)
coin+&

I\ - 946.+1 Jb.

B= 914.1621b.

Three ba.-s, h;r19ec1 of A ~ D ~ pinned cat B 1' C os .6f1own


in fig . p-:s1s, FOrm a fovr - link. mcc;hQn16m . Determine fhevolue

:a1s.)

of

that will prevent motion .


using Rotafc:cl l'vccs

/\

Method

~--

~Fy,0

Ass..60 =
A.

!ZOO 51015

=163. 11:"9 lb

~fh=O

........ A

c '200 coo>"'fC

A C0661>

22307 lb.

Fv O
CGtn7s" P~+5

2~a.01

n 1r.

ax; +s

p " ao-t . 7 1CJ

using Force 1n ongle

~/
200 /

'" - $.-

$1060

""' ., Lt

cv;

fn
Pf"tlblem 3 19 .GOlu f;on .
~Fh

by aone low
0

Siil .?A

-~
ain ~

c - zaa.01 /b

10

+oO

~&

200

~ ~ 6o

=o
H 1 't<X> .sin e- = soo
N BOC - 400 s tn60
~Fv

H.

453. 6 ~ 4-S<t lb .

by sine law

-'--. - _Q_
Pl~

&ltl4!0'

p ~23.Q7{s 1 r179')
S tn-4$ "

p .

3)4.719 lb

Jlll'J) COrds ore loc:pcd Qn:XJl'ld o smo/I "J'oe&r soporofi"'3 ~wo


cylrnder& each weighing -400 lb ~ pas~ , as shown in flcp
p - 319 , over
frict1onleGs- pull"Ys to wei9h~s o f 2oolb ~ .otoolb.
Oeiermne lhe o~le '6- t,.., tm normol P'""'&ure N bof . the cylin derS ~ fhe s moofh horizonfol surfoce

aita:) The Fink truss shown in

fig. P-322 iG ,supported by a rol -

ler a l II ~ a hinge at 8. The given loads ore n0rmol to tne


inclined mer;nber>. /Jek"mine fhe re.oofronc, a+- t4. ") B. l/1hf : F<eplace ihe Ioodio by thei_r reioultont .
1000/b

>

-EMa o
60 IV.. "

'i1COO(sin60')(-+o)

Ft,.. .. 4618.B lb "' 46~0 lb.

~Fv

Rev t RA

Rev ..

Rev
i;fh

= sooo

sin60'

SOOOs1n.so- "'l-618,S
!l.309.-+ Jb

Rah eooo

~0

RBh = 4000 lb.


4

Re.

"

Rav'-+ Rel\" " (!laoq,4) 2 + (4000)'Ra 1(2ao9,.+)" t ( ..ooc) ?t


Ra : -+61s .a Jb :::- 4620 lb
0

fan-&- "

!l309, 4

4000

-e- "'

ton- 1

-e-

= 30

Rs "'

.<f620 lbs

!l<I09A
4000

ot

-ao"

wifh the

l-ori2ontal

323.) lru f ruGIO shown in


of /\ ~ a roller ol /3. }.

fig. P-323 i~ .Guppodecl by o hinged


lood of ~OQ'.J lb ii:: oppl18d. ot c. Dot.

A" a.

the reootionG at

c
'"""'"

~M/\

30

0
Re. fZOOO (c.os:30)(1s)
Re 65990. 76

l 2ooo(s1n3C"X<FO)

30

Rec

2199. 36 lb ~ tJ.200 lb

F1iO

1< .... 1-1 = 2opo Cos ao


A.--h -

1732.0S

-o
R>.v = Ra -

lb

.<!Fv

2000

s in ao

R--.v " 1199.38 lb

R,..,4 .o

RAh 1

IV..ve.

I (113~. os) 4 t (1199,35)e.

RA

R ..... " .2106 lb

Tone-

R11.v 1199-.3 5

RAh
'-&

1731Z

ton_,

.os

1 1 ~. 3 5

1 732. OS

-eR.11. "

2100

lb

-e-

3 -t. 70 ~

d own fo the
= 34-.7 .

left a i

2000 lb

.32+.) /\wheel of 10in rodius corrie~ o load of 1000 lb, os s~


in Fig.P;32.+..(d) determine the horizontal force P applied ot tho

cente,r which 15 necessory fo sforl the wheel over ihe .!I-in .


blocK . /\Isa find the reootion ot the bloc!\ . (l:l)
the force P'
rnoy be inclined at ony ong\e w1th the hof"irontol, determinS:
ihe rri1ni.rnvrn voluc of P to start the ""heel over trc block.; the
. ongl~ thot P m9kes w1lh the horii.ontol; 'It.., '\he reoohon ot

,r

the

btocK. .

sine- ~

It)

a/n -

pz

SAo ,. o.s

-e-- :so

o)

i:la

zfv1a ....0
sP" 1000(1090s:90)

P-" 1732:051 lb

:::EF>< Ro

pc Ro

cos30 : 1132.o&r

Ro ..

2.0oo lb.

b.)

if

P iG min imum

i~ wilt be .1 t o ~o

hence,

-e- .. ~o
.Ma~o

10 Pmin = 1000(10 CO$ 30)

Prnin "

B~

lb .

..:tFi< =O

Ro cos so

= Prr)in COSfio

Ro e[Pmin(a:>s60lJ/cos30
=

Ro

[8610(COS(;O")]/cos .90

= .soo lb .

~.!!l.r Determin e

the omounl ~ dir ecli.on

or fhe

.smollest force

p required to tori !he wheel 1n f 19. P-312s over block . Who f i6


the reodion o t lhr: block ?
P. c;oa~P
Py s;i11"tP

=1.74
,B=+1-+1

C-OS~
~1rfo=O

.tOOO(o)- P~(b)-fY(o)-O

.sin & .sin

MOO (t.89)- Pcoso<..(o.t.-+) -Psrn"(.(1.99) '0

PcooO((M-+) ~ Ps;ln0\(1.99) 3790


(o.6+) P(-slno<.) t o.6+ di' co~ .... ~ 1.~CJ P coS<><.
Cl.O\

71. -.1 : ~

1.~'l .Qe. .cin"<


cl

b . o.6tn.
fr =11 ...11 , 0

a 1.&9 N.
i

a,~71.+1 ~ ~

-&- 71.+1

'(<u+c.os
... t 1.99511"10()\ .. 0.6-t P51fl0(-1.B9Pcose.;
d""- .
~ .. (o.6+Pin<>< - 1.B9Poosoe) =
de<. (.O.,G+c.oG:o. i i. 9'311 1,....~

1.119 Flcosoc ,. o.6-ty/ sine<..

.. Ion o<,

1.9 9
061

<>< fon _, 1. 9 9
o:64f

aaoo

_P_ _

Grn9o"

su" 71.~9

P .. 169+ lb ot

R
.sin 18.7 1

71.3 with

Rs1n90 12000(1;1n1s-n )

R " 641.6 lb

~ 64-!Z

lb

the horizonfol

The cylinders in f19. P-326 hove the indicoted weights


~ dimenG"ions. /\s'1ul'T'ling smooth contocf .surfaces ~deter1Ti1ne lhe reoc tionG ot A , e, c, ~ 0 on the cylinders.

326)

FBD of tho bi g cylindc.r.


l'?c '

~fv'=O

ff>O

of

406 t Po Slll3(,. "' R~


"400t ...ao ,.,nat:I Re
Rr.- 6001tl .

small cylinder,

.:Fh-O

l10(:>1b

RA =: Re ccc;.:w
'Rio- 400 ~so

~o

Rio.

.346.-t1

lb.

:i.fv O
200 - P.c .sin so" .

.Re .; <4-00 lb
~-o

Ro = Re cor;.3'J"

Ro 100~~
Ro a6+.41 lb.
327.)

f~ P "'- F octing olo"":J the bon; shown in Fica . P-Y27


Deformine the volue of p "->f.
~,-~Mc,O

l!:!(f) emsa.1 .. 1so(ti)

~(1f)

f"' 390.1s lb
.tf.Me"O

18011
1o ()heOk. :

fco;;

~a.1

1s(P).s1n6B+ ... tSO(q) - 300 (12)


P = - 1.+7. 6!2 Jb (- meonG compre9eion)

Pco~6is..+-

:aoo aO

'a901S(COG&1) - ( - 147. 62)(c.o56a.+) -

doo " 0

329.) Two weightles'> bors pinned

toge-Jher oi> c:hown in fi9 P-

:a!20 s>upport o load of 350 lb . Determ1ne the force P 'llo, F octinq


respectively o long bo~ AB ~ AC tho~ mointoins eqvilibr;um

of pin

'}--o' ~;.jpcoi;~~,,~~

ton o< ..!L ; ex .ae .66


10

~MeO

l's.n"<

_ _.........___ -

(+)fcos;O<. t (a)fs1n"t

=s.sO(e)

-+ fkos3ill .66). 2 F(sin:se.66). ~o(e)

___L_

F .. M<>.1 lb.

~Mc

2P.so1np ~ 4Pcoi;~ a110(10)


!2P(e1ns6:ai) + !2P(costxS.a1)- 3.500
P~ 901.39 lb (tension)

~lb

'

'*I 200/t'> l'eQpectively,


ore connected by a rigid rod curv~ porallel fo the smooth cyliiidricol, .surface shown in fig . P-3!19 . Deterrnne the onglea 0(
~ -&- that defined th6 pos1t10n of equilibrium.
a,9,)

Two

cylinders /\~fl, wei,9hin9 100Jb

-tr~<><

.. 90

e- 96-.

~MoO

100J'cos0(

100GOSO( ..

2oorcos&

flOO

cos(9o- oe)

100~ -~oo(co~ e"~.s~)

100 OQGO(
~-

:200Sll1'JO Sin"\

to-l"( - _..:1_00_~

!JOO &1nqo

C-0$0(

<>< .,

e6 33".9+

~-90-"'(

'30 - ~33 '.s-i-'

-e-

6a. :.16 ,.!, e.

h2.) Oet.ermine the reochonG for the beom shown in Fig. P-332.
30011>

"!!i" 100 lb

OP

..t'.MRr

~(4)-100(1+)(9) tR1 (10)-300(16) e


R, = 1.Sso lbs.

rt . -.s

~fyrO

. R11Rj = .aoo+ 10<>~.._) -t ...oo

Riz - .s2q lbs.

Determine fhe reochons R, ~ Rl? of the beom Jn Fig. p-333


looded w1th a concentn;iled- lood of 1600 lb ~ ci load varying

3:Jaj

from z.ero to on intensity of


-iOOtYn

'

f600ib
I

't

O::MFc o (t!

co

:f!M"'1

f1 t fa

12'
R<

16 Ra "'16 F.c t 4F1 t 1600(:a)

eoo lbi:;.

l6R2

~r., - o

Ft

t'

3'

P.,

12F, - ""W0(12) ()
2

F1 -

ped1 .

400 lb

~ 1600(16)

t S00(-4) t

1600l3)

R<Z .. 2100 lbs.


e

i4co{12 )]j:z

~fvcO

{f'f<l<l(1l?)::th) - soo
f11 ,16oo

R, tR2

~.

1600 t

soot 1600

R1 .. 1900 lbs.

~ .) Oeten-n;ne the r-eocf iol\G for fhe beam loocled

06 Ghc:>NI'"\

n_9ur-e P-33.,..
"'MR, Gl

1sR2 ao(a)t 1&0(<1)(1a) -t

60(6)(6)

R2 - SS81b& .
.;:F., O

6'

Re

R1 t Ra

1:20 -t 60(6) t

1.50(6)
2

R1 - 930-SSQ

R-t .. 37~

lbs.

ii'"\

33S.) :rhe roof tro cos in Fig. P- ~ is supported by o roller ot A


o hinge al B . find the values of' Hie r-eacfions .

eoolb

:tMaco

so R..... = .S00(1o) t 600(10) +goo(is)


RA "' 966 .67 lbs.
~MJ., c O

:soRev'" eco(15) t 600(10) t .soo (Ila)


Rev" 9::E.33 lbs.
-2'.fh =O
l<eho

: . Re " 9aa.9albs .

l'l~".)

The conh'lo-.er boom shown in Fig. P-aa.5 rs built inlo a wall


1fi. fhiok. .so that i~ resf.s o.goins+ poirds II ~6 . The beam is 12'
long~ we.ighb" 1ootbperfl. f\ conce.nf roted load or 2Q:)O lb is opplled
of ihe free. encl Compute the reochbns o! A ~ 6 .
~M,..c()

2Re

!l000(1z) t 100(1~)(6)
1$600 lbs.

Rs :

R.....
R.-.

~Re - !lOOO - 1oofo~)


c

1 S6CO - 1?000 - 1

:zoo

R.. ~ 12 4<lO lbs .


337.) The upper beon'l in Fig . P - ;;i37 iG' supported by o reochon
ot Ra ~ o roller o~ A whiok separo+e.s the upper ~ foer
bGOmG . Oeler-m"1ne the volues of the r=;)lctrons.
4<XJ()lb

600b

19oolb
10'

4
10'

fl.i

600lb

+'

10 '

r~

'I'

~MR.,'<>

10t<11 - 6<X>(1.f.) -19GO(+)


f<11c 1~lbs

+'

10'

! +'

:Fy r O

6oOt 1900 ,. R.I\ t Ra

191)()1b
10'

fRs

Rs" 9001bs.

-o

Con6idering the loWer beom.


~

~MR 1 0

10~ - 1600(H-) - -teoo(4'-) 0


R11 - 38-40

lb.

ZMRit O
fOR1 t 1600(+) - '4000(6)

R1

lb.
on the page 69 are
to be rnoved 00--i zontolly wrth f"eQpeot to eoch other 7' lood

338~ The two f~

1160

n. beams shown ~ fiQ. .a-160

P shifted to o new position on CD so that Gil three reodions


are ~ual . How for oporl will R'1. ~ R:!!!l then be? How for w-.11 P
be from Df

:::iR 960 ..
R ::1120 .,, R 1 Ra =-Ra
~eririg

ihe lower beom,

considering fhe uppe.I"'. beam , .


9601b

11-><

fAc

f,,O
Re =R1iRe
320

P,.c -

~MRs

+S!lO

P(x') ~ Re;

(12)

P(x') 6-90(1 ~')

6-4 lb

960
~Rot

x = 6'f0(1!Z)
'l'

Re('() ,.. R2(12)

6+o(1a)

960

y .. 320(12.)

}(. ""en.

6-fO

Y "' 6f! .

. Ra \._Ra
S,....

'1s

'1s

60 oparf .

olao

en.

rrom

D.

3.99~ The differeniiol choin hoist .shown in Fig. P - 339 oonsietlO"


of two ooncerdric pulleys riqidly f'oste.ned toge.1hel"'. The pulley
from form _
two sp~t~ for on end le"' choin looped over them
in two loope. In one loop is mounted a MOYQble. pulley Gupporti~ o lood w . liE'-91eofin.g friction~ friotion doter-mine ihe mox1rnum load w that can just be ro'1seol by o pu'll PopPliOd as

>

.stic>wn.
~Mo-0

wfa (DI~) Wfe(d/2} t P(O/~)


w0,4 - w~ = ?o/2
w/+ (o-d) "'PD/~

W-4PO

!i(O-d)
w- !2PO
(D-d)

tH<>) for the 1em of' pulleye 6hown in t=19.P-3...o? d o 1er-m"1ne


the rotio of w to P to mointoiri e<fuilibnurn . Mc.glect o-i.le Pnct1011 ~ the weightG of the pi.tlleys .

W = 3P t3P t 3P

w 9P
W/p "' 9

3-tl) If eoch pulley s~n in Fig . P-3'40 v.eighs


fold P to mointo rn the eq11ilibnum .3W1 :a6 t

no

W1 !ZS2 l0

3P - a6 t

w..

P "(36 t !l5:Z.)ja
p .. 96lb.

a6 lb ~ w

12olb,

342:)

jeep oro_givcn iri

The wheel foods on o

f1~.

P-a42. Deter-

or the beam at "

mine the distance ')( so thot the reaction


twice oi; 9reaf os lhe reoohon ol 8.

1&

R.1..-~Re (Gpcoo fied oonc:li4o"or"l)


.:201YV.-O

IBRe ~ 600(x)

!ZOO(x t"t)

1sRe soox + eoo


~Fv-0

R..., -t Re
3Re

=- 600 t '.lOO

aoo .. Re= 800~

.avbG-titufe '1. in 1,
IS (904/3) BOO(><)
lC

-- -

900

=+fl

9-'f~) The weigh~ W of o frovellng cr;>ne ,,.

!JO

tonS

~f1ng as shoNn

ii' F;9 P-a-48. To pl"'evenf fhe crone fl"'()lro fipplris to the> ,..;!8ht when . corry;r;ig a lood p or 20 tons' Q covn !er we1_gh1 '6< is used . Det.
the voh.1e ~ poE>rt10n of-(,( <50 i hot the crone w"t/J rorrOir. in equ'i II-
brium both whe~ the maximum Jood P ;~ applied ~ when fhe load
P is removed.
Llmiling Concf1lio11s,
when

P=o

; R12 =o

when P 20 ; R1 o

P !teloro;

~M~s "O

(when PO

~(x) = ~0(6)
~J< ~ 1!20T.ons

- -

zMi=t2 0 ( -When P-20)

.subsh\ufe. . 1 i" 2,

151(-u5)

1QO t iS~ ~ 220

~x 1~&)

~ " 20./ons

From cq. Q),

-Ql\ =1QO
)(

1~0/20

.. 6

n.

20(1) t Q0(10)

= 220Tons---

A boom /\8 is suppo,..fed i"n a tior"1zontol posHion b-f a hinge


runs from C over o smol\ pulley al D as
sho"'{n In f1~- P -346 - Compute the. tenG1on T in the coble k the
hor'1zontal . '/it...verh'Col components or Hie re.aclion ol /\ ...Neg346)

/\ 'I..., a roble whiol)

lect the .size of the pulley ol D.


T

ton-& - 0,4

-e-

6~-"t-3

~tvl" O

flOO(fi.) I 1oo_(cs) - T (.sin63.+:i) (4')

T"

279.[?JR.

lbs.

.=;;fh-0

Rh

"'Tcos6a.4:1
a ( ~79. EJ~) ( COS 6'3.4-!3)

Rh

1!ZS.OQ

lbs-.

~FvO

Rv

J<v

Tein6a.+a "' ~00 t100


.soo - ('2.79. EJ2)( e rn &a."!-3)

Rv

= eJ0.06 lbs .

.'H7.) R6peot prob . 3+6 , r the cobl~ pulls the boom AB in-lo Q
poGit1011 at whion it is '1ncl'111ed ot 30 obo...e the hor'1:zontiiil. The
load6 remain veri1eal.
s1nao . JC/-t ton&' =6/+~ao

100

~fh"'O

~IVl" "O

Rh Tcos-&

4fT .. ~oci/J.)c,ot;at>0 i 1oo(6) COG30

Rh Q16.s eos60

Rh.

T .. !Z16.sO lbs.
~Fv~o

Rv+Tsrn-6-:: ~00+100

R" 300- 216.s (G 1r>6tl)


Rv =

!HZ.SO

lbs .

100 -!ZS lb.

.n

94g,) 1he f~rte .s ho-'I in f ig. P-3-te 'n; c;upported


p!l(()fli ol A"-.
B . Eooh rn~ber weighs so lb per
Compu~e the hor'1zontol re--

r.

oction oi " ' ol the hori"Zonlol ..... vedicol Cot"4)0Mnl~


t'!!OC..tion oi B.
Longlh

or lhe.

of ro .[8f't&"
Fi5 -

10('1.

:.:Ma o
.....,(112) =500('4) f (,f)O(,r;). 2000(11.)

12-466.61

tm

~l'h O

Ah 6h "2-t6S.61 lbs.
::t.fv O

a., 600 +.soo t 600 t 1.000


BY "

3700

lbs.

9-+9.) 1he truss shown in f19 . P-:a"'t9 iEJ supported on rolleNO al I\


\...., o hinge oi B. Solve for the compononte
ihe reooions.

or

6004b
~fh"O

Reh HOlb s .
~IAe

0
2+~ I 24-0(16) c60'.l(lt) t +o()(36)

R" ..

7-f<>

lbs.

~MAO

!I+ Rev 6lX>(11.) u

.R.,,

200 k

+o(10) - -400(11)
.

a5o) Compuie the tolol l"CO<itions of /\ 1!c. B for tho truss hown in Fi91.1re
P-aso.

.tFn"'O

Reh" 300 tbs.

MeO
.a6 RA. 600(.so) + 1a.oo(!20) - soo(!:Zo) - soo(20)
ft.- :: 100.s .71 lbs.
~fv

Re" t R... -

500 t 1!ZOO + .SOQ

Rev ~ 121-+. 20 lbs.

Re /(soo)" t (1e1't.28)"
11.1so. 79 lbs.
ton-e- .. n1-1.aa/!!oo
.. tt " 76.12
:. Re :c12.00.79 lbs up to the Jeri ot 11S.12
31>1.) The beam shown in . figure P- 861 is supported by o hi~ ot /\
~a roller on a 1 lo2 Glq:>e of /3. Deterrri1ne the resultant re0chonG

ot A\., 8. 1~'

::EM>. O
16 (~av)

Rav
~fh

"f<X>(1 ~)

~ 300 lbG .

By Rot10 \s.., Proportion ~


Reh
1
Reh - 300 (1/e )
l<ev - 2-"1601bs.
Re -.f1.50)a t (aao) 11

.. O

R.-.h - Rel\ -

335 . ....,

tbs.

mo lbs.
R.-. 1(100)" 1 (1so) 6

:EMeO

- 1ao.n lb6.

J6(R,..v) = 400('!-)
~v"' 100 1bs.

or

as2.) /\ pulley -1-fl. in diameter ~ suppor-\in9 0 lood


2<XJ lb le mcvn
fed at Bono hor1zoniol be.am (Ffg .P-3!.'l2): The beom is Gupporied bya

"*'

li1nge. al /\ rollers al c. tie:9leof1ng the weight


m1'ne ihe reachons ot /\ ~ C .

or !he beom, de-ter-

F~

f6D of tne beom,

of ths pulley,

~- o

a!Meo
r (a) . aoo(a.)
T: !IOOlb&.

Re

Re"' so lb6.
.ER,, O

R,.......

Lfv O

Re

Rc(9) " 100(..+)

200-

!100 - 200(11inao')

100

Pc

R..o.v -

rm:io'
~ f'h

lb&'.

100

loo.

"' ao lb& .
o : R.--h - 1 c:oe zoo
RM ~00 GO"I!:()'
100 -.sc>

1 73 .20 lbs .

Ro. (SO)<l t (173.CO)&


ton~ = sof11a . ~o

" 190. 27 lbG .

"

16.10'

RA, 1eo. Q7 lbs . ot- 16.lO"up tothe

right .
dam ore s~ i"
F9 . P-asa . The upwon:l ground reoction "'or'1es u nif'o l'mly from an
;ntef\Giiy or p, lb/rt . of A ~o pa lb/ 11 at B. Delermin~ p1t.,. f)2 ~ ol 211ss.) The forces oct'1ng on o 1-

n Iengh t or a

so the horo~\ol' res;sfonce lo alidong .

Assls!Ql"ICe 1-o .ilidlng


~fh 9 0

F" 10000 - 6000 cos 30


F= "'!"903.98 lbs . '\:: 41300 lbs.
~~

.... o

R 6(X)() .srn 30 t
27000 lbG.

zt.-1e:o
P, /ri Rx -= ~(11) t6000 (-t) 100X> (6)

ec

)(, .. e.++ n.
9 -Q.++ .. o .sd

n.

Ri

=Ria (1-6e/e)

-~[1 _ 6Co.e6)l
18
18 "1

P~ Rh (1t 6e/a) ... ~1~[ 1 ~ 6~~56)] -

1no lbs.
1790lbs.

35-t--) Comp..ile the total reoclions of ,.-.. i..._ 8 on the fn.;w Ghovvn
in fig. P- as-+.

'klOO/b

1%0 . .,, 11' j -e- =!l6.S&0


~,.... .. o
eoRe.v t <Zll+o( s1n-a.s.s&)( 10) 11200(10) = fl~+o("" ~6~)(6'>) t
aooo(6o) t iz000(-40) t' 1000 (ro)

ion~

fl&v ~ 41-627. 48 lbS.


~Fh

- 0 i R-.h ..

20fX)

t i'~40 $11') 126.66.

3001 -78 lbs-.

zMeeO
80 R..,..,. " woo(10) t 1000(60)12000(~) t 3000(ad) I fi1'tO(CCS96-66'X_'JI>
t

e'240(s1n ~6.56)(10)

RAv ., 3376.09 lbs .

R,..

./(?<x:n . 7s)c t(aa16. 00)2

to" -El;

-+.517. s7 lb;;.

3376. 09

3o01-75

--(% km_, 3376, o~


3001.75

-6i;" +e.3s
i<,._ " ""517.57 lt>s up to f he ri9h~ of ~ " 48. 36

3ss.) Oeiermine the l'eaoi10ns al /\ ~ 8 Or'\ the fink. fruss- .sho.v


in fig . p-3ss . Members CO ~ f6 are respedively perpend:cu !or to AE "BE o1 their- midpoints .

ton -e-

lo------ 60'-------~
,.4.0 , BF :~ 16.7ycas 26.lifi

t%o ;~. 26.!>6 '

/\C a6 -

1~/cos26.s6 "

l\C~ e G 16.77

AO BF f9,76

ft

fl.

~M&"O

R,..c.os:a0(6o) a 1aoo(....,)i ao>(+1.2S)t "f-000(16"17)


R ..... .S361 . ~7 "ti .5360 lb&'.

... -o

60 Rev

1!ZOC(1.s) t ~000(1e.7s)

Rev 61?11-.67

"Ii

-.000 (+1.:w) t '!000 COG'16.s&'{t1flll)

6130 lbs .

~Fh O

Reh "t<lOO 4>1n26.G6

..0004'1n2~.s6' -

R.-. e1nso
.sa61.27.sinao

ioo

Rah a92 .096

~6.) The conti lever truss ~1-iown in Fig. p-350 is &up1X>ried by 0 hinged ot /\ "'i 0 strut ec. Det ermine the reoction ot "t.
I

""e.

1oooll:>

By Rei;olving t~ forces lo ita eqJ1vo lent for-ce triongle,

By .s11ne low,

~-~
0

101nao

Re .. .34-6+.1 lbs .

s 1n9o'

357.) The uniform rod in f19 . P-357 we1.9 hs +20 lb"*-, has ,fs ~rit
of grov"1ty al {). Oefermine. the teMsion in the cable ~ the reocA~B.

iionra oi the Gmooth surfoce6 af


ti~,,..,

MA ..O

2T t <.20(6)" g N i;1n;\e t 911coe-


0

bul , sin -i1i

cos~

2T t 2s20 161is1n4e

e'

T-

rt COS-t5 '

- 1260

" B N s1n+s - 11260


~Fh= O

:. T ..
T -=

: T "n cos-ts

SUbi;f . eq_ :Z. in 1 ,

N cos+s

2 S 4.S6 (C.05-46')
1BO

H
~lllo=-0

e HCD&+s

- 71icos4e

lb

- 11260

-1!160

254 . .56 lbs.

2FV. 4~o(4) Nsin-48(6) t Nc;os-<1.S'(6)

R... "' 6

tls1n~ - 9-10

R" "G (12s.f. .-'6)(Sin4 5 ) - 9-fC '" i'40 lbs.


:;isi..) A

bor- AE is in eguil;b,..;urn unde r the oofton of' i he


fo rceG" shown 1n f i9. ,P - ~i. , Oe.ie.rmine P,R ' T.

r.:..c:.

F!..

a'

-4-00lb

:ttMA O

R (16) t T C06 &(9)i TGtn Et(6) ~ 600(-4)

r -400(9)
l6R t 9 Tcose t

'

-Fv

6T& 1ne- 6000

~o

TccGe- t R "' 600

soo-Tcos~

su bcol elf 2 in 1 ,
.
16(600- Tcos e-) teTcos&t6Ts1ne- scoo
16(soo-To OG36.91") t BT co s.3.:>-eJ i-6Ts1n&;.e7
0

T= 1!29.S.71 lbs.
Fr-om

eq. '2 ,

R - c;oo- (12e.s.71) cos 3<587

R.

4120 . 5 7

f!Js. ( upward )

600o

zf'hO
P t Ts;an-e-

., ~

p = -f00 - (1!Z96.7'1)Sin36.B7
P 371.<ff lbs. {to tile lef-1 )

Anfl . 1-- of f169li9ible w-eighf redg in o hor-Uontol posi410o on the emoo4h p lane sho....,... in Rg . P - 359. Compute the
distance 'X ot whi'Oll lood T-1oolb should be plooed from pt.

" 369.)

8 1o

Keep the

bor h()Nzontol .

fig.P--..,,

P---....s1

By Sine. Low,

Ra.

Re

"""-o

5'nao-

300
.&otl106 .

100>< .. tD0(9) ~ R-. C()f;~(t2)

1oo x - 219.Bt(o::J630-X1{l) - 1eoo


x - '"'t.82(1 .
i16o) Rererring 1o Prob 389~ who-I volue of T ocfi~

from 13 w i ll

lteep

the bor tl0rlzof"11ol ?

AE
Co& 60

00

s.7s0 .

g ;76

T (_.. 61) = 200 (1.39 )

T = 60 .3 lt>S -

,AC - + ,.99 '

= "t.3':/ - 3 .. 1."9f4 .

ui: - 6 - 1. 39 =

.z:Jr1c - O

o+ x

4-61

s f\ .

to Prd>. ~.,if T 300lb '6i.. " 3fl. d~fenn.oe H-.e


angle -G- o t whioh the bar - ,u be 1
nolrned io the hon~onfol when

361.) Ref elY'l-ng

tS

in

.poGt;on of' equil;br~um .


/

(n.(eo&&-tsin9-) , HfooGO{ts.o..))
1 t ton60

i 10'?&>

llsnlj Aooly1ic 6t>omeiry .

._y=y

..!J:!J1 - m ( lf.-7'-1)

11 fan60 12 (CDG9

for RA.

= fon(6($)(x-o)

~ -0
~

fo(-

"ll =

Jtfan6()_. eqRA-

Re.

s10Q.) - x

12(cos&-~s1n"6-)
1 t fonQ>

~ - 1!ZSfl& 7 -~{'x-12a:1S&)
_y = 12(cos-e-+sin&)-~ - eq,Re.

- ' ; 1!Z fQl'l 60 ( COSfH Sin&)

ft f on60
.Mo =:o 0

~11ga"66-Gn&) - .3~1 ~ ~r9(X)6&- - tt(CO.Sei&,no-)J

L- 1 +-fan60

ffi0066-a..&- -

600SD-

1 t ton60
-

noos&- - =->(COG&Glll6)

1 +on60

1 t fon60

33COS~

60(~6in&)
1+fon60

33aJ6&6ttan60) " 60COS& t oos~


~33(1-tfon60) -60] &9,.s1n~

~{1 ttai60) - 6Q] = tao&(60)


f~ "' ().5026279

-e-

= "16. 6Q6391

C hopter 4
AnolyGtG

or

Structures

40f.) Joint 8 of /ha iruss shown in fig . P- -t-02 ,s euQjeded to

the fore~ eJterted by the hree melTlberG AB. BC, 'ii., BO. Mem
f,B'l+c,80 ()t'Q. in the .s;omc;i crlroight line.buf 8C iG inclined of anongle of-8 dEigrece wifh this s!ro igh~ line . how ihat the force in
DC must be xcsu . 6enerolo~e lhic 1'6sul "'i then ehow thol the for:
ces in ~ember CO, OE, EP, Fr ,til,HK, "'-,J I<.. ,s a/ro 26<"0
.An6. At o Joonl euqjected to lh e ocfiotl

"!.

ti

,..Q.. C
>

of three rnembeni but no olher load


-.r two the members ore c;o11oneor,
the Force in the third mern~ mucot be

fl

or

zro .

..03~ Deiennorie fhe forces in eoch bcir

or

ihe truss shown in fig


P-403 . Hlnf: fi'r'Gl deiermine wh'1ch bctrs ccirry no loacl us"1n9
pr1n oipie de"sbpeol if'\ Prob. .<\02.
P

ce

CP - o

'.FvO
P C061n60t OEs1n60
G1hce : CD=DE

p 2co s 1n60
CO .. o.577P "'OE

co= eo o.S77P
.:!i':Fn O

BF "' 60():)5 60
~F o.!!!>77Pcoe6'0

f3f

0577P(O.'!>)

BF ,. Q.289P

- T

#le forces

40+) Ostermine
shown

in

or the NX>t t ruSG

the rnembers

o.nsider-;ng the tJJhof& flg,,..re,


~Fv. :o : RAv + Rov = -too+ 1oosin30
RAv t Rov 1SO- -0
"rt:.:._::=----....-+~c-~-:_:..'""'~llC"-;-R__oh
__ ~Fh O: Roll ~ 1o0oc::os ao

RoH - 6 6.6 lb.

1oot>

@pt.A.
@pL 8.

~Fv "'O

RAv ~ Aesin 30
RAv o.sAe-@
1001b

~fh-0

100lb

AC " A8

1fv O : BDsln~ +Af'>Stn0 100 +1oolOlnao

o.s BO

if Ae:: 100 lb

MA0 " 100 tao

BD-+Ae=100(2) .. ~oo-
fh O: BD<Xls:eo" "A8casac' t 1ooex>s.:ao'

BO,. A0 ~~-@
~.

cosao

f:-C = o.866 AB-@

RAv"o.sAe
R-o.v o.s6oo):: so lb.

ff,.._ AC .. o.B66(>.e)
a.B6b(100) ~ $<;.611:1-T

.sin-+

@:Cr\

A8t 100 tAe - 300

AiS .:1001b-b
60/.8t1001" !ZOO II:) - C

:i;Of.11 .. 0
AC CO .. 86.61b ..-T

.=!:fvao

6C " 100lb ... -T


406-) The canHlever truss in fig . P-4-06 is h1'r"lgQCI ot D ''E. Find

the fores in eoch 1'1"\<9mbGr .


@

pt."

.:!!f'v O

A8s1nac = 100 : Afl rooolb- .. T


a!f=h 0 : AC

ABcos~

AC " 20:0 COS 90'

/\C:. 17.3.2 lb - - C _/

~fv cO : 0Cs1n6d+at>sinao "'1000tA~sin30

@pt. a

o.eBP-to.B6lSBC" 1oootzooo(o.e)
o.seo +o.9668C eooo - -CD
~Fh = O : BOcosao e 0Ccos6'0 -1 ' Aeco~.ao

Ae

0.9,;i;eo-o.~ec + ~000(01 066")

10001b

ir eo :zsoo1b In

8y Elimination : 1 ~ e
( o .s.eo + o -e66ec = 2oooro.s
+ ( o.s&!Sao -o.esc 178Q) o .966"

ae -2000-0.e(~~oo)
0.90<;

ac"g66lb.-

--- -@

c /

0.12500 +

O.w

ao -

. @ pi.C

o.

a ec -

o.+~3

1000

'
" 1.soo

BD 2000 lb

,L- --T

~i=..... o
CE

CDsm 6t:J 8Cs1r11;b t 1000

AC

CO Q6G(o.g66) + 1000 -

1@1b

ao.aofb ____ T

o.s6'6
~fhO

COcos<SO.,. l'\C

ecoocro - ci=.

CC= 2020(0.r>) t 1732-t 866': (0."t>) = ~17.!i lb - . - C

407.)

force

In the coni!lever- truss shown in Fig. P-407 ~ cornpute


in rnemben; .Ae, ae, ~ OE.
@

pt . A

-'"'"c:.c---=-"""

~"'0

ABGtnaO c1ooos1n60"
t\B

10'Xllb

==

11se lb - .. - T

@ pf . B .
A

Us1~

R.ototion of' A,os,

Lf'v-o
se

@pi. 0

~ +

.fv O
OE Gin 60" 100()

PE 11s+lb - - C

1000lb

the force in eoch member of the Worren tru~

-409,) Cot'l"lpute

shown in

Fig. P --100.
~lb.

30001b.

conoiderin9 the whole' ngure :


--E~a O

R....v(20) WC0{19) I "'<)00(10) +


3000(:>)
RAV

~-o

=A6s1n60

/\Be

.+~.50

.- '

'

rv " 0 :

l3C " <1910Slr'\60 - 2000


s1n60

BC,. ~6QO Jb - ..- 'T V/


~o:

~fhsO

eo

ABco,60 "' 1-.C


c

l\6sin60' .. 2000 ~ ecsincso'

o:ii66

t\B +9o7.62. ... 4910 lb-c ./

/\C

'\910

J-c

lb.

2000\b

-~
RAV

~.50

pt. 8

(@

@pt. /\

(cos6o)

~+55 lb-.. -T

so c ABccs60' t

8C oor;60

4910(0.a) t a500(cH1)

BO,;, 3755 lb - - C /

/
pt . c
~f... "0 :

ec 61n60. t co $tf"\(!O.

-1000

2eoo(o.966') t CO(Oeo6)"' 4000


CO 201g.9+ ~ Q020lb--T
.Fl1 " 0:

BC COS6<) t AC ,. co~ t CE
2600(0.!5) t 2+ss .. :wao (o.s) t CE

Cl: .. 1300 t 2-4.5~ - 1010


Cf. "' !27+~ lb. - .- T

.l(F\, O

@ pl 0.

De&tnBO' .. cosina>' t

~~

.aocxJ

PE !lO!l0(0.966) t 3llOO

OE! - S49+. 2 ~ ~90 lb

CD/ I\oe

fhd force in memberG

411.) Determine

r.a.

/\8, N:,.,

~o.co. '-, ce of the

rontilever t~s ,sho-Hn '11'\.


P- +11 If the lc:xds lf/Sr-6 opphl9CI
of C \._E 1n<>leod or ol l3 \_O, specify whioh member(; would
hove their internol force ohonged.
ton&c

100

~
30

-fT- - 33. d.>.

pt . /\,

r:::Ae
~

AC

pl

~-o

. ' aoo

,. ec ~oo lb

.i:'.Fv o
ABalflaa-6'" 100

"'e 1eo.2e 1b

~FhO

ec

~OJSaa

...,. aocon.~

eo 180.2& lb -

/\13 -

@ptC,

l"lco

~,,.,,.,

.....c

'

tono .icua
10
-& ea.12

Ci!

oe . ~ i::etS.!lf'
w
JO'
~Fv-0

CO.olil~ ,~

- BC

co - ~olb--T
~Fh

C~

COGOG6&.12 tl'C

""

~so coss~.12

er: "' aoo lb-..C

1so
./

..T

I'

~FhO

/\C ..... ~ 006 ....,.q


AC-

1~0

lb

Determine the force in each bor of ihe kuss shown in


Fio i:>-1-0.s cau1>ed by ~ining the 11.o- lb toad oi oonstont velocity
of 8 ft per SOC Wha~ change iA iheoo fo_rces, "if' any results from
placing the roller GUppor a D t.., the hinge support at A?
-tos.)

~Fv=o

Av+ Ov "110+ 3/5 ( 120)


bvt Av =ov
.. 20v "'12.0t 3/s(1w)
2 Ov

192

AC

Dv " 96 lb. '" Av

ot

A:

~~- Ge
/'.c

~- l\C

11

A6
iO

AC 12e 10.(1ent:1dl'\)
AB " 160 lb..(comproi:sion)
ot o:

BO .. 160 lb (com~io"')

co
bv~ .

eo(&Ao) - Oh

Oh - 961b
; CO 32 lb ( iCl"lSIOC'I)

at C:

l11tercl-onqinq hinqe ~roller .

.su.pport will rot change the forces


in each oor e"G"J>t f oc- AC '*. CO

..-eFyaO
~ BC
BC 192. lbGension)

/'w +!)..,

= '9'; t

9,/1o(Ae)

= 224 lb (fen;sGW"I)

CO " BO(Q/-IO)
" 126 lb (tens.on)

4C9.) Determine the

force in rnG.nbers AB,00, BE,~ OE

Howe rcx>f irus& shown

W'\ fig_

or ioo

P- 400.
.iMH,.0

,a..., ( 40) - 60d.il0) - 1ooo(20)- '4-()0(io) =o

/\v "' 10.so rb.


~..1!!::-..._-=~~..;:;;...-=-11--=-~" ~ :t:M" o
H.,(o40)--400(30) - 1000(20)-600(10) =o

Hv = 9.50 lb.

oi B,

fB

~~

~fyO,

BE s 1n6<> - 600(S1t'l 60') O

:. AB "' 2100 lb (~ion)

K. 18ZIO it, ( tom;ion)

BE IOOO

lb(a:mprc~ion')

_D,. ~ 0 ,

BO 9E.~c;o GCO(cos GO') 2.100

SD 15<>0 lb (eom~ion)

,..10-) Determine ihe force in eoch member of the Prutt


ot 8,~fP

'1CMI\ O, "800(M)1800(1'1)
180o(e) = Av (3~)
A .. .. 'l7co lb .

~A.

L
s

BC CO
~

- 3-

CD ,.

21~3.33

lb-..-T " OG

=o

o/"13 co + c~
... avoo - y~ (rn;s. 33)

~ ; EG ~400 lb - - T

...

/\.B +SOO lb

1);.

l'C

Ac

~"' AB - ~
3

Uii119 Hew AxM,


BC 18001bC "' FG

~fy o,

e<;

~F.

r=--"c

DE~

---C

=F H

AC= 6 H 3'6001b-- T

8~

in the orig~l o.,.es


AB-BO ~ DI' ~ +.soo lb--C

/'I!

at C,

roof frvss-

41~.)

Compute fh8 force in eoch rnsmber of the Irv~ ~ in


fig . p.,.,.,a. . If the loads oi B '!..., D ore 1 sh,r~ ecl '<ef'lioally down
wor-d fo odd ~o the loads oi C \ i: , -.vii\ loore be an1 ohonge
in ~he reoo ti0ns? Whioh members, ~r ony , would 1.m def90 o
ohafl99 in inler-nol force .:.
B -lb

W '

o:ine>iooring the whole


~Fv

frgur-e .

pl. B ,

100b

!=V.v t R'"" ~ t 200 t 200+ aoo


~AV f R,.,, ~ 1CSOO lb --<D
.;EMr

50

R.-.v(oo) ..oo("I<>)

+ 800(110)

t .f-0 (~) t~ad)

~ Fh ~o

- 000 lb

41,.1b5f, the volue of' ~"'v in

~00 :: ~ l'\8

fhv .. 1000-eeo - 12010


@ pi. A,
tori o - ..ZQ

80. 98% ' - 49~.5


... "t9~ lb -:tC

10

,.... ~

-<>-..,,,.

~ ao +~~ ..

~ (.+q2) t

fa. (9es) -

BC - 701 .o+ ,,..


- .s95 . 9 t

7Q.)

ec
/

lb - "="'f

"" 985 lb-- -6

:E.Fh o
AC

"fOO t

@ pt. c,

P,,..v "'2!> ,sin63,.o;s

ac

4-00 f

~Fv "'0

l3C .L co

1\0 C0668.4 S .

.. 9 9 15

{ij

COG l>a.+s

IC~ 44<).~0 ~

f (20()

700 .. ..L. c o f

+40/b - . -

T/

'le

~00

- co t11e.oz.
~

/
112aJb - ..-C

:EFh O : /'C t ~CD- ~

ce -

440 t ~(11~) .. 11"'!1.76 ~ 1"1-4<> lb- ..-T

@ pl

~fv - O

E~

@J pf.

~I

~l'V -0

oe ""'<X> lb

- .. -T

Rrv-

>ii Df'

or .re(1~0)
OF -1609.97 :::: 1610lb- -C
the force in each member of

.+'3.) Determine

the

P- -+13.

Ghown

coi;;1ne LOW :

o 2 = b + c - abcCosA
6 1 ,. b 11 9 11 - 2b(9)~A-
be a OG +Ct. - aoccoee
be e

9- .11(~)(9)CC$1 l10

b - 1:a .oa '

n l ,

@ pf. /\,

6" (1.J)St+9 - a(13.o&'K.9)cos/\

I\ !l3.3P

c:ont. <Yl e, :Efv O

so,11nao - SCs1neo"

'

o.& so .. o.e66sc subO'f . il.

1~000
/'le. 13099.CS' ~ 13100 lb -

'-;I

Fh O
5

1\.8 .. 11<'000 lb_,,

-c~

~k
~Fh o

/\a BDcosao' + BC00550"


1~

.. o.e66ao t o.G ec -

o.e6fJ(~"*; ec)

o.sec

1 ,

ao -

AB

AB- 1a1ooo:os~a9 120a.3.-t 91b

BC 6 000. !16 ~ 6000 lb - ~


1n

l\.CCOS.l1a.a9

Ir'! 1 ,

o -966 (6000)

0 5

10392 l b /
...

10~1b -..-c

M.) Oe~ermine

the f0t"C8 in member AB, 00, g._ CD of the

truGS Ghown '1n f i9. P-""14.


0

~ Mti =0 . Av(36)

slooe ot Ae ~ 1
slOP8 ot BO ~ 1/3

3oof...21) t 300(16)

QOO( O)

Av 600 lb

s;;lo)'O ot CO c 4 / 3

ot A,
t

AC

~ Fy O

1/.(1 1\S =6 00

A0 048.-'.3 lb - ..re
..tFi< o
AC c 1/ ..(1 AB
/\C: 000 lb-.!... T .(

ot

ot

C,

e,
~Fy O

~F,. ~o

300 -1 ">/ s

3/,f1o BO 1~ AS

CD

BO 632.~ lb - - C J
..:f?Fy O

V..t1o

~ 1'16 ~
BO t BC
BC 400 lb - - T

co .. ec
e

125

lb - - C

_)

+ts.) S olve for the force in members FH; DF. ~ DG of' the
trvss shown in fig . P- 415 .
D

considering fhe whole,


~Fv ~o

P..11.v

RHv " 300 t ~ t 900

RAv + R 1w c 1500

aoolb

R>\v

900fb

M.-.o
"aoo(9) .. aoo6e) t 9oo(zi
Rttv .. 900 1b:

RHv(d6)

;iJ
~f,,.~

R.-.v

=1500- 900

000 lb .

=fH G1n.+s

fwv

fH

900/s1n4e

FH" 1a7i:?.79""

1~7olb - .. -

@ P.f f,

@ F,

~+'<~

~F~o

~ FH w 3/$o DF
OF"!' .JiO (1-270)
(3)~

OF 946'.'6 ""94 7 lb-- C

@~~ ~
~\=.,,O

+foD6 t F6 "000
..;,, 06 t 600

900

DG '" 300(.s)
i

06

;;i7_g

lb - - T

q.,,,co
~

fH ~ FG t

Y..a (1210)

V.J!o Df

t=G .t
f6 c: 599. B<S

J1w (9+1)
~

6001b

U1>in9 the method of 'eGlions, de.termina the force in rnemt>erG BO, CO, ~.Ce of fhe roof lrvss showf'\ in F19 . P- m .

1 7:)

BO

it-....
~1o1c o

JV.v (1~) " 0 0(9)

so .. 11l0(11l)
9

eo - 160 11:1 - - C
~F=v

o
3-1; CD

RAV "

GO" .13/'9 (1'10)


co .. '100 lb-,. c

"MoO
l=V.v (2 4) ': ce (.g)
CE

CE

Hl0(!24)

9
:3!lOlb- -T

truss looded 01> ~ in f9 P- -t1a is SuflPO"-led


by a roller oi C ~ a t\1nge ai 6 . 0y ihe m&thod of Geolions,compvie H-e force in lhe members BCr DF, "1_s CF .
considering th8 -...hole fif)ure,

418.) The Worren

.:ffh:O : !:{GH

6()()

lb

..:fMc"'O
6()(J ('10) t 000(1 o) 11ooo(ao) 400(~~

RGv(..o)
100Clb

RGv 900 lb .

. 100?lb
~Mi:O

OF(ao) - R6(flo)

or c eoo1b --C

6C - ++7.:21 "'4+9 lb - -C

~Mo o

CE(:!O) t 1000(10)-t RGti(!ZO). Rc;v(ao)

a_s eoo('ll0) - !000(39) - 600(2 0)


QO

Cl! 1001b - -T

) lJGe fhe method of &9CfionG to determine /he rorces in memba


BO,CO, "', CE of lhe Worren TruQi; .
aooolb
----1.P

toCOlb

con1:iderfr>g the whole -f)"gure,


.ot:M;. o

Rv (ao) ~ -1000(10) 't aoao(\1t)

oooo(e)
Rve - +7!50 lb .

s1n60' () ~s

e-e.66 o.
' \.,~
30Me. O

Bo(g.&6) -<1000()+ oso(to)


BP - ..a2soo/e.66

R'<'s(ts)- co(10)- aooo(10) +-t<l00(11)

co

B097s~ .9lb - - C
~Moco

1:M~

( e.'6) CE = 4f-7S0(5)

ce :is1.so/e.uc

ce ; Zl-n.s lb- -T

~12s9"'10

CD 2125 lb

- - T

.. o

- co(e-60)

t 3752.<J(e .6'0}"'

aooo(s)

CO~ 1SOOO-(:a.l!SOO.f1t)
- Q.66

co '

~o~o.e

lb

i::

~020

Jb --1"

Oet~ fhs fON:B in the members Of, 00. lio, J:6 of thc!s
Kowe in>" Q\own in fig . P-~.

4f0.)

.!IE~ O ! Of(9) t100(1R)


Of c ~lb - -C

,Jtf..,-0:

,~ t ~

06 flOO

06-16001t> - - -- c

'"'o:

DP

+-4fs oo e6

moot

ta(~) - e6
f:6 4<>001b -- T

in rig. P - -tK, determine the f()r'C.O j) Br


then check ttiic: ~ult us1ng meihod of
GeOfions. Hinl : .To opply ihe. roelhod of secfionc:, {ird obtain ihe

f(lf"' the

4lf.4,)

by ihe

tNGS '"'1oMl

rnettiod of~ts..,

...olue of BE by inc;pection.

consl del-tng ttle ~

figure.,

al!f" O
Rott1~1b
~F.., 0

RA.v I Rov

~112'00

RAv I Rov 3c50C -

G)

~oO

R.-.v (19)

~(9)

rv._v
Mot hod

of Join ts

+ 1200 (12)

10011>

in 1, Rov .. 3600 - 2000 s 1600 lb .


BE1~1b .

@ pf.

"'+
I\

NII
N:;.

~v

!El'v' O

'4/11 "'8 e

RAv

AB-{s(~+
"'~ - 2600 lb

BC = !24<l0 lb
fttO

Rott

/:1 BO

so- ~(1200)
BO '2000 t>

@pt 8,

~Fito

~Bf'+ ifft eo 1200 t jy-sJIB


Sf s

%[1'J.OO ~ ~(~)-.!1,5(~]

f3F =2500 lb - -.:..C

~Ftt"'v

3(-"ao =- ee ; eo.- iv.a(1aoo)

k\6tlod of Seolioo ,

:~1

so;

~1~

12cxiolb-

R.ow+t:f ~BO-t!24oott~
f:I= =~(!ZaX>) t 2""IOO I 12()() .:. fZOOO
EF - .3!2001b

~-o
~ft, 8F t 1C00 EF

SF ...--t-(aeoo- 1.jl<JO]
BF f1800 lb- - C

tr-u" $hown

the web rnerobers .


~Et= ore perpendicular ro the inclined members ot their midpoin~.
IJIOO the mefiod ci' GeOiiOos. io ctetenni~ tha force in membeN: Of,
-tea.) In th& Fink.

in Fig . P-+S,

~~,.._CE .

~Fv-0

flfAV ~e< -1t2t~Hl.t1 - Qt<.

F''" fGV -tt<.

_& .. ~
10

)(

)( .. ,5.169,

::E.lt\a"O

Df'U;.st) ta (!1.s) +1(12.s) .. t'6 ~ (1" ")

Clf' .S-9139 "'..s.02 lr.ips - . .JiijO - y '


Iii() ~

Y'" 12.S'

bvO: _..A;Det + ~"1:iDF HZ i1


DE,.~ lUf"' - -T

Fiio
CE t 3/s oe c o/.Ja-OF
cc. = o/..ss(s.02)- 3/5(2)
CE -1- Klp1; - . - T

Show !hot ihe method or joinls connot determine the fOr-ceia il"I oll bors or lne Fon fink truss ~ Fig . P-~6- Then use
-4!16.)

i he rnelhod of sections to compute the forc.e in bors FH, Gli,\.., EK.


~lb

f0t1 ao

cos30' " 3o/c

0.

o/;;o

o ~ 11. a2 '

3 -4,&f- I

iti 1.s' i X2 ~1s'; x'9-::r:z,s '

,.a'.iO f t!

ccnsidering the wt-de rlgure,


P>.v Rov

2RAv = 1600

l:K

i<Av ... eoolb "-Rov


J.n F.g.2 ,
:;io .. Aetb i b 10

R.o\v

~ME40

RAv(:zo)t 2()0(112.s-2il) = l'ti6o) t 1oo(w)t2tX>(:t0-M)

. ..... E = 30-10; 20'

Fg~e
"'"'

I!

In Fig._a
sinao - O E

,._E

Di:. = ro(suno)
OE~ 10

~ (20 - 15)

10
F~ ~ 110Q. lb - -

wi
"'

FH = BOO(W) t ~oo(:z.5)-1oo(!U1)-:<>oo(1:z.s) f-200(5)

~li~M/\"0

"~
,_ ,

tni'\{:

::rd0(7i!l)t 200 (1G)i eoo(u,5) Gli(Ai:")

6H :zco(7,~) t'JD0(1s)t200(22.!j)
17.aa

G '1

.s19.oa

'"' s120

lb - - I

..tM11 - u

RAv(-ao) .. eK(11.a~) t 1oo(ao) t iJOO(atJ-1,.S) t .200(ac:>-1.!S) t 200(ac-22.e)


El< 690.94 ~ 699 lb - -T

43t.) Dofermine fho fbr-oe in the memberco Or, 06. ~ EG for ff-ie

Pori<er

trus sllov'tn in Fig P- +1 .

e pot>818 ot u ' 200'


~?V-o, (;Ol'IQider t h4 whole f "9ull9,
Since : R R;O
1.!R1 - 7(;,;io)
R1 ~

Ra.v

105 K

"~
Ll ~
,.
ts --EMco o

R-.v(7.!~)ao(-'D)tao(~)t !IS Dl'(ea)


-t

'f-6&+

OP'(")

OF 161, 8 7 ~ 162 Kips - - C

fv O

IV.v

30t&Ot

06- ~g

~ OF-t ~ 0G

[1os-ao-ao- ~(1625]

1)6 .. 3/l.61 "" .32.7..,;ps - - T

1aa.) Use the method of sectionc;- to oompufe the force in m91Tlbers


M , AD, BC, ..... 80
fhe. tru1;s -shown in Fig . P-+.:12 .

or

~M,. o

R.<.v(a2) ~(:M) t H00(16)

fe

. . . -o

~r

~ t %BP~AB

p0ca-%[%;?~ - ~J

ao 5001.<>+ s;:. 50001b--T


~

.FttO

~-o

ec - fr7

R.l.v (16) t ~,-..e(ta) ~ >,e(t6)


~ .89()9. 31 C.!591()11:>- -C
~Me O

R.-.v( e) t

AJ AD(Q)

1\1!> 41~

eo

sc-~(s91o)t +,4 (!000)


BC-6~lb- -C

~ " 0(19)

/\0 3000 It:> - - T

Computes the for-ces: in bo~ AB, -4,C , OF, llr.,.OE of the scissors'
irv" how"' in Fig. P-..a .
-fU.)

~1 - 0

RAv (60) 1:1(S>)1n(-.o) t 1~ (30} t 1!Z(2.0)


t ~(10)

~J.

" ..

'

,- --

-~>e.i
,'
.1?Mco
P,..v(!.>O) ~ Aa(16) ., o/..fa9 A0(:20)
t'a -

[ so(:20)]

(s(GO)-e(.14)]
AB - 70.75 "" 70. 9. i<.:.pi;

i. ..-

%MeO
RAv ('!&)_
01'

AC"'

+4i:;

/\CCI~

#'}4 ( ao)

/\C 48 -02

~ ..8 Kip!> - - T

Me; O
30(30) "1~(20)-t 1~(1o)t~ Df(32-~+)
t

~ 0~(10)

OF- +a.+5

""4-!2.s

Kips-.. -c

+21.) Use the method

of sections

to CO('l)pvte the force in ihe

members Of. EF. ~ E<5 of tho con ti lever 'truss described in


Prob . -441 '*. fiq. P--t11 on poge g2.
DE

20

"

2.

oe "40/.a

IO
Me 0 (R<Ssolvin<J OF ot
3/,Jra OF

(-'40/3)
OF'

o)

100(~0) Hoo(10)
lb --- T

~ 360. 56

Resolvil"\g EF ot E)
_2_ EF (w) " QOO (10) t QOO(W)

.C.MA 0

EF " 335 .41 lb. - -T


~M,.

EG (20) 100(30)

'lOO('U>) t 100(10)

E6 ""'\-SO lb -"- C
-+a2~ Refer to fho Gleoor-'bod in Prob. 41~ on poge 92 . ~ compvi e ~he. force in rnernbc:ro BO, CO, ~CE by the mo-lhod of

seoti'on& .

lb

.M.-.~O

f,.. (so) -400(10)

2COI..~ t 000(30)

200(;30)

Fv =
1an~;

7~()

lb.

2o/.,o o.&

-tr ... 26.57.

tonu.s1 = o/21>
0" 10 ,
~M oO

c e (10),. 1W(1C)
CE "- 1~ lb - --T
.!!Mc. o (~..tog BO ol f )
?20(....a) ::i.oo(w) + 000(20) y.{D eo (~)

Fv 7SO_,

BOc 491.93 lb - - C
MF' ~o (~solV.nq CD oi C)

wo(w)

800(20) - 1/-ie co(..o)

co .. 1118.03

lb .,.. ..

-c

423.) Use the rnethod of sections to cleief'rf\1ne the force octinq


in members OF, EF, i.... E6 of ~he ~o-we \rt1~s deSGribed in Fig .
P -409 on poqe 91 .
0

~M.-.o

H.. (-40)~ 400(1!0)t 1000(:.io)t ~('10)

1-1., = 900 lb

,. OP
~
E-

.1Mp

I'

H
4ilO

Me."0 (Resolvinq Ofol H)


1t.. (20) "t00(10) t Of(611'l.30.)(w)
OF c 1.000 lb _ .. - C

i.Mtt O (ReGOlving j:f' o I E )


400( 10) EF 6n30 ( 20)

l:'F ,. +00 \b - .. - C
fl.,. (10) eG (10 \on ao)
E6 c 164-S.4'5 lt>- -T
4 2-f-.) Foritie l rvss shown ,n F'ig . P - .+24, do\erm'ine thefoN::e

1f'I

Bf by the method of joiritso '*, then check th'1; rc:;:&uli using


t he rrethod of .seciions . Hini : 1o oppfy the rneihod of eections

first obtain ihe volue of 0E by ins~tion.


/\to

-....nale,

~Mo"-0

,A.... (18) 1~(12)

2400(9)

A.,, .. 2000 lb --

ot A,

~o

Av c4/!J AB

AB=
8=-,.

9'
lb

t2001t:>

81'

at F,.~l!f
.

~F11

1.!500 (5/9)

otC,

~F,. ~

. 1to0
CF - '3/15 BF .

-o,

10 ---C

+-

AC 3hA5

,....,
CF

2~

~o

'

"

~F

N;

BF .. !2.!SOOlb--C
By Section,

.MAv
2-400(0) t 1200(1t) "' +/~ BF(18)

BF "' 2.!500 lb - -- C

CF

1~

lb--T

427.) Oe~ermine lhe

fC>C"CC in
trv6G i;;hown in F io. P- -.t1.

barG BO,CO, "-Of

' , ua>lb
,.-t- 1t'
B .

or Hle nacelle

'b

~M,,-:O,

c;.,(i&) t uoo(r.) " ~ 24) t 1200(18)

Cv "

i' tb .

e'

_L

~fy O,

1/,w,

o~ t 1600

12llc:n fJ(}()

OE "'632.""6 lb - -T

ot Jdtnt 0,
. M& o (Res;olvin9 coot C)
1/,m, DE (18) ~ a,'1;(12)Cco)
CO" 5<l01b-'- T

1), o,

...,~co $
BO

BO ..

Y..c;o OE
lb--T

~.5.5

rncrlhoc:J of GCCtiorlS to defeirrnins the fon:;e in m"m bef"G Of. FG, ._,GI of the triangular Howe trvss shown in Fiq .
P--v.l8 . Hint : Fi~ deiermine by inGpeciion the forces in the wob
rnem~ of ihe right ide of the tru~ .
426.) l)Q!t the

by inspection web rnemben; JK, IJ, HI,

"'- H6 ron-ies
~

ton_,

'1/2

no load

26. .57

:fM1. = 0, /\v(60) "' 2 COG 2'57(90) t 2


(cos 26.s1)(40) - ~(s1M4i.s1X!I) - 2
26.57) - -

(1o)(sin

Av 2.46 ic..ips

.i!;MFO ,

~Hr:~~"J.---61

Av('30)

= 2(ooi;u.111X'.'10+20)i(S1n2i.;.s7)

(~)(.5+10)

:Mo o :

2.46(30) -2(10in26.e;')(~)

- 2(Stn2.6.571(10h ~(f>ln 2Ci-57)(30)t

2(cos~.97)(20)-t 2 (<Xl6u.sr )(10)


0

OF 2.!5~ip& --C

GI

~M,., o;

+ Gl (15)

o ....~ Kipii - -

f6(30) 2(cos26.1:17) (10120)

+(1)("1rl%.51")(6 t10J

FG .. 2.IZ4 Kip.- --T

4~.) Fo,.... the contilever truss .shown in ft"g . P-429, deter-mine9


the forceG in member' OF. FH. Fl, GI.~ FG.
@.

Lef\ of A-A
zMG=O.
s/-129 Of(24) =200(oo)t 200(~)

100(w)
Of ~ 1256.s+ lb - - T

MF o

( ot

Lefl of
Mt o,

~lb

:ZOOlb

e-a

.s/.1+1 fH(M>):-400(~o)t ~40) t200(60) ~

A- A)

MGI:. 400(20)-t ioo(-4<>) t2oo(6o)


61=1166.67 lb--C
~F11 0 : GI ~~FI

2G0(80)
Fi-1=166+. 81 \b- - r

c'*F'H

Fl 206. 2.7 lb - .. -c

L.cn of' c-c


Z:M...,O: 60 FG = -1-00(60) t-400(41-0)

2.00(20)

f6 733.33 lb --T

The lood.s on the Parker truss shown in fig. P -'t.30 ore in


tc.ips. One "'-iP equol.s 1000 lb . Oeterm.11'\e ihe forces in members eo,
ee. CE. 'iii.. OE

+30.)

Fv =O

Av tJv 30(7) Biii Av Jv


. Av ,. 105 l'.i~

JE.IY1 ol !he: intcrscx;i ion

27/.f13st 131:(160)
lw

Lof'i

LMs "'O (Left ola-o)


a.1 CE =109(29)
Ce

or b-b

~ ot \he

= 97. 22 Kip'-..- j

63.

88

60

8l, CE o

110(10~)

~ps

1nief'Ceciior'1

- -T

or BO~CEO

160(30) t1 60 OE t 30(135)=1o!S(110)

OE "' 16.07.S K1PG - ..

~"'E. " 0

~os (oo) = ~o(2!J) .. s/~ B0(3z)

0D " 1+:9.41

t 30(135)

BE~

...

of

K.ops

- -C

-c

For tho fromo

!oodod c.s shown in Fig. P-440,,de>t<:>r ~ -JOrliool cornpononts of tfio p'1n pi-os. suro ot B. Spoofy d irnotions (up or down; left or r"ighf} of
tho
OG '1t od.s; upon m~mber' CD . .

440.)

mire

H1C horizontol

foreo

3001b

.,M,.. =O
Ov(-4-) - wo(a,) + -aoc(G)
Ov " ssolb
LMo " 0

A" ( 4-) " :300(6) - 2.00(:Z.).


Av= .:isolb
'l:Me"'O

c
2
BH

3001b
8

At-t(4)" 3So(4)
.M

t- WO(~)

BH"' -t5()1b

Gv

AH" 4!l0lb

~Fy"O

+'
D

:!Mo "0
B11 (4) = 'E09(6)

"

~FlC"O

sso -Bv =o

Doi
~ .55('.)lb

(ten) ... AH" BH .. 4-eo lb

0v "'.S.SOlb (down}

Fy =O
e'(-~ - .3so A o

Bv ssolb
+'l'I)

The

structure shown in Fi9 . P-441 is hinged ot A

~c. find

tho horiz.oritol ~ vorticol ccmpbncnis ol tho hingo force ot B,


C,!,... A.

s'
1001b

B11 : 17s lb
.;EFx

~F'y -o ;

O; BH AH -

l'w -

100-100 00 0

A" "'

:zoo lb

CH=- BH "17.Slb

17~ lb

The frdme shOym in fig. P-+,4-.3. iio hinged to. r19id s:up. JX'rls ot /\ ~ E . Find the a:impoi{onto; of tho hingos forcos ot
443.)

/\ ""- E ~ tho fon:;ot; in mom~s . BC ~BO.


1tolb

1W(4) - EH (4) =q

.M.--=0:

E:H
:Z::f11"'0, :

~. ;Wlb

E11>-A1-1"0
c AH
'\ZOlb

E~

ot isolating bar Ae
.::pv'\a"'O: Av(6) -120(4:1, "0
. Av= 6o lb.
~ng

4'

4'

1ho wholo f'romo:


.2'.:Fv =o: Av+ Ev - 120 "'0
i::v =120-co

BC .

::l'..FH O: BD(.3/50) - Ali =0


eo "'(5h)A~

Ev.,, 60 lb.

. 80 - 200 lb - .. .;... c .
XMA O; 1W(4h OC(B) - BD(4/.:s)(a) ~o

J3C"' . 100\b
--T
: i
44+) fhc frornc -?~wn in fig. P- tt4' is svpportcd by o hingod ot /\ &, o rofler ot E . Compute tha,, horizontal ~ vorti-

. ~I . co!T)ponerits o.f1the . hingo forcoG ot B ~ C o~ 'they ad

upon member ft,,~

'

.('

.::t:MA"'O: 1'20(11) - Ev(1o)=O


E'.v" 264:1b
~Fv ~o : Ev -Av - 240 =6

'

Av "' - 24-0 -t. i=v.


Av ' .24-lb.
o~

BO

%Me-0
240(9) - 0..-(6)-0

lrolating bar BD
Bil

--tji;,,

61

" =

l"3' / 240lb

0 Otl

~~o"'O: 240(3).- Bv(6) ""0


Elv = 1QO lb { l)pvvord w

~peel to bar /'C,}

3001b.

lsololing bor CE

::i!Fv ,.0

Cv t Ev - 0,.. =' 0
Cv ":360 - '2.64

Cv :

96 lb { dOWr'l'NOrO w oih ro<>ped to

1v1oO: C.i(6) t C,..(.3) -E.,(t) eo

AC}

C11 = 264(2)- 96(3)


G

Ci - -1-0 lb [

Isolating bor /\C

"

1en

io tne

w i th t"C~pect

to

AC

.;;!M,..O: &i("-) 1 Cv(s) - C..(10)-Sv(~)o

s .. = "t0(10)+120('.!) - (96)(5)
....

Cv
II

BH

40 lb to tho r ight

.,,

445.) Tho frame shown 1n F19 . P - "4-45 iso .supportod by o hin ged ot E ~ o ro llor o~ 0 . Compvto the hori20n~ol ""' vcrti col components of the hinge for-co cit C os H ocis ,upon eo .

.Z.M E =O

Dv(0) "' 240( 3 )

Dv"' 9olb
~Mo

.. O

E... Ce) = z -40(s)

Ev

lc;olo~ing bar

Ov

lliOlotlng bor

,..... "

4Av

...,,

Ae

2,-0
2'

2'.M.-. O

Bv(6) c: 240(4)

Bv '"160\b
~Me O

(G)Av 240(2.)

A.v= eo lb

rev

0
~ Fy"O

Cv =1ro- go

c. . =101b
ZMeO

c.. (3) " 7o(3) t 90(7)


C H ?.001b.

1so1b

OB

~.) A three; - hinged or-ch iG composed o( +


wo fr.usscs
hifl<1e.cl logetl'x::.r o l 0 in Fig. P-4-46 . Compute ihc compoi:onts
pf tho reoction al I\ ~ thon fi"ncl the fon:;cG oc\ing in bo~
1\8 ~ /\C . liinf: first lliolole coch iru-.c; ow o fr-c o body .
.%1AeO

Av ( eo) -360C'oo)-600( 20)

"0

/\'I "" +20 lb

XMoO

A ... ( >) -36J(W) A,..(30) co

Att(30).,. 360(~)- 4~0(4<>)

: 3>'
I

'

AH

: 120 :

= - :3:20 lb

: . A ti - 320 w tho right

,>e -: -240 lb .. 240 lb - -T


447.)

Two trussoi:; ore jo,"n(}d os; shown i n Fig . P - +47 to form

o -throe -h'ir1qod orch . C.Omput.e tho horo=nlol ~ vod icol corn p<Y'Onls ol the h ingo forco o \ 6 'r>l<.. \hon deter mine i ho type
~ rnogn1tudo of force ,n bors 130 &., 8E.
2'1()lb
10'

tI 10'

nolb
10'

~Mc .. 0 (!;
n o(1D) t 240(30) t Att (10) - Av(40) .. 0
fw(4) - Ali = 1+4<> _. <D

10'

ao'

lsolot ing left tr:; GS


~

--"''--+_10,,,.....:
' ~

~Ft!

.2'.Me 0

Av(2.0)-A.1(20)-vt-0(10) ..o

Av -

AH

120 -

6t1 s .3201b
Z:Fv .. O

eq. &... @?.,

Av - Bv - 2+o 0

Av (+)- AH"' 14'+0


- (Av - Al<I ""1W)

Av

" 1320

3Av

02

4"401b

c:

=O

AH - BH-0

av=~. 1~
a.,,ii t BH 2
= (ro'.J)2 + (3W)i

"'

AH "Av -120

="""40 - 120

Bv "' -+40 - ~

B .,,. 377,4 .-lb .

At1 :srolb

A boom carrying the loads shown 11"1 Fi9 . P--+40 ii;


eomposod of throe 69rrionts. It is suppodod br four vertical rooctions S.,... joinod b)' iwo f rici.Onlc&& hinges. Ootcrmlne the values of ihc reoctionG" .
448.)

400lb
200 lb/ft

1001~ 0

@ member CO
c.. c ~oo ro/n
W;

W, =1oo(G)

- 6001b

ec

@me.mbor

j
200

WO l~A

P.2 w2

,.,. , I

~o '

13'1

W2 ~ 200(14)

~eoolb

30:>

~MR3"'0

. Rt (10) .. 200(14') t

2ero(1) - 3Q:?{4)

Rt = 2120 lb.
~ lt2

~Mco

R+(o) ~ . 6oo(s)

...,,

7 '

R+

co

Ra (10)"' 11 ~(3) + 300(1.q.) - 1.00(4)

fh.

1100 lb .

R.q.

= 300 lb -=Cv

449.) The bridge shown in Fig. P-449 mns'1sts of tY110 end


s=ctions, each weighing 200 tons with ~ntcr of grovity ot
6 , hingod to o uniform center. span weighing 120 tons. COm
putc ~he rcoctioni; ot A, B, E, .&.... F .

~Mv

@membor CO
C~

Dv(60) =120(30) t 60(40)

-40'
tJtiT , 0
C,_-=--"--"o;.,

Dv : 100 tons
1~T

Cv

:i:Mo ::O

Cv(6o) = 120(30) + 60('20)

Cv"
Ott

0 0 tonG

.>!MF "0

E,,(so)

!200(.30) t 100(70)

Ev "' 260 toni;

fv "' 4<> ton6


-'!Memo

A" (so)= 20o(w) - eo(w)


Av= 48 tons
.::EMA = O

Bv(so) - 200(:30) t 00(10)


Bv = 232 fons
4.SO.) A bi IIboo.rel BC w eighing 1000 lb iG subjected io o
w '1 nd prcGGure ot 300 lb per n CIS shown '1n Fi9. P - 450 . Nogl~ting tho wci1g'hts
tho Guppoding mcmbc~ , dctormino
the c.ornpononts o f the hinge fori:;cs ot /\ ~ F .

or

Wind

pressu--e == 300
~ .300 lb/fi

~t>/fl.

x 10 fl

- 3000\b.

.2:MJ\ .. O
3000(9)+1000(~) t FH(<t)

Fv (12.)

F..,(3)-F.i

=82..SO -0

:tMp = O

+'

aooo(s)-1co0(.-) .. Au ( 4)

+'

@member
c..
G

6'-

Av(1~)

A..,(3) - /\ti USO -@

ce.
~MeO

C..

C11(10) .. aooo(s)

CH 1eoot>
~F,.. O: 3000 CH -Bti 0
BH 3(X)()-1BOO :: 1500 lb

~member

CO,

QI

~
Dl4
@ member OF.
'4

%Mi;; O: CM ( 4-)

= Ft.1(4)

FH - OH 1ti00 lb
%Fx o: E01 -

isoo -1soo o
EH .. 3000\b

%F.O: Fl-4 rY.XXJ-AtiO

....

AH= ""'500 \b

fv(3)

=8250 t 1500
Fv .. 325 lb

@'2 ,
Av(3) 22.so t 4500

l'w

2250 lb

The frome Ghown in Fig . P- -t.51 u; h inged o\ E ti.... rol lor


supportoci o~ A . Dcto~V'lc hor;zon\ o l ~vertical componel"lt
of tho hingo fw s o t B, C, !..,, o . l'legk~c\ the weigh~& of the
451.)

rnernbon; .

2.'

3001b

It '

:E.F,. o

~M/\- O

Ett - 24<>

Ev (1Z.) "'300(16) - 2"'!-0(10)

f:" ... 2.SO lb


::i:Me

c:o

-o

E H - 2."'\0 lb .

':

Av(1Z) 240(10) - 300(6)


Av f

@ momber CE

c
0"

Cv

,/"
0

"+'
e

'

so lb
~~=o
DH (4) c ~"!0 (0)

OH = ...aa lb
%Mo = O
Ct (..-) = 240(4')

CH .. 2;...0lb
~t.'leO

.
T

. IA-(6)=so(6)+aoo(~t) t~1t)
~

~'

Dv .. 810 1.0.
Ol OM " 8ti 400 lb

~Fy c O

8v .. .so+ 010 - 30()

0v ,.

SliC)

lb .

Choptor S
Fr i ct ion

-'15.) A block we.1'ghfn9 W lb is placed Ufx:Jn o plane inclt'ned at on ongle tr wi'th the horizontal. Discuss who t
)'Viii hoppen
th6 onglo of friction Is (o) greoter
than e-, (b) equal to -e-, (c) less than ir .
(o) If )J is greater than fr the. plock will not .slide down
instcod it will reforn '11.6 poGif!'Ot:'\ becouse rhe rrict1onol

1r

force iG so ,much tho! it will hold the blocK.


, (IJ) If is cquol to J- the bloo/<... w1'1l et'ill rcl .slide- down
bee. having -& cquo1 to flJ the. system will 1Still be, In equilibrium' he~ the rriotiorool fo~ Is ;n. as mi"n~murn.
(c) If ii> le~G fhon tr, then af1'pp;n9 oco....rs becau-se.
the fr1"otiorool force l.s not onou_gh to hold the- blool<. .
.506) ;\ -tOO -Jb plocl~ is res-ting on o rough horizontal
. GUrfooe for .whioh the coofficient
friotion is 0.40 . Oet.
the force. p required to COU6e motion to impend "tf op~
plt'cx:J the bloc/\. (o:) hori'z.ont.0111:_1, or ~b) downvvord ot oo
with the hor;zontol : ~ (f)whot minimum force l's rt-qlJi~
to stod 'lhe motion.

or

Q.

sin u.e

-tOOlb

sin. 68. 2

P= 1601b.

p - .. 41-00lb
G{n~.s

Pmin

Sin 21.ir

400lb

.sin go

Pmin .., 11-8.0Jb.

sin<l!e.:.i

The ~oo-tb block Ghown In figure is in contoci wifh o


45 incline,. The coefrrcien~ or fr1"ct.1 on i5 0.20 . COrnpute the
value of the horiwnlol R>rce. P nccc<S<>ary to o) ju6f stod
Hie blool<.. up 1he incline (b) Just prcven-t motion down the
inoJi'ne. (o) If p:4ro lb, who! 'rs the omounf 'direction
..507.)

the

fr-1c t;on. forx::e.

o.J~

f .

~00~ .
~
p

e- f\

or

"?

.s-,n"--.s-9.o+
-.

P-

.!lOOlb
sin l!IOQ6'0

ea.,.o lb
6001b

sm.!'g.of
P=300lb .

sin.30 .96'

c)
.f,,

::o :

400 (COSi.5' ) t F - (~oo)( COSH,.) .. 0


F" 70.71 lb

oOO.)

The. 200 lb block shown in fi9 . P-1.505 hos impending

motion tJP the plane cousod bg thet horiz::.or:iicll farce of

"'fOO-/b . Oot . lhe coe{ ficient of .s1otto f;rdion between the


contoot Gurfoce&.
efld "o: /'I= 400(s1nao) t 2oo(cos30')
N "'.37-3.21 /b .-

-<!h cQ: F 4oo{c.cs~*).-.zoo(s1n~o)


c

F ~ 24611 lb .

ft .. J/{i

" 24641

o.66

'373.2.1

The blocks shown in fig. P-,!;Q{l ore connected bH f'lex ible


inc"ll f e-n.s.1 b/e eords passin9 ovor (r1c+1onlcss pul/;.1:;1s . A. t A !he
c;oeffloiont or (rict
ore ~ o...,o Si.,, f1<.. o. 20 while ot /3
fhoy ore fs o.to ~ f1<. 0.30 Compu te the rno9nitudc. ~ d lN:.C tion of the fi-i'of1on Force ootlr:.g
eoch plooK.. .
1.!>10.) Whot woi_ghj W Is necc-i;.sary
Glod the sysiem oP
blool<& Ghown Jr\
P-..510 rnov1'nq to the right ? The. cocf50g,)

ion

"'

FIB.

ro

flo1'ent of' friction i~


I

0.10,

~ Jhe pulleys ore.. oGSumed to

be fr1otronlesG' .
sol.

;.#;...

fBD Of 81ook /\ :

.!JO(i

1A

/\

TA

f..,
A

JC)Olb

'

3M1"

-Ef!1- o : "'" = .:ioo(coo a6.o')


H"' - ~-40 lb .

. r" - f"- N"

(o.~)( 1-40)

FA .. +t} lb .
~fMO: N112oo (cos .s~.1)
tie: 120 1b

:. Fe f,.. tis c (o.;)(Ho)


f& c 136Jb.
~ . .!510.

reo of Blool<..I\

:
. ~01 :.tr.s.7(

"' ~.

;\

""
Fl3D~of
~~ 1001<. e_=

fl\ - 601b'

ll\

11

~Fy o : H,, "4-00 oos tJO.

fll

.He ~i6 -t lb
h 0: Te " Fe t T" t'4006in ao
Ts 2 (0.1)~.'f) t'o tioo
Te-= .294 .bt lb .

1j..

f /30 of Bloc><. C :
~~ o O

W =T8

w. ~0-t . 6+

lb .

find the leos f voluc or p ~qu;red to couse the s ys.fem Of blocks 6how In r i9. P-s11 to hovo ;rn~r:d;ng m:>f 1~11.
to the. left . The coemc1enl of fr1'ciion. '1 ~ 0.1 under oooh
blook .
bli.)

rt~P

of

~loc,k /\

-:... _:
~~
~
~
1

-~
~

:
'

'

~f~-o : N .1100- p.s1no<

T
sin ..1:!11"

~11 -o : T tf-Pcos"'\ -o
T

67.32 t(o.t.)(soo-Psino<.) - Poos"l-0

aOOlb

127.~t-o. ~Psin"I-

100

11 I

100

.sin 79,69'

r 07.82. lb.

Peas<'( -=O

p(o.2s1n"(tVOS"'<) 127.32 lb. - P = 12 7.~.2


dP p(o.'.1. CCIS"< -<Sin<) "o
o.11sm<>< t cos<><
d6'

p " 124 . 8 lb .

o . ~o:ISCC - s1n0o( eO

o. 2

= ..SinO<

..

tone>(

COS<>(

0( -11.31

_.

;1t.) ,A, ho~neous ploc;.k of we4ghf W rest upon the lnollne shown in Fi3. P -512 Ir iho. .eoe ffi cicnf of rr:ction i(;
o.30, determine the grootest hoi,ght h_ al which a fOrcx:.. P
paro llol to tho- incline ~be opPlied so thol the block. will.sl!de vp the. incline w/oot tppl~ over.
; >-<.
~ ~f.!1o = Nwcosa6.e1
Y: ~ '.
P
~f, .. o : P FtWf:lna6. &7 '
p (o.&)(WOOS56A7)

r
+

, ,,.

\ ..

. .. ,...

'II

4,97

W ( &1na6.&7)

p o. &+ w
:E'.M.<.-=O

P( h) =wcos .Jh.11(e.) Wtin 36.&7'(+)


o.&t"M(h) - .f.~ t 1.&w-

h - ..:f__

o.&+

.s1~) _In f.'g. P-~12 , ihe homo,ge.noou~ l:>loc~ wel9h6' 0oolb ~ ih


coe-rfi.oie.nt of p--.,<:;llo" is 0.4-0 . Jf h =.&In., c1ct~ine. . lhe
forc;e P to, ~u ~e motion. to impcrd
. ;a;;r~~o : rJ.qooo::is 36.e7 " ~ :z40 Jb .

\ ., .f: ""

11~"' ~

f = (O"t){Z40) - ,96 Jl7.


~NlA O : ..SP" Ws1na6-i!i7(+)r WCOS>.e7' ()

.SP r (300)tf1036,&7(+) t CXJ&-'6.&7'(z))


.SP - 1.WOJI?

p" 2+olb.
s1+.) The 100/b cylinder -shown In fig P--514 i6' held ot rest
on the ?JO' incline. b!:j o weight P suGpended frorn o cord
wropped oround the- QJlinder. If Gtipplrg lm~6' , determi ne p~ the coe-fT1'o ie.nf Of fr;cf1an.
~/"" o : Hcos:60' " Fcosao
WG!li 10011:>
'tl..coi;;oo' r-Ncos.30
(.

p,

'

)"-

>;

COGOO:

0.&770!

cosao
~MeaO

P1(1) ~ r(1)

' P

-r-H = F

~F~-o: ftGin60i Fs1n30-P100

N(s1n6.0 t 0.5773s1n30':0.s n3)=100

N 100/o.s773 - 1 7.3. u lb
p "';"'-71 : (0.577.3)(170~2) : 100 lb
s1s.) Block/\ h Fi,g . P-si.s woighs 120 lb, bloc!<. B weighs 2ex>lb,
fit..-. -the
C-Ord ;, porolbl to ~he inol1ne.. . Jf' the. coeffi'cie-nt
of fr1otion. ful" oll .sur.foVOb' i" coniovf is 0.25, ddermlnethe. o ngle. -e- of the. 1nd;<"'e. ai wh.1oh mohoi'\.
fj Impends .
I

or

120
.sin 7s.~

__
R.:_1_ _
.sin (00 -~)

R1 ~n3.7 .sin (go-&)

Fh

123,7 ( sjn9o'cose- - CoS9()Grn-e-)

f< 1

12.3.7 cos-e--

~f'l! O: Ws1n& R1Ew11+0t- -1

~005rn&~~ - o

(R1 t

R~~

Ri s1n1+.o+

Gin 1+.01-

. (R1 t R~) " B!l-1.-f am~ (j) .


: R.z C<X1+.o+ - R1 cos1+.o+ Weese
(R"-~R.) co~ 1+.Q4 200~

R2-Rt ~ ao6.!2. 006a-@

Rt.ii+.ot-'

82.-4.+sin'(t - Rt -

>

e2+.-t.s1ne- - R1

from1: R~

R1

e21.+Gin9- -R1

~ w6d!

~.z

OOS"9-

cx:x;&-

: but A 1 123,7cos&-.82fo.+Gin&- - .t(13.1~)206.~COG&


024.+ Gin&-~ "f-53.6 c.o&-6-

Ton e- -f- 536 e~ 28 .'ez. .


s::z+:+.'

or

516) Reforring to Prob. SIS, If ~he.- cocfnc.1ent


fr1chol"\.
is 0.60 ~ -<7- =~o what forc;e, P qippliod to /3 ooflng, down..
~ parolld to lhe 1noline w;il s~cu+ mo~;on ~ whod le
feni;;,'ofl "- the- cord otfoohed to A ?

of A:
1,

FflO

~.

120

,,-'-"0.6 .-. -& 3o.96

'6

12.0 :::

f\1
r~o

r1 -

of e:

p~.to~,
30.96
A'

~ 12t.;:, lb

120
SJn.S9.M

R1121.19 lb .

.("f.~ 0 0: Re.00&30.~6 r 2(;\'.)CQ630' + R1

cosao.96

Fh.
~r,.~o;

flt

.S1n60

Sl/1 .SG.O-+

Tt

R1

1!1-0 (s1n6a06)

T,

RI

the-

".S~.3 . 17

tb .

P R~.s;nao.<;J6tA1sinao.06-

..;1CJO s111 ao

Pw128 . 6lb .

Jn fie . P-sfe, two blocks ore co~nected ~ or .sol(Gl


S'frut otf-ocN;d to ooch b lock. wi~h
rrichonle-s-s pins-.
If tho C:.oomc1<:,.,t of fric-ti'on undor eoch block. is 0,25
~ 13 woehs '.270lb , find the min. wei.gh~ of .A to pr.event
motion .

s1g.)

reoo~a
: 'l'4 J.V02~
a.. o-1-..ot
-;;-

off.llf

II

OO

'-

270

.son-4$ 96

&n

1+.ot-

C - z01,9 lb.

,.,,;

iJ _c

WI\ ~ 598.3 lb

ii:t

~: .,,.
' c

szo)

'B -

fi'0 . P-~ro blook A we:ighs +oo lb


0.20, under 8 , eqmpute the minimum coefric.ient

Re fc,,.,.-;rtg to

aoo lb . 'If

f-

of frl'ot ~or"\ vnde-r A to prover>~


W~llOO

J'A 0.Z

~ ~pr

<t!'n 0 ; /" . c COG<i<>


ti . -+00 t (u1.9+) .sinso f. c(~'*)(o.'tJR:
rt = srs.07lb
fl:ao6061b
/A ~- J206.06 Q,;97

F l tt
I
In

fie

;;:, ,.: :

iF,9 ~o: ti .. +oO t csinao

fl
~c.

f OB of

S!Z-1)

rnol'on.. .

.. -(}- 11,3

'

.s1a.07
P -.S 9

.r f 03

under

both blool<-& ~A we-1ghs 400

lb . Find the- mox. vvei,9h~ of f3 that con be s torted vp thelnohne ~ oppl.Y1n9 to I\ o l"'i,ght wore hJr1zoniol force P

of .soolb.
r~o~9

_p.-o.a

.so~
f.11

C'.F.!j- o :

er,. 0

Ii " 4-00 f Csln.ao

: 600 : Ccos3o t

fBOofB : ro.3_ . 0 16-7

.o

0~

~
'

..soo c;cot;<Jo t (o.;,o)(+oot Csn~o)


...;eo .. c (1. 016) - c .,. .374.02 lb .

We

~74.o.e ( s1n-+" a)
s in 76' 7

We

s:u.) Aopeo~ illut;'. Prob .s17, oGSum1ng thof tt-e .sil"Ui i~ a


uniforrn rod wei9h;n9 300 lb Hint : f/rvd 1so/ole fho 6"!ruf
0$ o Freeho<i!J d,-qgrorn~ ~v;ng a~ end fbrce,t;- info ccm-

to

pononfs aof1ng along ~ pcrpendiculor

the fi'tru f.

- 0 _;;;:.
~

-ry.;~R
c

A.

~p

3&0

SIO&e.3

a1n 611.,.

C-32+.0lb

"'

~F~ Oj r1c5S0ta24.8(61nao)

Ii. 712,"'fo lb.


~n. O : P - P ~ c coi::ao' )'-'1~2+.0)(0. &66)
p" (0.2)(712.+) t a.e1. 2g
P- '1-.a:-+ lb

sza.) /\ force of +oo lb Is applied to the pulley show'"' ;I'\.


Fig. p-s23 . The pulley is p--cvented rrom rotating b~ Q
fore<:- p oppliea to the. erd a fhe. broke le.ver. ff ihe. coef
of rrict10n , ot the bro/l.e surfoce. is 0.20 ,det . thevolueof'P.
p

~M>.-=O

iOOlb f(20) ~ 4-00 (10}

r -:zootb

.sz+.) A

11

.'.

F*:

1%2

.$:'/l'\p~O

f1=1000/b
1000(16) - zoo( & )
P{4e) .. 111-()o - p 300 lb .
hori'=nial orm' hav;ng o bushing 21n . long,~ sli'pped

P(u) c H(6)- F() -

~a 2 ,n . d ,arnefer vert1eol

in

rcol, as S"""I'"'
Fl9ure . The.
coef of' fnction bei the i:xJGttlng ~ ihc. rod i~ 0.20. Cbmpute
11-G min. lengH"\ /... at w/c w~i9ht w con be plocc:d to prevent
-the. orm from G-lipping down the rod . Neglect the.weign+

of

tho arm .

.M" 0

W(L-1) t F2 (f..) - Hi(:t.) ~ o


<Hjtf(t.-1) t o.2(t)~ - 2p{ Q
Ott L. - o,A t QU- 2 .., O
o.-+ L - ~ " O

i.r.!JO; Wf1tfa
f)l ;.Oi N1 c N2 -

L -= ..L
O.'fo

F; "Jl'l'f1 o.2H1 =o.2N,,

1-~e>tn

F, Fa.
W F1 t ff. e o.eN1 ( t)
W 0.4H1

.s2s.) /\ un;forrn !odder fort . lonq ~ wei'gh;ng Wlb jg plaoed


w/ ono end on the 9round "*i lho other ho.nd ogo1nsf overt1ea/ wa/I . The:: angle of' rrioflOn ot oil contoof surfoc.es iG
20. rind the- m1'nimum vo lue
the ang le -e- ot w/o the lod
dcr ccn be de.fin~
tho hori~oniol before slippingoccuri;-.

or

w/

/5 -

{u::D61L....i9)

- -

,'

'rt

lt4

tI s{Tan~l-;cHtL5";" .. ~)

I
~1

Ton10 t "Ton~

:._F_~:i.~---1-----i ~ ~mMb
t 006 ,

@ {!J-M1) - - m (-x-x~
(..!1- Li;ine-) - To11 u:l(-,.-Lccr;'9-)

2Tonro'Lco&e -12Ls1n& La"o


(Ton70' 1Tonzo )

1ar'70X - L,1ne -Ton~o)ct Ton~iiloose)


)l.(Ton10' t 1onzo) " Tonzo" Lcxx& 1 lstno&

X 1on.W 1.coc~ '1 Li;in&0

Ton

fl()' Lee~~ i 1.G1n~

Ton70.- Ton 20'

T
~

co~&-

21on 20 )

oxe-

Ton 700 tTon20

- L

2 Li;tn ~ =L G1ne{r()1' 10 Jonzo -

Sin& ~ .Z. 3835

@LM&O w ( 11 - ~-s=e-) .o
1on~o'L ax&i LG1n&

Ton~ t Ton10

_yTon70.)(

L
-<:aSIT
-2

!Z

Ton-&<= 1. 1917.S
-9- a,.50 ~

lb~ .rs cC>'tcr ol 9 rov:fy


,s 8fl' frcrn the bottom . The, !odder 's ploced ogo;ns~ o ver+i
s26 :) ti ladder

2on long

we:gh6

-fO

wall ro ih::lf H mo~ an orglc:. o f 6J vv/ the- 9rotAnd. HcMfar- vp fhe. ladder con Cl 1b0 lb man olirnb befOre tt-e ladc:ler
iG' on ihe "~of sl1pp1ng . The or-ele of (riof ion oi oil contoct surfo ce- r!O 15
0

c;t;1I

r---~(St'3)

,;o\b-

I ~b
;. I

-.( t

@!1/1'11\1b -

IS H

~ 1Q'17SX

~~-~.) -m(ll-71.)

R~

{.!l-11,3~) Ton1s (x-10)

(10, 17.3)
G>J~.

VJ'

11\e. vol...e of.!i:

-,,..,-=~'4.!~
~ .1..---__J
To,.. 110' ')( -na2 -Ton 1SJ'. t 10Ton15'
o.o,
~ 11
(rorn{; .. To,, 1s) 'lr - 20
@~M~ "O w(&-8C.OG6<l)r 160{x-$) 'X ..Sf) .
"'f'O (i~ 16ox - eoO

160')( ~ 8'f0 _.../I !5.ZS

f'i .

sv.) A homoge.neou1> cylinder .3 fi.


aoolb i~ resting on t wo inolined

in d iometc.r ~ wei9h1n3
p lone os Ghow n in. Fi'C:3 .Rsf7

ff the- orgle. of friction '" 19 for- oll contoc\ burfoces com


puto the. r1109nitvd e.
the c:oUple required to storl lhe cyl1
d er rotating covnierolool<..wl&e.

or

ot ~llibrium

.f-. a: l'f~ oos1.s' rt, cor; 1s

~H1

--i=;
~M" 1!>

~ r~ (1) t f, (1.s)
E> (~o.s)(1.s)1tz oo.-,s)(1 ..s)

8 . 1+1.6 lb- rt

.:!!.F~O: H11>1n1i:' 11i11w11'.. J()O

N, ~n7~ i(t1. ~~) "'""' ,,. 300


1"11 (s1n7t cos1s to n1s)

""

111 Z8!i:P8 lb
...

N:i~77.6Slb

" I; _M.N,

:. fz Jll'fit

(torn&)(~7&) T()n H. (17. 5)

f177,6 lb .

Ft"ZO.elb.

528.) Jni;:food of' o couple, determine fhe minimu m horizontci'I


forc:e. P oppll~ to"9V'\tiall~ to ~he le{\ al the- top ot !he.
~ ll'nder deiocrtbcd If\. Prob . ..s27 to ~tort the c~llnd er rofanng
coun lerdoo~w;se, .

~"1,<.0

P6~ rz'c1~

r.(t:sl,

P{1~ = .20.&(1,q ~ n.6(

p 20.a

14

+n6

P " ~0.+lb.

A& .sh:Jwri in Fig P - 259, o homogeneous c~lindet' 2fi. i"diametc:r ~ we1gh~ng 120 lb i; aoted upon b!:! o vertieo I !Orce
P. Oe-lermine. Hie mo,gn;tude of P necessary to Gtort the.

sze.)

~finder tum'1ng . A 's;ume that


p

W..,_1 UOlb

Cb,.

j -o.30

."

l=cosao ( 1)

e- 16-7'

~ 0.666

)(

"""' 0

w(~)

Pl 1 1 l<)
p(1 t

o-B66) " 120 (o.U6)

p = 10-'.0Zlb

..SS .69 lb

':'

1.006

~ao.) /I plan/\ fOft. long Is placed in. o horiwn.tol posi t10n w/ its ends restin9 on two ,nch~ plane as -shown
in fig. P- s30 . The.. ~le of fr1cliol\ iG" zo". Oe.-fc,rminc:. how
close- the lood P wn be plocecl to each end befOre .sl!pp;
,. ,,,.,...!" p

f,,

/',J (r.ei.~)

!- ..--_s(9.s+.1:1)

1rnpends.

,O~t(1:11

\\
~

J-i

'/

"""

')J;::.====:==::::;~

!:f * ton65;< (~-o) ~ - Ton .+0 ( x-10)


- ran60;ir t ton&'('io)
Tont>sx -Ton-+O'J< t10Tan-te'
X (Tonw 1 Tan8<f) = 10 Tan"
x(Ton6s tfon-te} 10 lon<IO'
JC c 9.2+
't "'2. B1
Ms : O: P ((~.2f:. (10-ll)) :,o
~Mi; 0 0 : P(2.a1 - ~) ~o
Tan2s-x ...

2.01 -y

..9 ?.'#- - 10 ~ )( 0
S3't)

o .

.!:l =z.s1a .
x ~ Q.76Q .
A uniform plorik. of we9h W "' total Length ~L {t;;

ploc.ed os shown in f(g . P- ~1 w;fh ,~


t~ inotnc<J ploncG . The ongb or rricf

ends

1on

ib'

I"
1-5 .

contact

w/

Oelcnnlne.

1he rnoxlmum value of the ongle


\e

Lti.d.,~2

' )
~

>

,!1~m~tb

flJ{COSo< Tonoo - ~K~1n"(

~ Ton~

~)J:;O.r.6a-i1~'tran)

~ "' o.7i!J 2.

(.H-~) -m(x --;i,)

:I

o(o..:J6.2

y t 21..Gll'l"'<' = -fon60'(x -~t.C.Cs0()

Y f on ts"x
G&}~. ~:

at whioh .sliPPing impends .

~c,i)

.. ---._---w\ .
::.--<t"lf"-.......--.....

0(

J(Ton1&' + TO'l6l>') " 2LrecTon60-ll!i..sina<.

')( 2 !.00SO(Tofl60- 21Gin o<

rori1G' flonoo

.sa2.) In f(g. P-5 32, two bl~ eoc" wci9h;n9 1so Jb ore connected QJ,j o uniform hor'.zantol bar which weighs 1oofb . lf the-ol"I . 91e, of rriot1of'.\ Ii. 1.s under each block. find P dreotea parolle-+o the. +~ incline.. tho~ w,11 o0....G"e :mpb-ld;n3 rnohon to the. left .

r.solof;n9, the- bar


1!00

. ...
@

~ & 1s' :.JJ.. o.g68

Bfoci< I\

i'j'

~sou.,,

f31ocl<..13

c_p)

f"':

~FyO : N zoosin'f{;'Ha>i;m.+i;

H .202.84' lb.

IA()

)t'-NB ~OO

ft,. "'

200

sin~

~
2.00 lb

~n.o : l"a+c-00~ - ~~ .. p
to c p

(o. 268)( 2u. tt)

pc 758 lb
533.) A un1"rorm bor /\6, wcigh;~g 41'2'1- Jb , is fo5tenea b_y Cl

we(gh1n~ 2001b os shown. 1\1 the


ver~icol waif, f ~ 0.268 while under the block, f o.~o . Determine.. the. force, P ~needc.d
lo stort mo/io,., io /he, ri'ght .

fr1cf;onless pin to

bloof<..

13

1s

L~

F+
- 1
p

Av

fCO

~fy O:

.
~-o

U<f{-f~-t~) fr Lo:.+.6- rl citiuL O


11~ns " t{o.u.e)t1e-t.~~ - li'l.,,sin<K O

tf 200 t Av

fie 701 .6 lb
~/). O : P AH t- f
p ~ ,t86. "t t (o.'X.101.6)
p .+27 lb

11'9.0 0.-'18 tte

Ne = 289.-t lb

JLt

l<H Ne 289. 'f

ftJ :,.rt (o,f6&)(t.ss.+)


f s 77.6 /b .

Av +l+ t re 501, 6 lb
WE0<5ES

. . fy O :

53s .) I\ wodQ& is used to Gplit log& . Jr is the. a ngle of'f7io. f ion be.iween ihe wedg~ ~ fhe. log, dm. l fie. m01- . ongl& o< o rthe.

w<Pg~ :~I if (~)om:.:, :::~~:)P


/

2F\[61n~cas~ f Gl'l%~?J P
dP . 2R[~(-An'>YL)(t)~(cos~':(.)Q:&f.Jo

d'<.

"'
-s1n(,611.nv~
~ oos~cose

.-o

COS~ CPS</>

CO'~

.s ltl "<12

"' 61np Slfl ""'1

.Siil

tJ

CQ4: ~

Ton 1 "caf ov~


In Figure, determine the minimurn wc-i 9hf of fl lock 6 thof
w"rll lleep rt ol leroi o forco P 6lode ~lock ,A. vp I hv ind inc
GIJT'fOcc, of J3 Tf'lc, weighi of /I ji; 1001b ~ the onglo or ('ric-

530 )

tion fOr oll surfo~ in contoci is 1~ .


P

reoo~f;~
P

.tO~

too~'li

"' ~
p

Ro

"'

~1

141 .4~

lb.

r,

~
,, .

+.Rt
..
7

:E.Fi.o : R cos+.s" = R2cos7.s

frn*)c~# ~ Re - g86.4 lb
co.s 1.s

n'

- I&.- r
N,

We

R.

~Fu ~o :

We R2 sin'ts-- R1 son4S 0

;;J

Ws (aa6.4J&-1n7,s'- (141.-H.)sin4>

P..t

We 2.73. Q lb
537.)

In Figure ,de/ermine the volu~ of P. just S-Af'f;cient lo

(;;forl lhe 10 we.d9e under- the +oo-lb J)lock. The o'"Ble. of'

fr:ction is w for oil oonto.ct .surfooeG.

R1_~ ~1_9.-z lb

_
'

411:1.'l

&1n&)

6in70

pk a41.z 1b
536) In Fi9 , _s:J7, defcrm;fl<!. fhe volue of P octing to Ilic left
tho I is r~u;red to pull !he wedge- ou I from under tt-e 4!00- lb
PlocK
r"BD of/\
r----:---,--z'.

W'fOO

,.,,

11.i

R&

.
f.

Ai
R% 301.7 lb

&0"

70

_P_ -

sinao

381.1
J;;on 10

...... p ~

203,f

lb .

.sag.) If the wooge deGcrib<:d fn lllv~frotion hod o weigh!


of 4001b 1 what value
p would be required (a) to
start the- wedge under the block , ~ (b} to pull the
wedge out from urdcr the; blOCk .

or

feoofA

~t
.~

'C'.J*""'~
., ..
.
W

Fe

..

105

R1

~oo

~~-

.sin 10&'

s1n10'

Rt 601.tlb.

p.

"r76 7 lb

='r<.. O : ~ Gln r;' ~~ oru:l&-'O

R-: - ~Gtnc
OOGf~

.
~.!j 0

R, ca; G
R,1

''

~ ~ 61<1 Ito' - +)

R1 s 1n1;'

i;' -

c' 1s

R1 (c't.' -.s1n& ' ton u :

R, (o.g;a)

"'

(r,mit )

+oo

.. -too

R, .. .,.11.1 lb~
~

P7.3.9 fb

s-+o~ Ac ~hown in Fi9 P-s-te, two blookc we19h1ng ~oolb ~ resflrg


a hor-12:an/ol ..surfOoe. ore. ~o be puGhed OfOM btj a 30
wed9e . The orelc. of f"io/ aon 'rco 1.s for oill oon+oot $vrfbco.s .
wtiat volue. of p i1; rc;;quirod to sfort rnovc;mCJ"lf or the blook.IO?
Now would this . o~wcr be~ ,r the woigh~ of' Ole or tho

"'

bloci<-6- were inonx:a~ to 300lb f


+'

..fu__ c ..E!f!_
~1r

t OO

61111~

~ 1 1a.~ tb
~1 .

F8

Ft. .

'

o , zft, Qn30 P
p . ~ (7.3.~).sin zo

P.

~ 7,9. ~

lb

. , ""'

'

't ,

"

.&!...-

-- ;Ra

-~

"''-'.

... _,.,

Q"t4'C.

Ra 1og.01 lb
~F~-o=

P (1og,&1)&1nao .. {73.a) r;.nwl

p,..:Pt-Pi

P&:; 91,slf>

p,.. c

18.!l lb

.st1) Determine fhc fcrc;e P required to start thl. wedge


dlown in figvre P &tt . The ongle or friction for off surfaces

io COlloot is 1 5~
'6'11. "0: Rtcos111 -Rtsin11t. .. l!C:l0

reo of 8

R1 Rt COS1~. - 500

tOOO

llf

!IOOlb ...

:
--.+--l

.SIO 1!!"

eoo

:FycG> : R, cas1!':-R. G1ntll' :20<lO

&u~liluff:.

R.1 :

fttc::c;>&1.s;- 500\c;=f& - .Re.sinti2~


0

n; "1 .

rwbf /\

~ P~.

, , , , !"& :

~-fs

I/)

r'

_P_ 11.55.+
Jitlls

.Sll"l 1.S.

All (c.os1s cot15-) - ~col;, - ReG1n1cf<XX)

R2 (C<l'1s'c.ohs -sinis) = 2000 t .500col 1&


R2 (3.a-t<S) ' ae66
0

Ae ils&.4 lb.

""

P-

9~ . + lb

.s1n(>0

Whoi fora. P muGI be oppliOd to the. wedge~ .shown 1r\


f igure p. h+L to sfad lticm vfldor f ile block f fhe angle

.s+2.)

fr.c.1.00 for ofl c.onlool

or

(;Urf~

to._

is

reo of/\
p

w~
F~o:

2~"'5~ 10:::)0

R -'01.g ll;)

.s1n10

'501.9
.i;1n

t10

p 1.31.9 lb

the vertiCol position of o pos1t1on of a rolumn


Gvpport ing a 2000 lb lood two~ wodges ore usod os shown
in f igure P-s-+a . Detormine the fbrc.o P necos.sory fo \Sfort
t he wodgos if the angle:, <: frlcfton ot cJI .surfaces is 20~
Ncgle-0! tho fr1olion ol ihe rollers .
.5+3.) To odjui;t

FBO

of/\

~.~'

--O=~ cO

Rs1n60

faoo:B ?;5

---=-P...,. _A_
s inas

= 2000

R " 230G .... lb


4>1n6G'

rt.~ ....
s...+) In illus . Prob 53+ . ff lho af"910 o( friction is 10' ol illustrok.. off surfooe&- irt conloof dc-fc;.rrn1ne. ihe rnO>'imum wedge
angle,~ thot will ,gjve t.he wedge o mechon1col odvonfoge. ,
~ .e. mal<.G. P le~ them ~he weight W of the blool<..

_J r~.b~illle'

Ri(,.,,,.,o --<) ~ 1oao ~in100


Rs .. 964 . s

_P_ _ .fu.___
61'1(2011.)

"'n1w

1000

.s1~t"')

(61n 70--0!.)

subfitvl~R-1 .. ,. P

.Sn(10"""()

sin(toto<)
_,. p(sin(?o--'V

sin(!Oto(J

(s1r17o'cos-. - c:o&10.,,,.,.,.) tooo

(&in w cos~ -oosz.osm-<)

1000

...:9:..:;8...:.1'. _e:_

sinao (6'110'--<)
1000

cquo1'"9P1000

t,

(.stn70 CO$~ -eo&70S1f'\O<')

+ .son10~-coe1CG1fl"!S1n.zoco"'~

(G1nwoa5o1 - COS.6111<><)

Gf'l"' 006.20.

(sino'-s1n20-) cos~ ~ ( cos7o t ooe 20) 6m"<


.sonO( _ .s1n10-s1n20
- 0 . 466.?I
cos10

00.S

fon
o( w

co.silo

= o . +663
2.s

.si119le threaded Jook.screw Mos o pifoh of' c ..s in.


~a meon rod;vs
1. 75 ,n, The c,oeffoi eni ol Gfoioq tricf1on

s-ts)

or

ofo.1.s,~or l\1nofic fric.llo" ofo.10 (a) Oelerrnin~ the force


P apPlied at the encl of o lever 2 N. Jong which w ill -storf
Jiff Ing 0 wo!9ht
2 tons (b-) whoi voluo of p wlll Keep Jhe
jookscrow turning ?

or

(0 )

~o.!>

~r

tor.-e

o.1'

( b~

f1<- o.10

21r

c5,71

-6-fon~(~)

P = v;.< fon ( tP t ~)

-e- ion- (o.(Ylei!j)


fr

r"

2 .6

f.. o.f5

o = .2('1 . .2+;n

fo,.,

W a Ions 1000/bc

1 cq.&

4 "o-10

ton

P=(~(1.7s) fan(s.1(12.&)
21'

</ 8 . .S.3.
P :ti.c_ ton(~ t </>)
P {...0()())(1.75) tan(2.6f/3.~")

P. =

-+2.6 Jb.s .

.2+

p .. -57.+ Jbs;.
threads on o /r;ple,lhreoded Jock.screw ii;; % ;n. The meon l'Odius i.;: z il"I . the
coe{fooiont c r,-.;o-11"on o.10. -hot food oqn be raised b~
eJXerti1"9 o mome<i/ of 2000 lb ff ~
Pifch e % i" (.since i~ ," lriple /h~dcd mi.1lliplj th::.. pikh ~a
. -. P1"tol, " ri in
&f-6.) The. d,"bfonoe. be-tween odjacenj

fon & ~ ~(z) O. IS9


tr c 9 .03.
h,. x 1 n

12~

f Wr ton ( <P

e-) -

0 . 10 -

tontj

o. 1661 r1 .

WOO lb-ft W(O. 1667 ff)

~ooo lb-n ~

w(o.Oof-3a.s13a n)

0.10

-~;'71'

~ori (s.7f t9.03


0

~lbFl

O.o+3tl573~

-s-t1.) /\s

.shown

;n fig

P -~....., , o Gqvorc threodi=.d 8crew ~s


a pres&ure of' !l tons. Jf the- sO<'C'w I"

U&e.d in a vie;., to eiier-1


cblble lhre.ode:::I ~ hof; pitch of 0.25 if\ """'0 rneo" daomoler: or
1.6 iri . d~terrni'ne fhe- torq_ue.. trot rnV6"i be applied ot EJ to
croot& fhic; >presGurc. AsslA'l"e the coerncicni of' fhc hOn. cf

rr1Ct1or\ to be.i

0 .10 .

j - o.1s

Pllvh :o.2siri ( r:nco double. fhn::o.wi

fa"; =0 .15

/J c863

lo" a-

ton' -tooa 1b6


T W r tan (t(> t ~)

{;J

rrull ipl'1 b~ 2)
in
- o .O.S31
11 (1. s 1~
O..S

~ Ian- (o. os31) ~

3 .037

-t<:OO( f.s.,,.x
fon ( ~.53 t .J.037:)
T = 102.dlb - ff
'
6+&) 1" 6ing/t;. lhreooed .sguore. .screw hos o 2 Y~ ffy-code / 1N:::n.
1he. root diameter is !Z.6 i" \_ thoout.sido d10motcr is .3 if\ - The
c.oefr.'C.ent of r,.,;of,o"' "" 0.10 . Def~ne Iho m0menf nc;ce.ssary to sotar-f Hi ng o Yertioal oxiol lood o(' -ro. 000 lb . wnof
moment is necessary to sfo.-.l. rowen~ the load ;'
p. _1_ . _L
'
D m Oo - ~ .. a - E:.f. "' e.e I"

if"'-)

:z.:;in~

TPJ

P c o .1'1n.
T~ Wr for- (

:irr

t; t &)

.. "fOOOO/b (11' it

., -fOOOO( 1.-t xii-)

8ELT

ton e- -.Qj:_
.n(H)

fan&- .. o.o+GS
-&- ,. ~. 6

1 ~) fon ( s .7f

T~ 681.St lb- rt
Tceo - Wr ton ( -t-e-)
T~ ~

o.+

2.6-)

~
:.

rOI

r,.,

1.4 ii\

Ion, - 0.1

<p--&-71

to~ ( ..s.71" -z.t'.')

253.6 lb - {1
FRlCTfOti

tv,...n.s or-ou~ o t;fof;ono~ ho N:20nlo1 cir-um ;,.. ufred to -suppo"~ o heavy V"c:q.,t . /f f he coem'GIC/11 of f r1olio" i~ O+. whoj wC:.9h i 00'1 be .suppiod bg
eiier"ting a .so -lb (Ora; al 11-.e olhor oncl or the rope.?

.sso. A

rope- mok.i~

1 14-

.o'j.;.,,c:>nloo\ \<: (a6o go)

l ..isolb

...!L ~f~ -

T1

r1

Ta

= 4'90

lt eJ~ (.so) eo.+(<t<;O~)


.so ( 12,B-1+) - 11..s7 lb .

buf T1W -" W" 11~-flb

.651) I\

rope ~pped

or

f vlexi oround o poet \'Ii II .support a


+ooo lb wren 0 (Oree, of 00 lb is exerled ot the o-

woi9ht
ther ond Cb!enn1i-.e \he, ooofTfo1enl cf" (Ncf1or. .

~'501b

ongJe, or cen!ocl : 7VJ


Ln

~.ofO()Ofb

~~

f~ ..

Ln 80

Ln

~o .. f (1w)l.~J

f ( u .01)
=> {

~
12.57

o .a-19

or

I\ boot GAC:.rtG a plJll


-tOOO lb on 'o\ s ro'f'<eer w/o ~
wrapped obout o cop~ton on the dook.. If' lhc [ o.ao, how
!Tlon!:j lums must \he how.ser mokearound 1he oopstori eothol
S S.2.)

tho pull o I the other does not o)(cecd


Ln .JJ._. jp

..50

lb.

Tt
~ Ln lv'r~ Ln ~ Ln 90 1-t.61 rad

o.ao

0 .30

= 837.1
6:37.( 7360 = t.33 turne.
5S3) ;\ torque. of 2-fO lb fl . o d G on ~he brokedn;rn s hown in fi9
p-,ssa . Ir loo broke bond ;s ;" con\oct v:h t he. brol<ed.-um
tt1rov9h :use
the coe(Jl'ci"en t of fn"c t;Ol"\. 1'1c o-ao . Oe-tern-:,ne
tho fOrc;e. p ot the- encl or the bro l<.e lever .
J.f..61

x 1ejf

""

'"

(o.a){"50' Vie<>)

T.
_.JJ...-

12

.. T~
~M,..

"-r_,-

3.702.

(T,-h)(a~J<-fP...._) = 2-t0 lb- ft


To - T11 = .360 lb

T1

T1

(0.711.gg) = 360

To

"160lb

3.102

T1

-H33.23lb

.:M11

=0

~ 11 (.2 iA. .x .1C:L)


P I../ 16"'- ... .1fl.)
12A
laj'P,,
p( 1..333 f'l) +9a.a3 { o .1667)

p ~ 61 7 lb .
.s.s+) /n fig P -SS+, the. c.O.erf:-101ef\t offriotof'. is 0-20 bol-woe.r- -!he
rof 'ifii the, fhoo drum ~ be-tween. oll surfOCC!>' ir.. oonfocl . ~el"
mlnl!I the, f'(lin/mum we9hl w to pre.vent downplaf'\e motiol"'I.

or the

1~lb bod!:1

ry O

ri1

W COG 36&7

~F... o :

T~ -F1 Weina6.87

T~ -(o.2)(wwr;36,87) Ws1na6.e~"
Tz

l,

.o!!:Qj - o .:
..2'.r)!. -o: T, t

11

f'-111

= 0.76 W
ti'L 1000 coca6e1 + t11
tl2 ..... eoo -t n,
f11

.f ..

+;~11"

1000 &in a6: e1


600

b.Jt T1 1.sn~ Ji1 -wd,,J1,.ti1; 1.s1(0.16w) t (o.i)wc.os <J6,ti-1 1 0.2(.rotwCDs~ll.

r.. =076W

Hz".8<>qtH1j
I

;600
H !lW t o.J6W t O I 6W

=600 - 160

PtW -<J-+Olb

I,

t.!i.2. 9 lb .

In F19 . s.55 , o neJI; ble ber ~ run& Oom /\ over lhe ccmp:iund
pulle<j B y,,., bocl<- OVer P to o 200-lb weight . The- coe;fr,o;ent o(

SSS"-)

bciween /he be-It ~ lhe cornpou.-.d pu,lle._y P . f ind


the rno~1;.111.Jf'll wei9ht W ~ho1 oon be supported ~/out rotof(n9
iho puff~ P or Glipp;ng fhe, l)e,ft on the;, pulle>~ P.
fa
(o.at&)(wi , n/1
~...:=.,/.--N,~..,9-'-~
Tu =(}
.
= 1. 6+g

friof O"- i11

Yr

,0

T3 1.6-tg Tt.

bul Tt W/~
l

T3 t.Hg,(w/z)
Ts "' o. B24.3 w

i/Ylp . 0
(..!1)T9 t Teh)-(2tf~ - (_3)T1 O

.s rt. -

2T;5

311 0

..s ('N/:1.) - 2 (o. e2-+~)w


o
)

8(;1 t"W 600


W I 7o+ 7 lb

-'3 (!Z.ot>)

.. c

C hopter 6
Force

S~tc:;m~

in Spoce

f;OU Dciermino tho mogniiude of the resultant. ii' pdint;ng ~


ifi; direction c.osines for the following system of non -coplonor .
C()nCum::n+ forc.oS . .300 lb ( 3. --t . 6) ~ -40? lb (-2. +, -5); uio lb(-... .!'J
-3).
Q>MPonlilfTC> OF

f()RC E

A"aot:>.lb. ii
e. '4001b. -Jl.

(0)

Jt

--4-

- !I

6 71

!J

-3

7.07

r. 1/X) .... -+

roRcE<.s:

pu;T,

"

JC

.,.e1

~ .. ~.A~

p.

.. I\,. " 11s.2+ lb.

= o.&13

. Cx "-11:L1s i Cy= 141-+'t; Cz .. - &1-. 07


~- ,. 115.~

" poiht10q

230.~

+ 141 +1- -

~ -

230-+7 - 298.06 -

boc~'MlrUs,

\.. upword to the lefl .

- 11Q-23-113.15 - - 117. 11- lb.

- 153.65 t

o.76

(Os~. 1:s2.-f6/it~.90

7,07

~y ..

117-~<U-00

o.3Q.S .

~~y = 226.21/',e9G.00

~- _k...H.....-~- ~
0

296.90 lb.
m

.. B. "-119.23 ; By= 1239. 45; BJC. -tl9S.06


~

~ t ~i-' + u"
= (-117.1-f)" t 226.2..~t(-t~-~
..

Co66;. il/R "'

...aii_. ~- .la_. JL_ ...100..


"
y
)I!;
0
(;.71

-t52.'44i

oall2 +y-t:t

3()Q/M~1 l'w/-4 . . ,Ay c -153.65 lb.


3<lOh.e1 A~ .-. Az " 2.30.41-7 lb.

"

- a..&A7

~26.2f

-6....

I(,

.300/7,91 A.,_/a

-:zge.O()

.i-t1.-

- 117.H

ioget OiGtol'lGC(O) use


y

ll'.CDMP.

2304-7

-m1.1s

lnl'AL(~)

)(

YCCMP.

rns.2+ -153 ,65


.-119.23 ~ ..+s

lCCOMP.

2~6.24

lb.

81-.87 .. - 132.'16 lb.

fnc

603.) Doter-rnil"IG
mognitude of tho resultant, ih; point1119 and
its d 1i-ecl io" c;osines for the follo..ving system of non -coplonor.
concurrent forces. 100 lb(2.3,4); 300 1b(-3, -4~!l); fOOlb(o. o. -4).
fQRCE

COMPO'lettti; Of' o
'l(
y
~

2
B~lb- -3
C:r !la:> lb.
D
A fOOlb.

-+
()

TO'il'.1-(<,)

45

...

0161'.
(D)
.630s

7.07

FoRCE.S
x Cllh!P. YCOMP.
,:17.14

)<.

Ji!

~~

.',/\.,, .37-1+ ; Ay c.ss.71 ; Az " 7"4. 28

B"/ic " By /y =B:z/~ "' 36<:J/7.07


.. B,. .. t~7.2ga ; 8.,.. ~ -169. 7.3; 8~

74.28

()
0
2.00
- 90.'!.5tl -114.02 406.44

..6x_.. ....6.x....- 100


y

~c:c:MP.

-127.,.... -169.73 2H1-16

to got(o) uso o = x2 t Y ..+z"


~

.SS.71

212.16

~"

37.1-+ - 127 298

- -00.158 lb.

i 0

:%.y -11.+.0!l lb.

.::Ell - 7+.:t.0 t 2.12.16 ..

20) ..

.
406-4-4 lb .

Rt "' (~><)2 t(~)e (~z) ~


.. (-90,1se) 2 t (-11 4.o~)t f (-+-06.-H-) 2

R 507.7 lb .

po;ntlnq forworcl \..down io the lef\ .

Cas"<7>< ~~/Fl. =90.1ss/.so7.7 "' a.11s


"~,,/~ "11+.02/.so1.1 - 0.225 .

eo"ey

Cos-ch ~z/i:t

..

4e6.+4/.so7.7 ().Q.58

or

604.) Dotermino tho mognitude


the resulion~' i\s poinl1ng. ~
ifc; direction cosines fur the foll<>'Nlng sy~em Of non -c.oplonor,
conGu'rren~ forces. 200 lb( 4 . 5,-3); 400 lb(-6,44- ,-5); aoo lb ( 4,-2,

-3.).

..,,

DIST.

FORCES

.lt:.

lO)

>(COMP. YCOMP. ZCOMP

.s

-3

7.07

11~15
1+1.+-4 -e-+.01
-213.66 182-t-4 -228.ll.'
2 22.6iJ - H1.a2 -166-98
62.12 21:2.56 479,g

COMPOtleNTIO Of' P

FOJ:tee

)(

..... .. ~It>.
4
s 4aolb. -6
C= S00 lb. +
TOT>.L (r.)

-.s

-2

0.77
.!1.39

-.3

togei Du.Ge :0 2 x"-1y"-1~e


~ 6L~=~ ..
y
.i!:
7-07'('

~ )(

;. A .. "11.3.1S ; Ay "1"\1.44; Az -&t.07


.fuL_. 12x-~~- ~
JC.
y
z
0."17

~ a,. -= - 273.66 ; a1 102.44 ; Bz: "-220.os


c. .fl:... CL 300
>l
y
z
.5.39

.. c.. = ~22.63

Cy" - 111.32; Cz.

- 1~. ge

:%..,, 113.15 - 27.3.GG + 222.63 " 62.12 lb .


~y

11'1. ++t 102. +-t -111 . .:12. 21'f...!)6

\~ .

z - B+.S7 - :228.6.5 - 166.90 " - t79.Q lb.

R ~ " t.~.y'"t~z~ .,. (<;2.12)-i t0212.S6)\(--..79.g)'l


R

..528 . .53 lb

cos&"/.

62.12/..s2e.53 .. 0.110

cos e-y

212.s6/s2e.sE o. 4a~

q;>s &z.

c
c

+79.9/..s20 ..s3 o. gos

:. po;nti119 bockwords, \ upwords 1o ~he right

605) Three concurr-ent forces P, Q., to..., F hove o resu ltont of' ..s lb.
d;<'ected forward ~ up ~o right of -e-" " 60, -f7y = 6o'. -f7-z- 45. PeqYOls 2() lb"' PoSSe.s ~ hrovgn .the oragiri '!he p01nt (~ ,1,4 ).Tho
'IOlue cf~ is olso .zolb a.._ it poss= i hrough fhe paini (s, 2 , .3 ).
Peienn'1ne "1he m ogn'1tude of ihe ihird force F ....., the angles
rnokes with the reference axes.

01sr .

Cohlf'IOtlliNTs; OF 0

FOP-CE

= 20lb.

&~

2010.
p" P'

)l

1
2

.s

x
4

FORCES

lO)

"COW'P.. YCOMP.

9 ,73
16.2.3
?'
ll.6

4 . 60

6 -16

Jr;C()MP,

4 .37

17. 47

6-"'t9

g.74

("

;<>.9

i'
3,94

whel"<! : R "S lb. ; -&,. "l>O ; &y 6o": f71r;"' -tg

..!2_ ~ __&.... Pz
y

)(

.. Px

i!!

~S.7a ;

120
...... !18

Py- +.37; Pz 17.+7

'!!

-&" CIJS- f/F

cos_, 22.46/33. 68

-e-,.. ...s. 2

s ( casGO) "' 2.s

~Y ~R eas&,. .. s(cosGo'),. 2.s


.~ = Rc;o&-e-;r. = ..5 (cos4s;) - a ..54
fy

= 2.s

Fx2 t F.,. ~ -t f;r; ~


:!

(-2~..ot6t't (-a.'!'!16) t{-a.t,7)

F = .3.B.68 lb.

616

:. ~ii "16.2.3 ; ~ 6.+g ; ~z 9.74

z.~ = Rcos&i.

~= .:t_ ~ ~
J(

-et1

-Goy

cos- e .%,/3.3.68
4

7S.6

-G-2 COS- 2.3.~7/33.60


-8-J. -.~.3.
1

- +.37 - 6 . 49

fy .. - 8 .36 lb.

...

Fx 2.s - -1 6.2.3 -s.13

F,. - 22.... 6 Jb.

pon I '19

bockwa.-.::1.s,

"i dcwn......o.-.::1

to the lefi.

Fz - :a.s+ - 9,74-17. 4-7


Fz -

2:9 .~7

lb.

t\ force of 100 lb is d lrecied fr-om I\ toward 6 in the cube


shCM'n in fiq . P - 607. De-termire the moment of the force obout each
ol the coordno~e oxes .
607. )

J,"

F/

,;

// / /
/ /,;
ii
~

: // /

.., ' ,, -"


<

fv

I/

'

,/?'.)

v...

1,
~

~ I0~.3

-R

'

,,..,.

da:-tt3:zt-+~

4
B
~

f,.

-:;-

fL.,.

-4

100 lb
c; . <l-03

,.. p,. t;2.!5 lb ; Fy .. ~ .9 lb; fz 6~.~ lb .


:11:1w1. fy(4 ) - fz(+)
I .w;,g ( ....) - ":Z.5("t)
~IY'h " -":2.4 lb-H .
~My .. 'Fw(.+) 6~.!5 (+)

~Mt -Fz(4-)
- 62.5(+)

~My 2.S'() lb. n.

f\llz - 2.!lO lb.

ft .

608.) A force of WOlb. IS d'1recled from e toword c in the cube


's~ 'an fig. P-">1. Deterrn""e the moment ol the force obovt

or

eocn
t

i~

c,oordino-fe oxes.
~M-

f,._ (1)

.LIY'\y ~

'f

d 2 t3 I "t
d .&.:ie..s A .

~48. 6

th- rl.

- F.., ( ,,,.)
-1~8.'i(+)

~My -.S94t4' lb-n .

.-!:Mz f,. (1)


..

)...J._..l-.._...

~Mz .. .s10.g

fa

:. r,. '""" 1b. ;

FY.(+)

- 7.of,3(1) t 111.+(-t)

lbff .

Fy t 111,.... lb.; f1L1<16.6 lb.

60Q.) /\
shown in

forGe of 300 tb. ic:direded from 13 loword 0 in lhe cube


Fig. P-607- 0Gten1'1ne the momenl of the forc.e obout

ooch of the coordinqte

0>1es .
y

!
I

d 1 o' t:9'"1'
d .5 fl,

~My " - Fz ( +)

.. -Q'40(-4)

'
J5!_. x.. ..f.Lc 300
0

.'. f x

...

.!5

o ; Fy 1BO lb. ; Fz. 2'1Clb.

~M.,

.. -960 1b-fl .

z"iz - Fv()
.. i00(4-)
~Mr.. 720 lb- rt.

~l;'\)I

fit (1)

.. 2-4-0 (1)

%Mic ~ lbH.

45t0~ /\ furce cl' '400 lb ir; iirecied from C toward E in the cube
~ in fig . P- '10 Oeferrnine ihe. momenf of the force obovt
eoch of lhe ooon::lino!e oxes .

111 ..

Fy(+) -

fz(... )

.,, .a26.S(+) -163.'i l()

d 1

+"42 t:Z a

d ...g

~M11

n.

6!j:z. & lb-0.

My ~ fx(.,..) t fr (c)

163.3(-4) t 10:!t.9(2)
~My

.. 079 .8 lb-0 .
-f,.(4) - Fy(2)

~l. ~

:. F. 163.3 lb. ; Fy 326.S lb ; Fa 163.3 lb .

-163.3(4t)-.326.5 (2)

Ml! .. - 1306.2 rb- 0 .

fOt'Ce P. d irecied frotTl

f toward 8 in the cube s'"'""'1 1n Fig.P601 . cour;~ o momet'lt My 1600 lb- 0 Oelerrri1ne P 'ill, als moment
obout the x '- z ollCS .
611.) /\

d' -.'+s .~ c
d,5.38:>

n.
:. Py =600 lb. ; P.e "'t<>O lb.

-*'"'1t Pa ( z)

1-MJ -Pr.("4) t Py(~)

1600 Px (2)

: - iOO(+) t (>00(.:Z)

eoolb .
2L L
:.
p,.

lJ.385

M, " - -tOO lb -fl .


%"11! - P. (+) .
c - 900 (+)

P1077 lb.

lb-fl .
612.) A force P i' directed from o point A ( .... 1;4) to.o./Ord o poin'f B
(-3 .+.- 1). If ii COIJlOCS o momen~ Mir 1000 /b-tf, Oelerm.1ne ihc!
men~ of P oboul lne X ~ Y oxer; .
M2 - .3:ZOO

""

~- ___sP!.____71

d' 1t~'+3

/' :. I ,,,, ,,' '


/ '
,

d. Q.11

:-!-:.-:~

...f_

G.11

.&.. &.. - 137

:. P, nyg,11 ; Py

3
&

.311'9.11 ;

Mar .. p,.(1)t Py(-+)


1900 7/g,1, (P) t .s(+)%.11

p.

~11

lb.

P.1: .!1~.11

p,. 700 lb-; Pr; 3001b; Pz - s00lb


~M~"' -Pr(1) - Pt(,.)
~ --!?00(1) - 700(+)

M11 "
%.My

-1700

1t>-f1 .

Pz (-t) - Px(}

.!500~....)- 700(+)

:,t.'\y - - 80 lb-fl .
61+.) The .sheor-le9 derrici-. .shown in Fig. P- 61+ suppod" o ....ed ical
lood of 2000 lti . oppliec;I ol /\. Points B.,C. "'Dare in the. .some hOrizontal plone ~ /\.O.~ D o~ in lhe XY pl9ne. Determine the force
in eoch member- of \he derrick.
.
2
iY

i . - -- 2 0 '

d,...

10'--J,..(1o,1s.o)

(11>-0)2 +(1s-0) ~(0-(-s))c

O,..e 1S.708 .jl .

d~2

=(10-0)2 t(1s-o)* t (10-0)

d ....c w.616 fl.


\Ill
2
d....,1 ~_[10-(-wu
t(1s-o)

fso loting Top View,

Isolating

fro~t View,
10'

c,. c

.iMe"'O
Ox (s) - c~ (ls) . o

"

Cit =[11000C1J/1i;

c 1(,66.7 lb.

LJ._. ~- ~ : J2.I...

33.&+i'

1s

ao

- 0"(10) 1 Bit (1!5) Go


Bit [2000(1o)J/1s
B>< 1333. 3 lb .

Mq.c,& 0 : UX>C(10)- Oy (c.0)~ 9


Oye1000 lb.

c~ zM.c O

.L&._ c C
20. &16 10

~..Qi_ . Ci!
1!!
10

l'\C :

13741-. !) lb .

'. l\D - 2:236.1 lb.;

Dx 2ooo fb ; Oi:. O
AB" 2-494. .:a lb .

b'l!) .)'

ber.s

The fromoVW"Ork shown in fi9 . P-615 con.sists of throe me


/\!::>. /\C,"" /\0 whose lower ends .ore in ~he .some hori ~on

to! plone . i\ hori:r.oritol f()rCe of 1000 lb octing poroll~I to the 'X


axi& iG applied o1 /\. Determine the
in each member.

force

~ (6-a)'(6-o)' (o-or

d..e 6.7o&
~(a-o)~ +(6~)s t(o-(-3))c

d.t.c

7.3!1,

d,;.(3-o'f.. (6-of +6:;-0)%


d .-.o B.367'

l.atote R igh~ Side View,

lsolote front Vio.w,

zlAc O

..i.MoO
-tOOO(B) t Cy(9),.0
Cya62S lb
*Mo,.o,c

V O ! Dy(6)-1000(6)0

ay-10001b.
.oil.~

6.'106.

U'

/\C a 765.'- lb

e,.

0 1 37Slb .

~, +

/\C Jd_
7,u '

1000(~)-Dy(&)

AO .s~ . Q.+ lb.

"6 "1110 It>. ; Bll .500.lb.

616.) Refe rring to Fig.P-615. replace tne 100010 force by a verlicol downward lood of Q()(){) lb. Determine the force in eoch
mernber under this revised looding.

ly
!

A(.a.6..0)

d...s 6.706 fl .
1
d ...c -A~-o)~ t (&-o)'' t (o-(-3))

(6.0.0)

d...o -vf.a-<>)1 _. (fro)1

1 (- o)

d>.0 S.367{\.

lsolote

l&olole fronl View,

Jl.ighl Side View,


y

...

.tMoO
( '2()00 1000)(5)

.Mo.o.c. 0
Sy(6) - 2000{3) wo

:.Mc.

7$&

~ -~
6'

-o

Oy(e)-(200:>10C.\'.l)(a) o
(Jy 37S lb .

~-~

By 1ooo tb.
6 .708'

-c,. (6) 0

Cy =621'

_t,,Q_ ... ~

'

AC 76J5.6 lb.

a.367

'

AO .. s22.9 lb .
011.) The pointi; 0, C, S., D of the c;on I i\ever fromework iohown in
fig . P' 6 17 oro olloched too verlicol wall . The 0-lb 1ood i~ porollel to the z oxiii ' "" the 1200 lb lood is vedico1'. Compute::. ~he f'on::e

in eoch member.

!"'

d-'fl ..k~o-o)"' t(o-o)~

D 0,6,0)

.., J ,,.

(o ..! (-+))d .:. 10.n '

d,v:, .. /(10-0)" (o-o)"+(+-o)" 10.71'


1
d.-.o /(10-0) 2 I (6-0)'t + (0-0) 2 11.(,6

.1

' r'

-t

~o.ci...,.,.,}

. o;,? _..q_____ _
c ., .'!,,,"''

..

,,/'' co.o.~')

1l!001b

lc;olote. Ff'Of"lt View,

1.solote Top 'View,


~lt1c.

-0

e,. 11'00 lb .

..,
Mc.o,e - 0
i2o0(10) -

o,. - 2000 b .
..M- _!?L_.~
11.66

10

Ae - 161S.5 lb-- c

MeO

o. (&) o
10

/\0 " 233t lb - - T

ROOO(+) - ..otoo<'.10) - c~(e)

c .soo 1b
~ ~
1077

10

AC 538 . .5 lb --C

61&.) The unsymmetricol cont"tlevel" f rort1ewof'k .shown in Fig. P-61!,


.suppo"t' o vel"ticol lood of' 17>1b of A, Pciont.s C tt... 0 ore in the
(;()rne vedico l plone whil" 0 iG .af+. in fl"Ot'I\ of ' -fhi.s. plone. CJmpuJe

the

forc;e.

in eovh rn:>mher.

d...e..{S-3)1 t(o-(-,))1 t(+-0)" 8 .1't.9'


d ...c /l.a--0)* .. ( ..-a)' t(s-o)* 1a'
d.\o /(e-<>) 2 +(.+-o) 2 t(o -(-~))' 9.1~'
~

IGClote 1op Vievv,

.1:Mo :0
c,.(10) -1d00(~) -aoo(3) ~o
.MG,o~

C1 = 8"1Clb .

1100(e)-8" (10)- By(~) o


e,(10) t Sr,(a) ;. 1"6a::> - .. -ff)

-1lQ....
12

.....@__. -.fu!_e~ 6~

e.77.9

:. Br 6/15 e,. ,

~ (10)

a1

f.C 1%0

-+

~ c tJ

6/., e,. (3) 1 ::1~

1000 lb.

0.. 160 lb

:. B;r" 1200 lb.

/\0 160
9 1"8

'-.

-a-

AO= 1133.3 lb- -T

Bi .: 000 lb .
61g,)

lb-- T

1000(+) - 86a(9)- o,. (10) '0

5Ub6. lo 1

,'. /\e " 17SS lb - -c

B~

---g-

the 11c:x> lb lood in Fi'g. 610 acts horironthe direc.hon rr-0m E toward/\ .
from Prot>. 'fl.. 610

Solve Prci:> 618

killy Ou~wor-d from/\

1'n

dAe = S.775

n.,

d"'o

d,oc, "

= 9.168 O.

12

ft.

lsolote

lsolole front Viow,

1op View ,

Cy

"

c.. "

.a!t.10 O

~lr'\c:,o 0

- C..(..o) t S,.(l.) +0E(3) - 1700(_1) c O

0"(10). By(3) - 1100(-4) O

c,..(10) ~) t 1!(3) - 3"400

B" (10) +By (a) .. 6000 - -

c,. - eotb.

.M- _fu_ .. ~ Jk_


8-77!1

!I

'"

..

:. By 6/!1 B,.

- sub8. to

10Bit t fD/s e,. (a) 6000


13,, g,. 6800

a,.-.sootb. :.

ey~lb;

r
AB 877.~ lb - - T \ ea-4<X>lb.

IC

00
-0-

/\.C 12olb ---C

'

zlt'lc;O
- o.(~) -S.() B;i(.9)+ r]Q'.)(6) "'o
D.(~o) .. - .!oo(+)+ <f.OO(a)+ 13600
o~

,.12M lb.

~ 1200
0.1~5

..AD ~. 1~-.+ lb :

620.) The frorroewdr\\ shown in fig .P- 6W support.- o vcrf1col


\ood of 2000 lb . Poit'ts B, C,~ 0 ore in lhe some hori.r.of'\tal plone .

Determine 1he forco in eoch member.


IOOOlb
/\ (O,',O)

~ ~ l/J-.--4-)f t60-<>)t t (0--0)'

d.-.&

1<>.'7703 fl.

tPv; (o-o) + (10-0)'

d"c

'-.
(-4 , 0 , 0 )

-t

10.7703

(<J-(-4))'-

fl.

dAJt1. (P>fJl. -c)'' .. (iD-o) (o -C-s)) 2


dAO - 1"'h1"'t

.fl .

lliOlate Front View,

_..,

%McO, -0,,(&.ti6) tBy(.+) ..o


By= 6.66/+ Oy

1l

--

zMs "'0, ~<XlO(.+)- Cy(+) -Dy (12.66) -o


. . Cy(4)~ Dy(12.66) 8000 -..:. .-
.

" .

IW

lsolale RiQhl ~ide View,

Ma -:.0, Oy(!i)-Cy(-t) O
()y 4/5 Cy - -@ ~u~.

ll(;W

to @

in,

I\

Cy<'."4-) t +/.5 Cr ( 1~.c;G) 0000

c., (1+.12e)
Cy ;

in@, Oy =

't/!5 (sQ;.~)

n, By = S.66/+
:.

~
10.'77()3

0000

.!)6Ci.2.5

lb

.....!53 lb.

(+.53) ; 9807"'9 lb.

99().7.+!5
10

l'IB 1056.a lb--C

vertical lood P =0001b opp(1ed to the tril)Od shown in


fig. P-621 cou&es a compress1Ve. foree of 2~ lb in leQ AB ', a
coropreS;Sive face of 263 lb leg /\.C. Detecrnine the force in leg NJ
'-> ihe c.oordinales "Xo ~t.o of i\s l&Wef' end D.
621.) /\.

d-e -./.o-c)' t 60-0)2 (o-{-e))2


d"8" 12606,
8

co.o.-s)

dN:.
.

J..&-o)at(100).. i (6-o}

d...c"'

-14.1-t2 ,

lsolola f ron t View,

lsolcne Righi Side Yie,,..,,


800

G,r

~Mc OJ)

-Dy(, 20) I 800(') - By(t"t) "0


- ...oo(, o) t .+eoo- 2799.6 O

:. By. 199.Q lb .

- 2-400

..!:__ ~ - ~
1~.tu:

10

:. Cy
~Fy

~ .JL_
10
o.rn1

200.1 lb

.Ma O

Oy (xo)-0y(6) O

[zoo.1(elJ/'400

'l.o .

- :ZOOt . 4

~o 0.991

0y 19g.9 200.1 -eaoo


Dy -4()() lb

1Co

-4()() ilo

H .i'H

Oz ['4CO(o.9g7}]/10
Oz 39.6& l'o

/\0 i..t60) 1 I "IOO' t(39.09),_


/\0. +32.7 lb-- c

+'

_.t_- ~-~ ,Y.._


1-t.ii-2
6
14.1-.2
6

c- 160.1 lb.
~

. ,

-120.1 lb.

ou.) In fig P - 6U, 1f P 1~lb ~ lhe COON:f,noles ~Doro


~L o 2 i1, compvlo \he forc.e in eoch leg of t~ tripod .

Y..,,S '

d..a ./(0-0) 1(to o) t (o - l-91)1

cl.-.e .. 12. 006 ft


dl\G ... /(6-0)T. .. (10-0)~ + (v -o)1
d...c : 1+.1'1-2 f1 .

o...o ~ /(o -(-.!))1 t(10-0)1 (~-o) &


dAO

= 11.356 f1 .

l~lol&

fl"()n t View,
~1:1

4!'.Mo O lJ
Cr(,.) t 1200(2)- ey(1o)o o
.=;==---c:..L...::~

By(10)-Cy(-4-) .. .2-+00

--

--

GUbs. (i} fo@.


400Q;Cy(13)(1o)-Cy() 2~
1~000 - Cy(~~) -Cy(+) = 24()0

-Cy (ao)-- - 9600

Ct 3zolb.
:. Sy.a ~ooo -(320)(13) ~ 36e lb.
.5

Z:fyaO,

c,. t()yt Sy-1~00-0


32Ct

Dy +368 - 12000 -

/\B = .366
12.SOCO

Dy~ -512.

AC

lb.

320
~

1++-H

/\C ~ +.s2 . .s lb-C

/\0 471 .3 1b C
----6.Q_
11.358

.S12
10

1\0 -

.s01 . .s lb -c

6%3.) Oeterrn1ne the mox1mum safe vertical lood W

thot con be

supporlocl by ihe tripod srown in fig . P-623 without


ing o compressive lead of 2~lb i'l any member.
w

ex~ -

o(o.,,o)

dM ./(o-ft)y+(o-o)"t( -o)i
do>. 7. ~3 '

, B(-a,c,-.s)

"-I -;r'

(- 2,0,4 )

-....C

doa = S.06::t-,-__
'
,...__,...,,......_...__,
doc -/(-0)"1(<.-o)f t (o-o)'

--~--1--1..
,, _.
F '

"

dos ~o-fz)} +(G-0)2-. (p- (-e))1

(<~,0.0)

d GO

7.211

.,. if. DA.,

2400 lb.i\

24()()

....6.a_ ~ _&_ - ~

7....3

:.

'

....

6-+IA-S lb; Ay '!Q:M-36 lb ;Ait 1262.91 lb.

lsolole Righi
w

lsolote

!;1<:;je Vie>N,

Fron\ Vie#,

Mc. .. 0
A"'(<t-)

LM,. o
(S,.~A.y)t -Cy(.,) o

-13y() o

Ct 1(~~ "60 192.ot. 36)

1{12'4-.~6(..-) By(.5)

a,,

1539.<fSe

chsclt.. if 00 721-00 lb
__!m_x f .53'1. f i
~

Cy 1'731.Q2+ lb.

..OB 2068 ..56 lb., .


. / ., , d()C16

ooi

(;1.o;.,.o

,r

oc :.;. 2AOOlb
_QL- i731-0.2+1b .. CX>2061 ~ lb
7.211
6
/ll does not~

check

.111,..,o-o
w(<) , c (;~ -=o

w(17.a1.92<t)(:s) s1~.01b .

lt 08 2.+o()

~-.az..-~..L
~06f

Ar(-+) -Byle)o

l\y -. '-'/-+(170616) n.32. 1 lb

,f'

O.A.

223-2.7

7.~

DA~ .24'00
'

0/\ ' 270-+.s lb oo


\'Clive~

not OCO!:f>i

H\1s

if e~ceeds 2-tcolb

:. 00 'f 2-iOO lb

:. c'I'

<1:391 .3 lb

0 . 1006.9 lb .
Ce ~ o

at the riQh~ side view,

check

7.211

,:J

B. =sgs_;w lb i Sy " 11~.1~ ;Bz i""6IM~lb .

zMc o,

woc f"IOO

~-~21._ ~
()

Isolate Tep View.

z"1t. O

1331.:s (.5) - Air (o) o


,..... _,. 739- 61 lb.

checl<.,..'

if

OJ\ 7 2.f<X> tb

OA

1d1.8

73g.{,1
1

7. -463

CV\

767. 25 lb.

;. 1his connol be a~l ~use 1t w"ill ~

1he hl"fii\

~ic;h IG 2<10lb
0

; . .so\e mgl(. value. of W iG 61~.S .8 lb .

El9,UILl6RtUM f()l. NON CONCURRENT SPl\CE FORCES .


The plore shown in fi9 P-626 GO~ 0 load or 1ooolb opplied

62'-)

ot E ~ "1s ..suppxkd in o horizon~ol ~t~ by three ver\icol cables


otloched d A. 13.~C . Compu~e the tenGOf) ifl each coble.

Fl'Ont view,

"I
B

RfghtS'deVew,

.c

1 '

tJ

l10e0

fC 3 '

te

zM,.,o ,

~ 0, - 1000(-.) t C~O) O

-C(,)1 1ooo(6)- 0(9) ~ 0

C -4-0albT
.ZFr O,
A I B +C - 1000 "()
"

1000 - -WO - 4\00

= <i<lO lb

/\ '10<> lb . T

6v.) Solve Prob. 6~

ir, in add1l"o n io -tre

-1000 lb ; the. ploie we1q~ 1wolb

Cen\rti1d :
Y~(1,~(9)j '/j(3){~(1ofa)

fl 1.

oC-w!a)

+yi(G)(-yi)(~o/.3)

10/3 fl .
y{(j!fJ(g) r 'fo (~)~('1 Ys(5))
~

.e

'Ji M(yfJ{ ls r;.)

f,...,,...+

toke

i:F

-1

,,

:t

I
c

J.

to/ 1
f1c

M,,,8 O

'

II

'"

~lt\t.-0

120l(10/1) 1 1000(1-) - c6o) ~o

,_
r I

fol<e. Righi Gici-J \/law /

View,

1200ls) 11000(.i;)- Ba::>M- ~ (CJ) O

0001~-r

B 800 lb T
ZfyO.
- A H 200 HOO'.) 000- 800 0

.A. 600 lb-T

Refer, to unsymmetri'col conti lever framework do.scribed


in Prcb. 010 ()(I pogo 1et6 ~ repeated hore os F<g . P-629 . .If the -.o--1.COI ,lood of 1700lb IG sh'1f1ed lo oot ol the rri1dpo1nt mem ber /\6, compv\e the cornponen\s
lhe reocf1on al e ~ ihe forces in \he P,N; N:, ......_ J\D.
.i;21,1.)

or

or

dAS ~ /(e-s)' 1 (O..f:f.))' t('t- o) .. 0.11~ fl.

d...c /<0-0) 2 1(.+o)t+(e-o)' 12 n .


dNJ - l.s-0)1 , (.+-o) t (o-(L))a "g.16&R.

@ Cr Ci<f 2 ;

Cr. . Ci<

.& .

g ,165

~
6

EL .
4

OJ
2

@ Dy D,,.12 : Dz D/+
Sll~@1'. lo@

(O.f~

G./2)3 -(c,.tO..)iO
El.!1 Q,. .

A!OMc.o O,
s,.(10)+ By(5)- 17oo(e-u) o

a. (10) 1 By (.3) 9::1.so - -


~Me O,

(!Jr Cr)(.3)-(c. t D, )(10) 1100(.i.5) o

(o,. C1)(3)- (c.,. 10.)(10) 1 too ~o - -

+ +2.!50 co

0 ..!5 C,. "' + 2.M

o, +C.. .too

- -@

EAe- o SJ

Top View,

Oi (a)-0.( )-C<l:) C~ ( ) o
0uf :
O./+ "-...
c
(o./+)() - o~(c;) - c. (~) t c,. (+) o

o. .

c.

c... .5.2!5 o. - -@
subs.@ to@

Oi< s. :z.i1

o, .!;()()

(). 001b

from@ , C"

AD . [eo(o.1c;s)1/s

AG [4M(1eU/.,.

AD c 91.6.S lb

N:, 12GD lb.

~~2.s(90) ~ '4-:ZO lb

lo Fron t 'l/iew,

R.efert'1rig

in (!) ,

.MoO

a. (6) 01(3)-C..(10) 'o

..SOO( <t)

-13 ~ (3),.

600

a.. ~ 400 1b.

Bx(6)tSi () ~ - -@

in CD,

.Mc o $
B,..(_.) - B"'(~) - 0~(10) 0

a .. (+) - ei.(.!9) 000

- -

-800(10)

8y {a) 93.50

By , 14-SD lb .

Af)O(j) a....@

B11(1-)-~ 000

.J3s!65" 4'200

Bi< ( w) +

e,. (10)

~ .sllOO

a. .500 lb .
Th& Boom BE of lhe st.ff- I~ c:!en-=1c1<.. showri io Fq. P - uo
rololl!l<A IO<-wov 30 measured In a hom:onbl plane- The rnod
/\B ,, ver-licol "-. '1s supported in o .r.ocl<at ol A . lhe point" I\ ,
C. i.... 0 ore In *'e so~ ho"";:ionlol plone . Oe-le~;ne ~he forces
i"i ~ho lt19i:: BC ~ BD \... U1E> componen\s of the l:eorinq reooron ct A
630.)

'i>

deo dee = frn;

~l.1~o

~o O,

/\y(5)- !!J<X:>(S 1 &.~) "0


,Ay

Ct t

i'

Cy(10) - JC166(.s) = O

Cy"

~Ba

.rm
in

lb .

. . ec

: . BC . ~

136p lb.

r,>

C,.(s)d)...() - soo(G-") O
(4(~)

Vie>N,

t - -

<O,

~M-- ~o

~;cJ.,

l&olole Righi

leolob Fron!

c 7$$lb .

15

"4 .

c.,. t ()y

9~

Dy(..) +:too lb
Dy. 866 --

"1& (),
-Aii,(1s)-t (c,.+ 0,.)(1.5) t .soo(&.G6)-(Cy 1r>,)(,s)O

: . AO

-/t.. (1s) 1 (c,. 1 ~)(1~)-0 - -

Dir .. 183

""~ (221.1t'1Jf...rQ

-te.a

~
le

"o 2a!.s 1b .

..j17!J
$UbO .

--::;- -:us-

:.

o,. . 61

lb .

:. c... 227. 7 lb.

l\w. " 280.7 lb .


Di 61 ('.9.)/5.... 61
lb .
, I
Ci! ~ 221.1~/s.' 227.1 lb .

Referring 1o ~li9h~ ~Ide l/iew.


~IAe O : 11z(1s)+(!k t CA.)(1s) +Cy() - Oy(s)-~ (5) O

Az

&

- (l127.7 t~1 )(1s) - (.83(s) t 19365) t l1:!iiOO


15

A"' -

1~.7

lb.

631.) The boom BC of the "'iffleg deNicl\ shown in F'9P~


is contained in the XY plons . The rno~l t-.13 ls vertical ~ ro.sls in
o .sooll&l ot I\ , Poio\-s /\ ~ 0 o,,, in t~ same hOl"Ion lo I plo~
Points 0 ,,.._ ore in !he .same "ertical plone. . Determine ihe-

forc,e,s in the legs BE ~ 60 ~ ihe com!)'.)1160ls

l
A
ringreoc
nono t ,,
.
.t 10 C

of the. bea -

dee ~1' t (~o-e) i

(10-0) .. . ~us

deo /(0 - (-10)) t (20-0)1. (o-(-i6))1 .[6oo


lsolote Fron I View
lsolol6 R.gh
2

View'

side.
0

iO

..&.:..

""

.c.Mo 0 ; 1000(110) - Ay60)-E,.(s)o


/\y('J.) i E,. ..000 - -

()y(10) - E:z(6) - e,,(10) ~ o

.&M& O;

0,.(z) ~~ - Ey ( 2.)o

(b(&) t1()()()(ao)- Ay (10)-A. (s)

.tM.-. O,

e~J'1o Ey/1r> : .

fo/isl:y
()y(t.)-1cltSEy - Ey(t) " 0

;..1 (1o)t/\ .. ($) - o,cs) 20000


1\,1 (-) t A.,. Ox -+<XlO - -@

Dy (t ) -

1000(10) - Ey(10) - O.,.(o)- El'(.!1) '" O

eul : Eyfc:i E><,Ao


20,. ~000 -

@ 2E,.

Ey~ 1.s~~

1oc;o(.i)- (1.!) e,.)(.i) - Oy(t) - i:,.

Bu:

-@)

#/.acey

Sy~

1000

1.i; e,

+!;:.,.

Ei<" 2SO lb.

lfy 1.s~ c -t. ~(u:>)

/\y (z) -1 A - Oy/2 : 4ax:> lb

Ey

!lr(2.) ' (~) 'IOOO

dOOQ .

8~d~o 8E ,.S1.5.4lb .

usrng

!.y(4) tA.Ca) - Oy eocx>lb~

(4;\y

'40/1.s Ey .. 0

2EH >10,1i0(1.5.e,.) 100()

0,. Dt/2 - &0b.s.iD@

I 1()0() - Oy

""'@* 0

Oy 1000

2!0,.

-o

.'. l:i. "' ~000 -0t)/2. subs. to

r>t/ao D1/10

A~(+)

f:l

(),; -40/30 ~ Gu~. fo

M;.. O;

"lf"ll t

-@

== 37.5

lb

Using 0,, -40/.ioEy 'IO/so(a7s)


Dy "soo lb.
80/JiO ~/20 : . BO 61.V! lb.

8000

- Oy 1000 --@

U~rig@ Ay(,..) - Dy 7000


f..y ( +) 7000 .soo

.... ,._,_ -5

~A (t) - ~ - 0000

- ~ =:Gr- " 7000}

Ar ~ 7.500/+ .-. Ay 1875 lb.


0-ing Ez 10/10Ey =#15(375) " 2SOlb .
02. [00o<10llfto =-

/\ (2) 1000

~ !ii()()

lb.

2.so lb .

referf'iflg to ll(9htside View : ..F, O,,

A.,._

Dz -

~. c Q

A~ I!~ - Oz 250- !280


:. Az O

=o

n_,

~32.) ~ boom of
di$rric"' shown 1n F"ig P-63a. 1G rotated back
30 from the Y:Y plane. Dele.rrri1ne tht- forees in BC \...BO~ ~he
0

compone,ntG of ~h~ beoring reoction


.

8(40,.0.0)

en

(4'No,ll,--')
1

b . &1n-:w (0) .s n.
0

S)(X)Jb

)( '

I
.,,

or Hle. ~ A'6

the .i;oci<e\- "

10 C0$ ~

1~~. a,;;;o
-J;ll'Ho0 pl) /t.O

i -- --~
i

8.-fi .

.. ;;;o - 200.
00 ~ 11.SZ fl.

.',

OA . 10 n.
.
(O:o,u.~)

ax.~ .. 10/c;&. . .CA ,. 20 n.


a./Jo-w)tt (~<>-a)"t lo-a)' " wfl. e~ OA t x
doo I (to-o)' +(2()-0)it(17.32 0Y dl&.2.flfl. l::.. -10 g,(i, .. 18'"" n.
dee /6t>-o"f+7i,)1 t(o-~t7.!>t)) 1 2.6.~6 fl . 'OC' ~10too60 . 11.~2 n.
'./. . a--f6!7.- "

0 _

ton">

"

(w,o.o)

- -- ---

-1T.3Z/6A :.

de:

lsoiot6 F~\ 'l/iew,

BC ~ 577.316

t&.'20

:.

2.0

ec " 816.3 lb.

a
1nEq.
Cy

....,

0.,

,oP

.'l

2t>

A. - c. -o~ ... o

-A, (io)2coo(16.6'}f.
3732.

c,. ..511.3K; (10/~)


: 268,6!!9 lb.

2.0'6 658 t-577.~

A,. 9661b
Ci. .S77.31G (i7.:?i.2)

lb

.tt-1.-.o G

0, 11s+.f>&+(10)

"

1.0

(Cy 0y) 10- 2000(9.i;t;) O

20

~ 577. 31il lb.

Oz 11.s,.:6s+(11. ::1~)

- -~)'

Cy ' Oy 1132

lb

eo ~ 16.3u lb .

Refern'lg to Froot View: ~fit ~ 0

"' O.
::E:Mc.o.o

lsolae

1732.- .S77.316 - 115+-'&+

H.~e

Ay ~

A"

Oy 1732.

00 1154 .60+

___..'!~- ~..-- .

Cy

2.0

P.igtr~ $Ide View ,


.~-:

Oz = .999 .<Jc; lb .
Rel. lo R'ight Sde view : zF.i: -o

011:

Cz -O

- A~ .-

A~ ~

l>

~
0t

- - -- _ ,...

~--------~ (;

u.32,

"Y

:fMo '

9:

.999.9' - ~ . 96
At. .soo10

1'T.;'lt

'1

-373!t(iT.ll2) Cy(i+.~) t2ll00{'12a2). o

Cy(~.C-4) H~09S-24 \~

CentroidG

Choptcr

&, Centors

or

Grovity

<:K \he shaded oreo shown in Fig


P-"'5, which i6 bounded by the JC oxi', the line ')( O ~ thepa-

10s.) Determine t he cenir-Old

robolo yO

KX .

J/' Kll
!:f/,< b~o

K-

y2 b4-}{;. /I -

f:

foa b % dx
~[~]=

ydJC -

Ax f.xdl\

- o/.fQ" [20"'}'3 -oJ

- foy.(ydx)

. r:" cb.rv.ro)dic

A-

01:

- o/.ro [ 2!""" ~ f%s (o)a/e J


/'I~ -j.ob - ""!."lslb

- s-

b%

!J

~~ob

Aij ( ~ ydA
.. ("

}o

:4 ~~die

..

Y~

r: ~ (~d")

hJ""
dx
lo 122
a..

A~. - b% [ ~]~
~~. b'\t&.

'

Determine the centroid of the quorter circle .sh<;JIM)


P-706 whose radius; i~ r .
t y, r"
A ( !fdll ~r'-ll ch<

'106)

x"

' ~ = r"- x

Loi ,..,. Niln<o<


,

; . 1>in-t - ,./,..

I\ ~o rco, .,(l"(:.()s"<d't)

.. C<lSO\ ~
- ~

Jo

OJ

)(

---r:--

lo'.h ,..o cos ""'d-<

r:

Ai - )xd/\ ( x('1o~)

-~ r"i~voos~)do<

( i<Jr~... dll

_c;[o\ + s1g.:zocJ:1'

- [ ~ .1:fi(r-,.o)'All,..

r-~,G) y, r']

ll)a

A~ f h ~di\ ft.f,"!!~ dx

rcoGO< .. Jr"...ll,.

~['% + &?f- o -~1

A = Tr "

. v~ (: (r - )te) dic

Aij

.d::::J"

. l"OXO"< cj.(

- Ve[r~ - r~l

~ Jr ~r"
+
~

,. ".!:.
sr
~

,.
- ~ [...",,. - "7s1,
2i;%'

or

the centroid
!he quodront cf the ellipse
!Ohown in Fig. p-101 . 1he equo lion of ! he ell ipse ,s if. + 1. c 1.
~
o" . ii'

101.) Determine

9~

1 - ><Yo~

y1 " b~ (o - 1112)
o~

.Ci

ij

}i.

r .r
yd:x

y ' % ( 400 - 7:a )


ox

bjo Jcl7dx

Let " 0&1no<


d'I

=0

>-

=(A 104.,.-,.-.)dx
0

1\ll: - r ll(ydx)

. .

COS"< cJ

s n<><.

"lo

A::1

% [-Y1.:"'%(0"-x")9h];

o/o[-~(o) + y,o"]
A ) : 'Sac. = ~ ~

Jo..C-y.L

1-

y: - ...a/f ir

AiJ ~f:~ y(yd~)

:f!C"

1:j d ll

",C'"X. ("c::-x)) dx
- b_hoc

ro.,. - )<~1:

. b/zo2 [ o.~

a.%] - ob,h

~ij -~ - 9 '" -H>hr


Compute the ON!O of \he spororel in Fig. P-708 bounded by
ox1G, the line)( -b, ~ the curve ~,.Kil" where n ~ o. What is
location
,ts centroid from H'IB line :ic - b ? Prepare the toble of
709.)

the

J<

or

he

oreac: ~ location o( cenko.1d for ..olues o f n o, 1, 2 , ~ 3 .


( t>.h)

!J IO."
!.(

= t,j/"'" = h/b"

~~hx%"

.1L.
[b """"''']
Ot1

]: = b (nt1)
nt:z

bvl ihis ~ is
reffered to(o,o)

x=

b - b(nt1) - b(nte) - b(m1)


--nt'2
nt2

709.) Determine t he

Problem

709

J\y =fob Y.e y( ydx)


b

ot

y coordinate
= 2

r:

H fl

the s pondrel described

i0

y11 dx
[. 2nt1

ll

lb

Y2 Jo h ~n dl< = h~b2n ~nt1 -Jo


h~11n [ b21"1t1 . 2b

2nt1
_

oJ

hil. (b'SQt1-tA
2 ( !Zri t 'I

h(n+1 )
4 0 t 2.

710) /.aco te

the

cen~roid of
c or;;;1n 1f'}(_

the

s;ne c urve y

the oreo bo1.>nded

/\y f>12 f1 ~dx

~ 11

J: ~

11

chi

~ ~a .s1n 11 TYi. d)(


- }4 Cl~ 'ff r'J)I - Sin 21/\lL

/\ "l yd,. I' o sn~x 011


-=

b~ the X ax i~ ~

trom x " 0 to x = L .

o7f (- cocoT~),~

QL

l..

-., -ov.-[cos9f,,. ~ c.oo]


-.9..h..(-1-1)
l

1 ~=~
T

-~

...

lo
B

711-.) The dimension~ of ttie T-section of o ~r;;;l - iro~ beom are


Ghown in fig. P-71+. How for is the oontroid of the oreo o~ the boes.
1
~ 1 1-/\y = ~Adx

T
e

g .. [1(B)(4t1)] +[ 1(,)(0.!!)J

[1(9)t1 (G~

1+ :i
~

..

43

.. ::i .o7 in

715 .) Determine the coordinates or the centroid of the ar-eo


shown in Fig. P-ns w'1ih respect to the given o~es .

Aij : ~Ad ll
flH6)(9) ~T(a)i] _ij

- [~(,){9)(4159)] f-[2T32l(~ t 9]

4U4Y - 307.23

9' 7 .+ 7
01

"l1.1'' t '1 -

in .

[V2 (4i)(9)(~.r;)]

[lleT(,) 2 (a5]

.+1.1"1 ~ , <)6.+t

Ji "

2.3"t

in .

716.) A i;Jender homogeneous w ir-e of uniform cross secfion ~ betlt


ioio \he "t-q:>e shown in Fig . P - 716 . Oet . the coordir'IOtes ~I a~ cen -

tni1d

[6 I f (+) I 9] ij 6(3) t 9(+!0tt\90') t


26.56"6

f(+'f._~)

g- =66

y 2.+e in
+ 9 (4-COs~o t -4)

-6(+)

16'. 1>6Gi: -

ecs.S"x

35.7rl0

~ -1 .a+r'n .

717.) L00ote the centroid of


'19

homoqeneovs

"-> of

tt>o

bent wire shown in Fig. P- 111. The wire

uni form cro~ ~lion .


~

rs""' _ a an :so
---;c-

'*

2.e6S in .

4 01' ..

.s1n:ao . ~96Sin

..:. :g 1.+a~in.

r2(2113,.. c~,. ~&?)) .t1 ~ f2(oal<(ad, !,ao))(u aij]


t

e.293"9 " e.9%


~ =1.086in

11a) Locate the oentl"Oid oF the shoded oreo sho""n in r:g. P - 710.
y
f(c;)(~2)(Ys)t Yz(4)(4ih Yt(,X4'1i1 a [~(,)(1i)(~, ..

,)1

r'
1

[ Y2 (f)(,)(~ 'fl -t
[1 MM(/3c.)]

...
~
721"

"[111 ( '

oi11 .

)(12)(YJ125J

]2t,. 360

ii sin.

-tfY1t('X')(o/9 .,)] +[Y~)(,)(o/a 6 it 11)1

no~ Deter mined the cent roid of the lines -trot form the boundo -

ry

or

the ~hoded orea in fig. p - 719.

[ 1~

t-

f180

+Ji90 (~h)( o/.[i) +f72(-f7%)(V-{1)


.sni(.~)('1.n-) t6)1

t 6 t ,Jff

+ffe] X

y: ...

. 49.39'

1:2( 6)

s. ~6 in .

g .. te(,)t6la> .ft90[(~){Y"'5} 6] ,Jn(~)(Y{f) +ffe("%)( Y./1)


+Q .397 ;r ~61.659

g" ..s.+1 in .
7ro.) The

lie

()11

cen1roid of the shadod

ar-oo

in Fig- P- no

"11;1

required

to

the '( O/llli . Oe.l ermine the d ifonce b lhol will rullfill th"u

requirem~nt

[ ~(-.)(e)J[ne] [Y1lb)(6~[o/sbl
es."333 in 2 in b 1
b' ~ +2.6665 m2
b ~ 6 .$3 i11 .

n1.) Refer to the T aection ol Prob . 71+ shown in Fig. P- 71"1- . To wrot
value shou Id %e 6 in . width o f the nonge be chonged so thot the

centroid vf the oreo Is ~ .sin . obove the boe ?


(1(8 )

b(i').'i>.9 ) 1(g)(H1) t b (1)(0.!!1)


!lO 9,. 5 b s "IO t O ll b
I

~b

~o

b - 10 in.
I

722.)

L.ocote the cenh-oid o( the shoded oroo in F19- P-71111 cn:x:il..d

by cuitlng () semicicle of d lorneter- r

from

quo ....~er c;rcle

-"';"1~ ('"~")(~~ -(~" )(%)

r~c

Jf- ~. rh - r;{2
Jl!!.r:
. sri
9
~
2

1"2 q

-1' -~ - o.637 r
1 2T

~'AX~.,')-(Tr%)(r1i)

Tr/a~ r~ ~i

11""'% -

1G"r

"' 6 .S7Srll
't$.6

l).7~C

n-

O.JJ"\-9r

..-a

$T r

or

~a.)

Locote the cenfroid

or

the shaded area in F1q . P-72.s.

[ Y2(1-1;)(c;)

j"

~ +.s ()- !('}1x .. 1.e(1.s)(c)(V.6.s)t+.!i)


~ (.._s){')(~.as) -: lT~..)(#)

24.-43142.

:i<+.125

x .a.o+ in .
2+.4:a142

g .u(G)(a) + (Y2)(1.s)(<;)(2/11.c) - Tr('>~(t; -

2+.-'l-st+:z

!i-

" Gs.see

q-

2 . 6a

~)

in.

fhe coon::linotes of the centroid of the shaded oreo

12.+.) Find

~..::::....:>...,.,,.,..,.,.mi--t--ff2{8) - Yt(t;)((;') V,4(B)"(Y)-T~)J :;< 12(18~)-~~')


(f/,()-t 12) - T ~+}7-(+) - T(:f/ (18 - ~~l)
If

144.!!93)( 1118.Sg-1
j( .. 7.74

il"i.

1+4.S93g c 12(19)()- '/2(f.)(1;)(0/9) - lT()"'{12 -

~)

- 1f (6) 2

1++. .s93 9

(12 - 3T
.+/6))
~

7.3i3.7e'!2

JI s .09 in.
cerih-oid of the shode.d areo enclosed

rze.) Loco~ the

b~

the a.irve

9 2 o><' the .stroight lone .shown in Flq . P-no. lf1nl ; Observe tho+
the curve !:1 2 01< rela tive to the. y Ol<is ,~ of the some !Orm oG ~ ..

Y)Zi

1<-x 4 with respect to the. X o ;ais .


2 0

><

[.2~~(6) - \111((;)~~~ ~ =[%(11){io)(3/5n~ -[yq&)btX% -12)]


12Jl -

)C

10

12'

.!!}7.(!;

+.a f'i.
=[%(12)(,)Wa '"~ ~ [~ ('")i1~)(~'5J

3"

i~;J
g' : .30.

7"7.) Loc.ol-e the centroid of the culvert -show~ ir'I F:g. P ?2? . H1nf :

,r

t~roioon

to

y,,_

is umeoessory
11->e G>rooo is subdiv;cled inlo th& element
be found in Table vll-1
~--i 1
[z(Y1l)(~)(s)112(c).o/1(.,.)(14)y r(Ye)M(,)(c;,) 1 12(6)( ~)

--+ .

-t"
,,

-(~/s)(4-)(12)('/e't5J

..S89 . 204

~ -3 .6ft.

osi'"9 int&grohon. compute i for the on90 bovnded by


tho l< Ollio, ihe. line. x 12, tt.,. the CUl"VS H .+X t
x.%e .

7'.) w ;lhout

%-

l'I

~';ldx

~,. { ..,.,. +l<Ye - "~e) dl!

A
I\

Ill' t
~S2

Ax = !LAd-.
+/

~/~+ - ,../,,

~~

:l'c

1J
0

+ll.. x/.i - x~ ) dx

~x'; x/~2

J.

- x/no

U01f1s

Tn.) Ii. rectangle ic: divided into two pod~ by the curve. ,y 't<.~"
o& Ghown in Fi(a ..P-729. Using it'io knOONn locoiion of the oontroo
of H'le lower port I\ o~ given in toble v11- 1, s how ihol the.cen troid of' the. upper port El ," locoted by j B Y:z j A 1', ~ B 2 gI\ .
Y _ 1
6iven thot y,.. -ln!i)n ~... b- JL
......,

\.i(it2

nu

I .

.. b(nt2)- b
nu

F1i-d ile ~?111

bhl~e
[ b(t>)- nt1"1

. . .?frv~
ntt)

bh(~)-lbh \(b~\
tnt-t.,"/ (i1tif)

bhn ie - b'h _ b1 h

n11

ntF

~ b~li(ntt)-

tb ..h

f(nt&)

;e " b~6v -si


(nt&)

nH

~,,

b(t s
1

2 l'ltt

~~ bh (l}4)-(~[~

PJL. - ~
.. bhttin
t1) - b n"'
~
~(<tnH)
..'hh:!Lge

lbh~Qi\)

(O t1}

~,,

~(<1nt1)
a

h(nt1nt1)

:.

B"'. 2y;

7JIO,) A boom ho9 lhe Oros' 6e0iion 6hown in Fiq. P -Tc!O, Cornpvte
1he. moment of oreo of the Gh<;lded poriiOti obout the hori20niol
cenfro10ol O'<l0 Xo Of tl-e,. en~ir-e se,o\ion . ( tlote : i\ lQ ~in 6"!.-on~'rft of mo+a~ol~ thot thic l"eGul ts is ~ in campv~l'l<3 H'le
)(tinum ~heo.;ng $\ t~ ).

n"O -

[ 1(' ) tU ( t )1 ~1~)(1)] ~ t (' )Q3.s) t 12(1)(7) ti2 (1)(0.ll)

' ._l_

3()~

171

~ -.s .7 in.

~M,.;,

1 (') (7,a) t ( 1) (7.!1)(-3,GS)

~~M 7.a.+ '1n~

n1.) Two 10 in,-1.S.3

fig .

P-731 .

_ib chonn&I~

Find the moment

the hof'izontol oeo~rO;dol ox\Q

oreo of

to.9El-1her oi; :shO"-Nn 1n


or the upper ohorine-1 obout

ere welded

ol O"GO
l(o

of

the.

enf1rc,

Geofon .

10i" - i.s.:9 It> +.47 in~

!i +.+1 (~
g 67. ..sSc;"I'
.,g - 7,&6 in

2 (-4~7)

t 10) I

+.'" (-')

9 .9t

or

A 0 rd in 0 bridge, fruGS i<> oom~ed


the elemenl Ghowti .
in Fi'e . P - 1a2. Refo- to Tobie. v 11 2 !Or- +he prope..-fies of the onglss
~ locore \h~ ceotruid of the bl.iiIt up cec-t ion.

7a11.)

1ax1

[ 9(1)1 "(l'..)1a.1s t4'.1SJ ~

,,~)~r,to.s) 1 11-( 'fr.Y._o)


(1.ce)

+ us(14-o.,.,.) + .s ... s
!K;, s;

5 - 2_79-347.s
!j 1<MH ;,, .

7.3!) Locate th& centroid of ihe built - up Geciicri ~on .Qhown ir'I

Fig.

P - 739 .

Refer to table v11 -2 for the properties of the elements.


{ lllf<J.7.5 t l(;(o.a)

(~)] ~
t

2~765

=1s(1Ho.ir11 -'~ I .s.-,5(0.-.s+ 0.211)

6.,Xos)(1i.,2e) 1(~'f.o.1)

g : Q61. 403

Jc9.~

in.

7,,.-.)

/...

ro9h~

frion9le

c:oinoid1"ng wifh 4id& h

or 1:"1cJei;i
lo

b~h '1c; rofotecl obou Or'\ o"lt i9

gsnero~e o ri9ht ciroulo,.. cone .

-. .. _ .o.
7.57.) ~iv<!I file e.1tpressi~ IOr- tt-.e. c;vrlOc.e' o-eo ~ volume genero
lod by rolohng 0 sem1orcle

or rod1us r

v 11. ~. Areo

of

obovt i 1~ o :omle.r .

holr c1r.cle.

-~.,: . *. T r't.

V +f~ Tr

A 11. ;: . le.n9-lh of ovrve


-1 f . 2~~.Tr

I\ - +Tr 1

voff""'Obol...d air 'il A

'tlT ..'l.J4. 1.bh

'5.

b'Th

Determined Hle vo\um$ of' the ellipsoid of revolution ~


fed by rotolinq ori ollipr;e obovf o ) "tis no.Ja- oxi~ ( Pf"Olofe. ollipf;()id)

1.s.o.)

~ 'I>)

1'5

minor- axis (oblofe ellipsoid

1ol\e the

the. smoll&-- .ge.mi oi<1.s os b .


a.~)~,~1

.. i

A'IU.UPll;'B Tob

lorqu- sem ~o,.ii;

oG

Vp~V.lE 2T. M . "


~-+b . Tab

5iR" ,_-

y =+rab'"

b)

.tioat ~fb' 1

~fo..
y.f

1-

~% ~

~ a}'bA (b ' - ~~)


Yoe>-Arc -~l .19_. ifob

A.1. (b 11tdy

~f.:o~(~"-'j~Jy
- Cl~il r~l, ,.rsti>

ft

-fTo11 b

o}b'fb -W$ +b- bfa]


1

A;

o/I [tb%]

T~h-~ = 4 alli

j"~

l"IC.) /\ circle
o d1'S+once o
wi II .9enerote
a i- groofe.-

.9T
of rodius r lie1> in the 'XY plone wi\h ,+~ ce<iter at
above fhe X -o 'l<.IS . Revolving 1! obou~ the X 0>t1s

o do1.J9hnut - shope ring co lled o torus, prov.1d&d


ttion r. Gompule the .surfoc& area ~ volume.- or +

tor'IJ.S .
V '= !Zf.ll . lr~

V21f'r"o

A Jo2ilr
A "flfcra

7.+1.) /i. 6'0 pipe ell;>ow hoG an i11~erno I dlo meter of +in . fhe rod .
of curvoture of lhe pipe'~ oent&"i1.Y1e IG 6in . find ~he interriol vol.
of' too e lbo.v
V Arco x centroid ~d1sfonoe. teve rc:ed - i()~ x c; x (oo' x l/180')
V = 7g . 96 in:'

7~.) find fhe volumo of the .sphef"icol we09e rormed by rofolir"19

lhrough on on9le

or

+.!I' 0

.semicircle of rod1u.s r oboul 'd~ bo&e

diornetet'

I\ l~"l- 11.7'~ . (irr/

/\ =Tift

Toto/ &iurfoce Arco

A.,.,..-

~ Tr
0

+ nr "'

7~.) Compvfe the .svrfacc oroo "} '<'olume '3,-,e.-ole.d by rolo~ in43
1he oreo .shown in fi13. fl-7-+3 throu9h O'"IC rcvolv+;on obout the Xo~~

"It-

/\19. ~!!

c:'

[!ifl. t ~] g ~r-T~L J[--4/j) t ~ -''{~][ fl~l

- - +---.&.."

l?1. 6&('

g - 132~ 73 2
~

_v ~ riT.
.'1
T(,.)t

~ ~ f~S''" ~

6 - 1.5.

=6.1..s in .

2(rn5')

1667 1h3

T(-+)( ~'i>- t

' )

fl(')

t f!.

(~ll't~ .s1n"~+J

!17. , , ~ - 1$~. 66

:g- - -5.vin.

A ::?f. 5 .7.

27.IJ9

A ~ 1002 n~

The r '1m of Cl pulley hos the cros.s ssotion ..shown in l'i'<;j P - 74+ .
If the- rim ii; rnode- of .stee-1 we.19hln1_3 490 lb per c..u .
rleform1ne. Hie

7+4-)

weight

n,

of

the

,..irn ,

11.s

'1

= 1

s.s

i1 e 1,61ifl.

Vff .(1. G1 +10) .11 :~


_

--

.10'

r. . c..')-2(1:"2i!l)] g . 4(4)(-1.) - 2( ri)(+-%)


V= g3g ,9 inll

V = o.+e.s.s fl 3
w o.4es.s ( 490)

w"

238 lb .

7..s.) ll.e oreo conioined between two concentric -5erri1c;reles


1.s i~ . ~ 3 in . iG f'ototed obout on o)C ;s 4 in. owoy ~
porollel- to !he boce diometerc of the oomicm?les. O:imput<~
fhe surr~e or-eo ,.., volume generoted b.Y 0 compfe~e re.volu -

of radii

t1on .

[TJ r~ -1.s)] ~ ~ <)' rW21 - %b-~>1 f."f~1]


~

!: :I

: : I i +'

'\Q.(;0:3

i? :!

g - 16. 75

~ "' 1.4-85' in.

v. 2T. (1..+et>t-t) . 10.603

" _aGs..+ in~


ihe surroce oreo ~volume generoled by o complele
re'<'oluho f'l obou
X oxis of the .shoded areo of' Prob 719 .

7+6.) Ooterrnine

he

~ " ''"

5 :o .!i in

v - 2f . B. Areo
: 11r . t: . 1!2

V " !2114.!I

/\ ::2f .s.+1 .
0

/\" 164+. 77

74-7.) Compu te

1n ~

(1HH~
in.

2Vt7)

o~ ~volume gerleroted by o .c.omplefe

the. .surf'oee

revolufion obouf the )( a>a i~


J .s.oa ,n .

or the Ghc:lcled orea of Prob . 72~ .

v .. n . 0.09 . [ +.s (G) t


v 779.92 in:11

'/2(G){u,) -

T()%]

A~

!Zr. 9- le~lh of c"1rc-unioibin9..,Areo


/\ 211' . .:i.9'-['1(11}{ t 6 t s t J,+ 1 .~J
A ~ +92.S7

'1n.z

.] for let19th

[3." t :r~ f~-. .. ~ B.. a 6.s) ...+.s(o)~ ~(,) . ~ .s'l-c~(-J,,,:" )(Y..rn-)


.

+ T (~ ((; - ~)

.
1'}.9 97 ~ - 7 0 .<>20
~ ,, 3.~;

in.

the ceritroid of o hemispher'C!


the axis of' .s;ym:mefry as the z oxi~ .

1s1.) Deter.mine

King

of'

radius r. ta-

A. un; form wire is beni info the .sho~ shown 1n F19. P - 752 .
The .stro igh~ segment~ lie In h$ i.-z plons, ~ the 9 in . length
makes on angles of 30' wilh the X axis . The -sem;c1~ulor ~og. rnents is m ihe 'lC - Y plane. Locofe the.. oen}er ol g~+y of Jhe w;rl!
754:)

(H11(.+)1g)x

-.(,.)(-+).,

e(e++~:ao~

:i'.s")l 14-1.91'1

i w.s;,:a+ in .

/~:...,,g

-:

T('+)(~r1)
.. ~fl

'16.,s,, g

:g 1. 204 in .

. I

'26-.s&' z

:;lG. &6'

'(a) t e ('!-.sin ao)

z ~ .9'4'

.Z

1.28

in ,

or

7~.) Locofe the center of groviiy


0 s teel r ivet hoving 0 C!,jlii'idricol body 1 in . in diomefer ~ 2 1'n . long with a hemispher10ol
heod of 1 in . radius . Use ihe resvlt of Prob 7"11 .
! ~ i 2" !, j. a ()
'

[%

(1}~

(o.9)~(t}] ~- [(%)T(1)1][u % ~1)] I T(o.!J"f(4t)(1)


:3. GG.S ~ "' G . s4-S
.
.

o-

"'

-t

.so ( 16 - 11)

;:

1 .7&;

on.

..s in from base

;from ~- 75S

l"

1-'5.) A body consists of o r 1qht cir-cular cone whose ba~e is 12 in .


~ WhoGe olfi i u de ji; 16 in . A hole e in. in d 1"ome.ter ~ + in. deep
been orilled

from the

bo&e . The Ol\iS

of

/he

hole ccimc ide.s w/

oxic: of lhe. cone. . locote the centroid of' the


re.sulj

of

Prob .

7~ .

ne-t

~he

volume . Use fhe

or

the height h
lhe cylinder movnled on the
heft>;Gphe,..;cal bos:e sho""1'I in Fig. P -7-56 so tho} the com~ite
body will be in s foble equ-1libr;um on ils bose . Hinl : /\.s lon9
os the cenler of 9"0"iiy does r>ol lie ob<Ne the 'f. -X plono
thcro will o'J(ist o rostoring couplo whon the body ;s i ;ppcd .

756} Dcfermine

,:1'('7/h(h/1) - \",1l'(l)C-'i_,:(I))
hj .. 2(1)

h 1.+1+
751.)

n-

Repcol Prob. 756 if the cyllndricol portion of lho body


replocod by o l'ighl con1ool port10n wi~h o
rodivs base ~ oll dude h .

in Fig . P - 1.56 is
2

_tp1'
h

J/a(f)/h(h/fi = o/!"1'~(1!~(1))
h'f.s - 4h11 - 1f
h=.3 .+6+ft .

759.) /\ s teel boll iG mo<.mtod on top of o timber c ylindor as


shown in f ig P- 759. Stool woighs -490 lb porcu 0 ~ timber woighs
100 lb per w ft . Oot~inc lhc position of the ccniCf' of' qrovi~y

,.e

WalccJ boll :

[413( (4~X ~,At] (-490. 11/ff')

Wsa. o.76 .02 lb.

Wimber .. r _
(siA-"

w, ..

109.0Q '""

( 7'1.02 + 109.oe)
t()

1!y,9')1 (MiJ\x 1ifl""')(1oc lb/n')


76.~ (ts x V,r) t 109.oe (1t l( ~:z )

185.1 ~ .. 286 . 'f6

~ U5

ff. from bose.

Choptcr

Momenta of lncr"lio

904.) Oe\ermi"6 ti'~ momen~ of ine,...\ia of o triangle. of boee b


~ oltdvde h wi\h re$ped \oon o'llico through ~re opeic porollel
~o the bos9 UQ:. tho tf'oni;fcr fonnvlo ~ ih& n9Gvlk of illu$ .
Prob &OZ.

'IJ":.
/ah

r. " bhY-'45
I .. f"

l\d" t:>h/~ bh/2 (2/3h)

tih~!O

2bh/g

I .. bh8/

SO!J) De.lerd1he the fl10t11enl of


1n F(Q . PSOS with rei.pecl io lhs

_g,..,.,"

f p ' d/\

oxes .

1:

p( Tf'#d/)
(ff,.dP
e -ff]~

J
.J

inedio of ihe guorler circle ~

-tr-+

shown

r~ for &ttl'~cl9'
r4/.+
Vr( - ~.te-)d.,. - r~ [+ - .su11<&/t]:
r-4/e (( -o) {"nu - sino)]
2
I;,
Tr/e

J:

Iy
Iy (S,"f'cos'f>fd8df

J di\

1'" f0T prJp""~fr !,'-,~ (cos~de-)];

~r r~ (CC5 ~~d9-) r -+/+


T

r~ [1'- - ~]o
g

S,: ( 11 ioMe-2 d<r


l

r'Ye [C.1'-o) (GOSaf - GOSo)

eo1.) Show the moment of inertia or o sernicirt;le of radius r


1s 0.11 r w i th respeci to o centroidal 01<1G' pal"l:)llel

to the ch:1~

I ,. " y,.+

I ,. I,. tAd 2
0.3927r4

Tr
-a

I~ + ir,. (&r..\r.

--r-

31(J

f,. (). 2.&2.9"'-r""

I .... 0.109~ r 4

""

0.11 r -4

eoe.) Oeiennine the rna0'16nl of inerho ror 4h6 quoderc;rcle


shown in f.g. P-aos w)lh respeoi too cet1troidol lC-oi<s .

1"-irr+

''
h .. o.1.+1-t-7r+

0.1%31""" ,.

r, - ().0549r+ .. o.os.s r -+
809.) Deter mine tlie moment of inerfo with respect to the lt-o,c1~
for the orea erclosed by the ellipse whose.. equot'1on is . t ~ 1 .

>J> dslermine

t~ rodiix;

r. gyro~ion .
.z!.

l(t

..

. . . ,.

o'-

ct

~~ ~~.:_::

~,.frtdl\ Jy.. 4lldy

..,t

.. i : , ;,dy

f,. : -+ yL( O/t>Jb'-r,) dy


i.

y =bs1n-9dy -= b~~
"+ffob 1 s1rt'&(o/1:1)(b 2 -b,s1n~')'Jz

+fh b3 os1n~c:o&,e-c:le-

" '1:Qb9
ab~

.f" 'Y2 -c=ae.)('h)(


f" (1 - cos(2e-) de-

,. ob'
= ob 9

1,.

(1

f:'t (1 - Y2(1

-t

1 t

R~ l h /;.

s . . e .-. .: ~Th
bcose- de

cos~-e)1 de

co.s+e-)) ~ oba [ e- - e-/e -

["0-h-sn-t0/el~"

= irob/1-

b bsane-

[('/+}(o.b~V"a.11J'k

R)( (b'Ayn

.;n-i&/e];,12

ob9 i?rl+-o)-4itn2l-~n%]

b/2.

a10.) Deiermine the rnomsnl of inertia "., the rodius of grrolioo,w/


respeol to lhe Y Ol<i6 , of the Or\90 C:Vf from the flr-s quadront by
ih6 curve y .. + -'X wher-e 1' 1-. y ore in inches.
'I

+ -ic

-(y-"'1)

~y_.
~..

"lC 2

whent"O

it2 . -lo-+). +

as.:

ll1 dA if dA ydY.
1
l 1 :.(. l<%yd, -r{ll 2)(-t-lc )c:ti<
,. :(4"2_"~"
17 [4/:1 ( ..1)-f(+/a)(r-of - (~ -o)Ys]

ly

,./:il:

)I~!~

11.

Ky .. ~~V(1,/3)

32fe - :12/:J

.. J-4-/.s

- 160-96

w/

011.) Oete,...mine the moment of inerf10


respect to +he X al<S
of the shod&:t porooolo o-eo shotJn in Fig . P - 811.

I, f y 1 dA

dA (a-x)dy ; )(

S:y.,(a - o!Py)or .
i~P>--olt(y )] dr

J, .. [ ov%

1<./ "-,. X/y

y.a/~ly'

- o/b(v%)J: [ 'YJ(11-0)5 -f/11"<~))(

=ofb

1
:-0) ]

1, .. a~;, - ob% .. obXs (s-a) a :zab){,


91~.) Oei4rm1ne the ly for the Ghoded paro0011c area of Fig. P-&11 .
Iy fx'dA
dAyd1-

ly

bWo d)t

:_r b/ro; ..~It d1


~ [ b/,ro .,.111)1.

J:

- (2hXb/..ro) ( a -o) 71" '2./r ( tJ/,ra) (a 7/r)


3
Ty - (2/1){d'b) :Za iy;

"1<:).. + '(>)(1.'!>)

.. S"4 in~
J-1,. +ly
L1h .s+ 270 in~
"',. _lJ/A "~~1%l) 3,97.31 in.
817.) Deiermine. the rTIOl'T\<!t"ll ol inet"ho ,.,., rOO~ or 9yrof10n with
re1:peci too polar~\roiQo\ OXtG of the Cl"OSS S&c\ion oroho)IOW tub6 wn"6 out.Side dlomet<!lr' iG: 6 in . ~ 'inGide d 1ome+e.- i& ....~

ly

&

cJA:.;;...df'
... .,,. :r Jpd/\

-~Ill ~'1'3of

J : tJf'
T

A 1r/+(do2-dt~)

tr'] r-o
f\.

lr/.ot(~- ~) ,
:J

.A1e.111n

Th(~-~) 102.11n.

' K. "'JJ/A

,. ~1021/~71

= 2,55

in .

910.) II hollow square croG$ section CO'lsists &' on 8 in square.


which is subtrQcted o conoenlricolly ploced ~quore 4in by.,. in.

frooi

or

fir'\d ihe polar (TIO'Tle~


ine:dia "" ihe p:h-- radius of gyrot.'on
~pecl to o Z. oi1i: po~ing fhrough on& of fh~ ovl.s;iae. ~
l. I, iAdi

with

1., l 7 +Ad

1.

/\ e-...t - ...e in"l

I., ~s-. 6(e)(._,)' 1;u;s.33 ;.,;4Iy, ~~e)3-. e(e)(...-{ c 1.:36.S. ~ in.+

J,

r .

lyj ... 134;5.33

~13"5.3a ., ~730.6'0

I,.t." ~4)6 + +(-1-/(2.UF 'l7'f.'33 in"t

Ir ~~4) + +(""'~t2) 2 = 277.33 in~

"-. ~
= -.J ~1741,46

I\

J1 '"

lllC t]~

= 277."!3 u11. .33 " .s.5't. ";,,~

J T J,-J2. ..

~730.~c:;-~."'" ~z11<0.1nt

in!

~ 6 ,73

in.

&1g,) ~let-mine the moment of nerlio of the I - 6ect"i01 shOWT1


in f~ . P-82.0 w/ reGpe:;\ to 'it~ cen\r-Qidal o>U .

,.

l\d

.n-11
.. 2_S

~Ay

:y e.(e)(e+) ..

~(e)6)

= ~~.n

'i,. .!!i 1 2(,e)(t.5'f' t}fl)3 2(e)(t..&)s.

1f.

"

l'tt 1.(e)(t) -32,n~

1... 290.r.1 in:4


920) Determine the moment of inertia of the oreo shown 1n
f1g .P 02D with respect to '1\s cen+roidol o~ .
Ar : 1) H2(1) t 12(1) = 3oO in~
/\tr b'\y

30'9 i2(-.)(o.s)

12(1)(1) G(1 )(13.!1)

y = ~ .7 in.

r,. :"()..(1)3. i2C1)(s.2)~ ~


-i112~ t12(-t)(1.g)"
12

~. 6(1)(1.0)1
12

I.,. .. 95.s.;a 'm:"


I,. ~ 1 (12) 3 ~ 12(1)3
12
021.) f ind the

for

12

1(,)a 103 ,n~

12

me.men-I ci' ineriia about the indicated


oroo .sho.....n In F19 . P - 821 .

J< Ollis

the shaded

I,.:'.[,. tM~

-~'

r., e~tt + s(1o) (!1)11 ~666.67 in~

iO

-
022.) F1'nd
..;;howri

1~1 = 26".i;;1 - 11,0.~3

ihe cenfroool mornenl"

= g~.14 in~
iner lia of !he ~rope:e.oid

1n Fig P- 02.
Ar 69+,)( ' / 2 ) c 72 int
3
j,. -[
+ 2 ( '4)(" )(o.s)~ z

";t)

r..(GXo.s)

I,. = 190 in

'1(6f..
12

823.) /\n equi\o\crol ~iong\e has itt; hose b

honxontol. ShoN
tho\ the c.en\r01dol mornen\ cf' inertia with ~pect h rori%00~01 ti.. V&f'\1eol o~ ore equol .
"lit

/b1 - (tY2){ o.~b


i, ,. b(o9'6b)3 = 1.90+l'10- &b+

""

o.~b(tl/~}3 1 .o.Mb (t!fe)3 = 11.801- xiOt b+

12

1il

Compute &e rnomenl of 10e-lia with re~t k>an 01t'1c;


possing \hraugh ~ oppo&ite ope"8G <:: o regulor helllogc:lr\

92.4-.)

a side o.

-& 360/ii

~~ c b/a

to1"130 ~L'Yh

Yff
o

b c:1J'~(p)

- t>1~Yti

h o-B/~
I o[t (o<B/i?)]a
1~

[ 1h~)(o.n,4) ] 4
9

4~

,.

,1
I

Compute the moment <:A W-.edia of fhe 10 in . .by ~ in . rec tangle shovvn ir;i f ig. P-825 obout the X Ol<so to which it is
.Oclined ol on ; on<}\e-B- =sin- .4/-!f. Hi()f : Resolve the figure
it'lo p<>ri-s A. B, ~ C .

825.)

!iS.1T
~

ton.53.13 : 10,.ib

b 7.6 in.

d ~15 -7,5 ~7-:S

j(,.

1o/s1n5"3.1'3 ~ 12.5 n.

11/d
h =cos3'i.87(7.5) "' " in .

C05 36.67. =

Cl)l;

.s3.1:1 : a/10
Q

fxc - HZ.& (4')3

1/tz

= "1i() .

2
(12-5)(6 )(1/5 ') = 228

in~

in.

fl!A

12.5(6)3

~ (12 ..!5)(<;)('

Y9,)2 ~ H7~ in:"-

BG

i .... t lye f.e 24'75 t 22.5 + goo :9600 iri~


~o.) The cross GGCtio<i shown in. Fiq P-926 ,c; lhot of o stroc
i.11

turol member ~n

:z

os a

of l. ". ly .

t..

'

s.,o~ion . Determine the volue.s

~( L. t-J'ld)

'

..t

~/4(4,tf I [3.!(~/.
tll

12

3.!J(il/+)(:z.2.St] ~

I,, +2. fl in~


ly " .lf:(f,. tAd 9 )

=-t.s(""+f [J!4,(ai>f t~+(3.s)(1.'37!:1)


12

]1

tf

iy .. 1..!l."M in:4
/\ g

+.B(~+)+[u(3.4)]~ .. 9 .~2s ,n~

827. ) The built - up section shown in Fig . P-627 is

composed of

two 8XGX1 io. ongleG ri"8ted too 12 x1 in. web plate . Determine
the rnomenl of inertio w1\h rc~pect \o the centroidol )( axic; .
/\1 [e(1) t!l(1i]2H2(1) .. 39 in~

/\1 y .. ~l\y
38Y - 12(1)(40) t[8C1)(11.s) t S(-1)(9.5)] 2

y = 9.!17 in .
I,. s L(f,.

Ad1 )

1(1v t 12(1)(.z.07'f 1[~

~(1)(~.g)-) 2

,.

[1(5)"3 I 1 (ti)(0.47)t] 2

r .. ~ ;m>.<;+ in~
U5ing Tobie Vlll- Z Propertie5 of Strvcturol .section5

f,. - 38.8 1n ; y - Mi.S in. ; A~ 13 il'l .


!,. ,. ~ t 12(1)(2.(J"T)f t [3S.9t 19(1-39)1 ] z

9.c;,.1 :

12

j,.

37~.97

in:+

r()rT(l

w.1 lb c honnelc ore laHi~d togelher to


the GeC'fion Ghown in Fi9. p , ~.20. Oeferrnine hovv for o por-t lhe
c;honnels should be placed so o~ to rnohe
equol to f y forthe Geci10n. ( Hcglo:;l ~he loi l~ce bors which ore 1no;cotc:d by
the doshed linec;.)

s20.) Two 12 in.

r.

--------1~--------

fn::>m Tobie vtll-.2


Prop. of Shvch,.1rol Ar-olysiis
Chonnel Size /\rY!O i,. TY x
12'20.7

J,. ~

j,.

A~

:;

__ J__

&.o.., 12e.1 .s.9 <>.1 o

.~

1,. (128.1)2

---------i----------

ly"fr~Ada

r,, =[.:..91~03(dh o.1tJ ~


r... iy

2
12e.1t11.J '1f3.9 t6.os(cl/2 to.1) ]
d

=7.c:;e in

d ol which the two 3 in . bys in. reciongles shD-Nn In Fig. P- 829 should be spaced so ihcrl f >

929.) Oelerrnine the du>lonce

J,, .

r
~-:.

I~ f,

1 Ad 2 ; [ 6(3)3 + 8('3)(c:l/2 t1s)2] 2

Ir ., 1,. :. [3(8)3 t o]'i.,


~ [e(a)3 + e(aXd/2
i2
12

i,.

.P-

2
t 1.1J)

17

120 " 18+2-1-( dh ~1 !l)


c;1 .. 1:20

in.

hor~ le~s four 6 by +by 1h ifl. ongles ore connected


too web plo+e 2.3'h in bv .5/,6.11"1 . to fonn ihe p\ote &i ong\e girder

030.) The

or

Shawn in Fig. P- 030. Compute i~ volue of'

i. .

y s 12 ;n,

.i. "

5A6 ,2a.s)3 .. [

c;~~-!5t

l.(o.!i)(1t.1s)i] <1-

[o.s(11-&) 5 ; o.a(;i.6)(9 .7!1)~]+


... 1~

I~ 2~7.5.5

in.

U~ing Tobie v111-2 Prvper1ieci of Giructurol sections

6~4" 1h : Areo 4.7.S in:t


; y a gg' ~ ~ , 1.99 in
1
6.3 t41s(11.01) ] 4 + [.5A6(2.3.:s)~]f1~

l . . =6.a in 4

\,_

r~ "[
fie "' 2~G. 3.S
Q31.)

1.,." 11.+ in 4

in ~

A plole 11... ongle column 1<> compoGed

of' four 0 by+ by 1 in.

angles w1 \h the Ghort legs connected to a web p lote 14 in by 1 in


Plvt>

t"' flongs pk>1es

eoch 18

in

by 2

;_. in

ru; .i;hown

in Fig . P - 031.

j,. ~ 1

[1\12s)'

t 1a(u.s)(a.375)t] ~
[~I 8(1~.75)1]+ t[i~i)S I 3(1)("1.'79)9]+

I,

...

1.::&+.65 in.

ly 1 \1)1 +[a.2e(1e)-a}t
[1~23 +1(e)(~.s)1] ~
n
,
I[~ 5(1)(1)t].+

.1
y .301g.ss 111 .

Usng Tobie vu1 -2 Properf1e~ of slrvcturol Sedions


s11+111- : Areo 11 io~ ; 1. 1t.6 1n;J1 ,9,6 in ; ; < a.o.s in

1. [1(-..)'.lA1

1' y , .~ in

i11(G.2)] [(1e(t .i.slV\2 -t 1e (Ms)(e.a1.a)~J 2


1'e1.99 .n.

t [11.6

I,.

ly rH.(.)'],/12 I [,9.<0 t 11 (:MS) 2}+ {r ~.25(10) 3.Y,tj ('2.)


i1 30-21 .1 in~

832.) Ootermi'""' lhe conlroidol moments of inertio of the bi..i1ll up colvrnn section shown in Fig. P- 032. . II is compaseci of fwo 16x1
plot.es riveted lo two 12 in 'J.0.7 lb chcmnel5 .
~

From Tobie Vllt-2 Propertlec:


Strvdurot Geciiooc:

or

rt- :ioTb : /\reo c;.o:ia an t , T 129.1 in

11
_!:IL

ICS

,,o.Tlb

a.9 in....

; y ~o ta. ii 0.1 in

t, . 118.1(t ) t[~~)'3
t
1
j,.

16(1)(6-:J)2] t

161().9 in~

Iy {3.9 r~.oa(+:i)2]2 ...[1 ~~) 1 ] 2


ly 0.56.87 in:"t
033.) FourZ t:>o<-c. eoch hovinq the "':te \.., propedies delermil"'IC!ld in
Prob &26 , ore riveted io o 1~ by 1 in plole lo form ~he aeoiioo
shown in Rg .P-033 . Determine the cef\ ~roidol momenls
inertia .

or

iw ~1[~t:W+(3..S)(1.2S)t] 1

( 1 . r;(/.V 3 , .... ~(lll/+)(s . ~)2] + t [ a.'~(s.')' t

"
o/..(3.S)('l)j

I. a 1297.3~ in:t'
11 " ..lll:!l!.[a.e(.:s/1)3 1 3.~(3/'+)(o. e1..;t"]+
12

1t

[a!t\-+.s)3 s,4(~.e)(s. s)t ] ... 1[ 3 . ~(a1+t


,

1t

+ 3/(a.s)(c;. 1~9) ]+

" i.)I

.s91.6 it"l.<t

to tre iop of o 1+ WF 34
beam Oli sho-Nn on fog. P- 8.EH-. The wios non9e beom hos on <Neroll height of ~ 1n . onorco of10.ooin1 ...,_ lir. ot 339.2 in :" G::>mpvle
034.) I\ 10 in. tJ.3-lb Chonnel iG welded

y I.,

the

moment of inertia obovl Ire cx:ntroidol

7< o~i& .

From Tobie "a1-2 Pn:>perheso of Structural G:c\.


10 15.!1 lb

Area 4 -+7 ot'l"

/Vea

1. ~o'll'l~ ~ 11 2..ain~

J11 ;)39.2 tn ~ !&,. t 0.1+ on.

5i o.64 in 1oi., y o
AT 10+4."47 14A7 in

~ 101n<

A1y :a.Ay
Ho,4.7y 10( 7)t'4+7(13.6)

y 9.o+in .

i,. ..

2..3

-t.n(-t.od) 2 t :139.2 t 10(2.o+)

T11 476.0l in:"

Q3!1.) Two 10in 19.alb chonn<'IS of'e welded toqe1ner os sho-.vn in Fig.

P -8~. Compute ~he voltx'O of !11 for orrongeroonts<D> '6...(b) . Eoch


chonnol web '1<0, 02-4 in . lhicl\ .
1

o.)

r.

Fl"Om Tobie 1/111-2 10 ' -16.3 lb


Al"'l:!O
AT y

+-..71n' ; Jy " 2.31n:"


il O.G4 in . St._ y ()

S:Ay

,. c u .9 ~

4 ,47(2)

'1 c

44-47(5) t +.4T(g.4j)

7. :;J in

2.3 t 4 .... 7(2.~) 2

+ 47(~. 3)~ ~

fir. "

&..Jic .. 66.9 in~

1\6.5 ,n .-4

2 (4 .47)y ,. -..+7 (~) .........7(10-~4-)

r, . "

7, 92
t

11'1 .

1 .+1(2.s~)~

11' ..

t 2. 3 t

.+ .ot7(2.82) ~

1't0.3 in~

ron6fer- rormulo ~ the re~ult of Pr-ob. BGO,deir"Crfio


0 homogenecvs t"ighf circu lar cylinder obout on OlliG ~hroogh onolement on i~s surface .
The Cyl inder hQ(; o moss M '1.. o rodiw; r .

6'7-4.) By using th"

termine "tho momcmt

or

or

I= MK' .. Md%

= Y2Mr1 + M(r)2

I = s,h Mrz

or

063.) By using the tronsfur fon'l'lula 'bi._ the resoult


Prob. B62 9
defermine the moment of inertia of o homogeneru~ sphe~ of'
mo~

M ~ rod'1uG r w1\h re~pect lo a


I ..

Mv. 2

tangent .

Md2

I ~ 7(.s lv'lr

866.) 8y using the tronsfer formulo ~ ~he resvlt of' Prob.663. de-

termine the moment of inedio of o rod -..dh re&pect to on axi"


through the cenler of grovity perper"diculor to the rod .

p7
I

YaML

Md 2

::

M(L/2)2

f = 1ML

11

or

06S:) Sy using the troniofur formula%., the result


Prob.961,determine the moment of" inertia of ihe rec"tongulor porollelepiped
show n ,n Fiq . B-2Q. with ~cpect +o a med&on line otfhe Z face.
Take tho median line porollel to ihe )( Ol'<i s .
T ~K.9 tt.1d 2

= M(b 2 t c 2 )(1At)

M(C/2)2

"1,l.f 2 t.1 (b 2 t c") 11/.,. Mc 2


I '"'
Z.,1<---

--"

1Az M( bi + +c 2 )

or

809.) Determine the morneni of inerlia


ihe rcxtongulor porellelep1!)(:d shown in Fig.P-869 or f 19 s-29 with resp ed to an o:rn; through
ore edge porollol lo ~he Y 01<is .
I c MK2 + Md 2 = 1A2 M(o 2 t c 2 ) t M(ofz.) 2 +(cf:z)--z
" "AZ M(a1 +c 2 ) t M[/(Qh)'

(c/::z)t ]

='\42 M(o~-tc1) + M(o7'+ + c%.)


I ,. % t.11(o 2 +c 2 )

or

812.) Determine the moment of inedio


o hollow steel cylinder
w'1th res~t to its geomefric mciG. The cyhiider- iG 1fl long ~ hoG
on ou+side diorneter of 30 ~on inside diameter of 2 fl s+eel
weighs

4Qo

lb per ft~
l/ 1f(M-1t)(1) 3~Q27_n-a

w ..

--3 1~

IS\,

..wo 1b/ffa (v)

~o lb/_w{ (a.927 f({)


w = 1g2..+.23 lb
I -Y1 M(R t r ") = "Jt(192.U.~) [(1.!>) 2 t(1)'l)

1ft.

'32-~

; I .. 91.11 ftlb-sec
By using the method discus~d in Prob. 070, detern;l1ne tha
rnoment of ir>edio, w1 \h ~Sped to the 9eornefr1c Qxi G. Ol O cylin der of rodiuio R from which is drilled o concentria hole of radiug r . Denote the mass of the resulting hollow cylir'lder by M,
"-. the mo~ per unit volume by 'Y.
973,)

the ctlnder
fue hole
I ;": 'Al MRt

M.,'f,y

'v't\ m1 for

R<
r

tvt~ ~v
M '6lfh(R2 -1 ~)

V21 rn2

fOr-

v, ~ vR'h -rn1 >ilt'i:l..~h


2-lfr'Zh- ~ , 'lltrlh

2
-m2r ]

:(': Yt[m1R
;= 1/2 [1SR1 h(~)- ~n-~h(r~)1
! = ~ ~JTh [R4 -r]
, =1h glfh(R1 -r 2 )(R1 +r3 )

. ""/t M(R2 +rt)


A slender ro:1 6 n long rotates about on O)(iS perpendicul
to it ot o p01nt 2ft . from one.end . The rodweighs 40\b. Carnput~
the rnorn.snt of 'inerlia obout I-he oxi ...
rolo\ion .

974.)

tt'..f--:s
'-'
... ::::J

or

I = 1A'-z ML2
~ "42(~i.t)(6') +f+-0/.;2.i)()t
::r ~ +.gj fl -lb-.sec 2
87S.) f()(' 0 hollow cost-iron sphere or W in avtside . cliometer
~ 16 in- inside diameter, compute th<' moment of inerlio ~ the
rodlu~ or gyrohon w'1th re~ect to 0 d1'o meter. eo... t 'iron
W'4Clb .

weighG" 450 lb per cu. fl .


Given" Oo = 2c>'':. R=-10 in .
0 "16 r .. ein

w "' 4~0 lb/ft 3

\/ 4"% [10-a:_ 9ll] ~ 1.193 fl3


(t1)'

4.SO_lbj( (t.183 fi')

.. .532. 3.5 lb

.... ~ ./Vtil
-

/C~~.""'t-..(1_++_)"""(..-32.,,...,-=-ct>)
-'3~-'35

.r

K. ""7.35

in.

876.) l)e~ermine

the momen1 of ir'lerfio or the COE>t -iron flywheel


Sh<>vvn in F1Q . P-07S with resP<'ct to th: o,.i~ of ~tolion. The f1yw~I
ho~ Gi,. ellipticol spoke<;, 3x-+ In. 1n cross secti0n which moy be CDns1dereo as sler>OOr roe.I.. O:lE>t iron 10!ghs 400 lb per cu.fl.

+I
_
_,,_

~,-..,

w,
Wr.

~T(R'-r')

=~T(~!- r

L ; rim

) l ; hub

w, "~TobL; for one spol<..e~


Wr(+so)11 (w'-2s 2 ) 12 : 1138.Balb.
17:Z9

Ws : (4.90)lf(1.5)(:Z)(23)
1726

=.$6.4S lb .

r,. a Y:z M(R' +r")

~(11.38.e3/32.2)[30t2a2)A++1

lr " 20G.0 ft-lb-sec 2


In Y-z M(R<t +r')

- Y:z(131.+4/32.z)[(s t+2tY14'I]
In : o.+290 .fl-lb-.sec~
Is n(i ~ Md 2 )
n ( 1/.2 ML'+ Md 1 )

... E{Y.2(..sGAS,h2.:z)(uh)t +(56.4~M.)(-t6.sA 2) ]


ls " 2.3.11 fl lb-sec"
I: Ir
c.

lh +Is

206.e + 0.4290 + 23.11

l ""230.*

fl - lb-sect

Choptc:;r fO

Rcctilineor Tronslotion

..

0.1 G\ cerloin .s;trdch of trock, !r01n.s run oi 60mph.


for bock of o stopped irain .should a warning iorpodo bo plocod
fo "ignol ori on coming froin ? /\ssurno lhdt tho brokos oro ap2
plied of once ~ rotord ihc. lroin ol ~ho uniform role; of 2fl p:Jr soc,
Givon:
Vo = 60 x s2so ,.. esfl/soc
Vo" 60mph
2os

100.2.)

.
yr'-vo'

:96<)()

O -2fl/SOG"
Q

-et/ " -2(2)S


s " 1936 fl or

o.3667

mi .

1ooa) /\ stono is thrown v.erlicolly upword ~ returns fo oorlh 1n


10sa; . Who{ wos i~s iniliol voloc1ly \._how high d1'd if go?

y(vo ~at
-Vo

-9.Q1 (10) .. 99.1 m/s

or

- vo -

-:M17+(10) ::i21.7+ fl/s

s =-Y29t~ .. ~(9.e1)(1ot .. 490 ..5 m .

or

S "" Y2_gt. "~ ~(32.174)(10) 2 "- 160E17fl.

so n h'9h of lho
some instoni that o socond boil , s thrown upward fr-orn tho grov

1oo+.) /\ boll is dropped From thv top of o tower

with on ini~1a1 vcloc'ily of 40 fl/soc. Vlhon ~Whore cb Hioy poss,


'!.... wi\h whol rolotivo veloc1ly ?

~vo.!+J!igt -10t - ~(32.2)i 2


80- h .. .141 (3!2.2) t ~

h,.. 80-i&.1l 1
h

--

-@

sub6fifu4c. 2 lo 1

ao-~. '40t - v~~

t = :z sec.
h 90 -16 .1 (2) 2 " .15.6 from H10
s' = ao- 1s.G - ~....... from the top

bottom

Vf, - Vot = ol
Vf:z 40 - 3!2.2 (2) "' - 2+-4 fl/soc;
Vf, 3~. Q (Q)

64.+

ft/soc

to05) A sW1CJ s droppod ~ o wol1 t..., Ssac iofar lho G0"1d


ar ~ ~ -.. hcol"d- 1r 1t1G "IOloc.ity or SO.id K: tim n ~
goc

whof ~ tho depth

lho""4?

l,.c J!OmG
\l!s ~ tltlOfl/.-=

t. - ~ b- lllOslono io6' ClfllOd


~ -t,

1,iilne

''"

tlbc;.$OIJll1d iobo t-cordj(iJJ-~ " d

far fhG.stGno d-= ~gt."


d ~(321)1.t!
.91hdiiub t lo ~
tHIO (.!t-1") = 16.'I t.,.2
..5600 -11mt,

<:'l(st

d a tt20(.s- t,)--<9

--

.s;;: ta

By~lic fq- 'JO' gJf t~ = -t-4945 GCJC..


9
d== t~1 (1.QH5) - .3.5331 fl 1006) Rcpool. Pn:ib . too!i if lho sound of

tho spbsh iG t1ooord oner

..ax;,

tor- fho &IOUlld


d

lG"'

-mo(<t-~) .- i@

fho ei<J<lG
O "' Y21Jt..,11 .. ,-2(a~) 'tit! - @

.fiUl>Sfifufe 1 lo~{
1120(+-t..)'' c ,16-1 ~ 2
+t80 -1t20~

1b-1 t. t!

Py ~ tq.t;= -b :qr-i,-.t..-_-.fGC-
.'
~o
d"' 11~.1 (s.793) 2 == 231 . ' 3 0.

'f0'6 g:;I

t,

= a793 SGC-

1007.) A sb1o dropped limn o Qltlf'n-G bodoon of on elevol;on of


1!(XlO

ft _ ~ ~ds

"MJlr'l(.il

"""
3

per sac;,

lob- of'OliJo-sbic i~ ~oofcd ..,.crf;c;oHy upfhe ~ tnd w~lh .,dccity of 2'+e fl por ...cc. If 9 is 32
-hen ~ ~ wilM the 6tonGs pas& eaoO olhcr :'

~t! = 2160/scc
h -11~f1..

'IOQ'.J- h . ~ (~)tt!!

tr

_9 = a2 fl/sec'"'

l: - ttlilr:.c; fuir- Hllitc 11sl sbic


l-2 - ~ fur fhc ~""' eb'O

1ooo - -t6t~

.:.... .. -@

tr2~(i-2) - Ytt(<nli( l - 2):%

h= 2"6l- -t9G -

1G(_:% t6't-l - G+

--

~t'i fvie 1 l!o 2

'lcool-.M>~=

2+6l

15'0()

-~ - ~ t "4-f - G+

= 312 t

-t ~soc-

.. h= ~-..-(s)2. == JO ft.

:\10oe.) /\.

stone is thrown vorlicolly upword from the ground w ith

velocity of +S. 3
por sec. Ono socorid later onothor .stono '19
thrown vorticolly 1>pword wilh o velocity of 96.6 fl porsoo . How
f o<' obovo .frOrn tho ground Wi II st()"IO bo o t ff'l(j some lovol ?
0

Yo,

+s.3 fl/soc

vo.. 96.6 fl/~oc

sutic;I.

t lime for tho 1si ...,fono


t -\ t;rno fCf- lho- 2nd ,;fono

.+G:at

112. 7 - SO,!! t

t -1,-t sec; .

for tho 1&t stono

h" +9.3 (1.4)

h" <+a:3t - Y2(32.2) t'

h =49.3t - 16.1\. 2 - --dl


for' fhc 2nd slone
1
h 96.f(t.-1) - 16.1(t-N
2
h" 96.G t -966 16.1l t 32.:Z t h .. 120.et -1,.1 l'2 - 112.1 - -
I''

lo 11

-,.!J>-.i-r2 .~ 129 ,Q l:. - ~ - 112.7


- 16.1 (Vi)~

h = ::16.06+ n.

1E.1

1009.) A boll 'ii; shDf vcdicolly , n~o tho 0;0 ot o vo locity of 19.3 . ~
fl per ccc . After 4f- sec, oriothor- boll iG .shol vort1.oa lly ,nto tho 01<'.
Whof lnif 1.ol volooi) rnusl tho !2nd boll hovo in ordor io moot tho
flrot bo II .::1a6: <1-

fl from tho

ground ['

Vo, ~9.3.~

fl/sec
h .386.4 fl.

~1mo

for

~he

seGOr"ld ba ll

1sl boll

394 Yo (9.-Ki+ - 4) -161 ( 9.464 -4)

t - 4 " Limo JOI" !ho 2nd boll


" Vot - \129\'2

Yo .,, 1.se. 6 9

ff/soc.

.S

396-+ 193.2 l - 16-1 e


By ~uodl"Otlc E~- t "'9.-+G4SCC ~
t ~ 2.!:i.36.sec

1010.) A sto11C
c;ity

or

thrown vcrticolly up fro,.,, -lho ground w il h

'1s

300 fl/-.oa

.stone from the top of

coch olhor zoo


')-si

n from

600 n

the lop

'6~ono

.S" vol t

. How long mus~ . ono

or the

lower .7

2nc1 Gtono

Y'2gl <

"'fOO soot - 1&1l

By ~uodrotic l " 17.11lS Goe.


'&.'-

vclo -

woH bo~orc dropping a soc.


tower ,f the; two ~iones arc to poss

200-= Xi@11.~)(17.1f~s - t') .,


2
200: 161 [(11.1ee) - 2(17.1a8l')
'lOO= +7S6 . 36 - S.S3.45i

- t''21
116.1t' 2

o -= .:ss&..38 -553.4.St ' 116.H.'!i

1ot1.) / , .ship boin9 launched GlidoG down the woys w'1lh o cons-font
ocoeAOf"Ofiol"\ . ~ tokes Q soc to ..slido tho first foot . How long wi11
shv to\<o to slide clown the YIOljlO , ,f thC.r long~h '1s '12,S fl ~

-s

6fl.9 f'I

.S 1 ft when t : S 6CC

.s i'l ~t 2 Y'l a t
1 c Yz a (e)i.Z
GI 0.031!1S

t :zoo.sec

or .3min, ~O&oc. .

rl/.sro

'/.: . .
.
1l 1013.) fvi automobile slorhng from rest spoec:ls lJP to

"lO fl persa:.
w1\h () constant occelcratlon of -4 ft por sec}, rvnS ot \h'1s Gpcod fq0 +;me,"'- finally comes to rest w'1\h o dcceloroti on of ~n por u:,c,~
If the total disionco tro.-olod is 10Xl fl , find the to\ol lime reqd.

Sol'n :

Givon: Vo :0

a= 4ft/sc
a~l'l/s
~d:

a-tfV1:

Vf -Vc>

d =~ooon .
total Time
!

40

o - sM~

al1
"'t1 - t1 '10.s:=

s. Y11. (,.X10)" -

2a:>H .

"' Y,. St/te - Se "'tO ti


Yf-Yo-ota
-.+o ~ - s ts -1.s = 6sec

/ s~= .otO(e)- Y2(a)(&)4

=1,on.

Sit S" tS3 "1000

v"'40M

~f~-;~

Vf o: ..ioft/s;

!ZOO+Sc + 14o~ooo ~ s2=c;>n


b"'IO =-t-ota --t.2: 1GGeO

1t ct, tt.. +i, c 10~16+8 ~ .3+soc .


1
101+.) A lroin trovelc- botwoon two stotions /2 rnile oport in o
rri1nirnum timo o( 41 GOC . lf the f roin ocee lol"otos Y.._ d oc-ol.o ratos ot
8 fl por t;OC11., slorl from roGi al tho 1st ~otion II<.,. c oming too .stop
al tro and -sicif io11 , who! i& Hs moi<i!"urn -.spoed ;,... mph? How long
doc:iG 1t \rovol ol this top spcocl ?

1
6iven: d '/emile o&fi/G Sain : Yf -Vtot1 ~Yf 8t1
tl 41

soc

Yf

Re.<:j<:l : rno;c . .!Opcod in mti


"-.. distonce trovel ot
thlG top speed

~fa

f;11/te -

-Yf "-at&
2

G3 = Yfia- YlZ l&)ts


S1 +s,. Sa o.!l m'rle.
264!0

~ t vrte +Vft1 - ~

~4<> ..

Vf ( t 1 th)

t 1 t~ f ts : 4 1

t1th " t1-h


2G;+0

"Yf ( 41 - !;3)

~6'to vf (41:-f-)
2"40 - ~

v)..'

8y ~uadralic fo""'u\a you g o\

's<... vf = BS H/s
tJso or <3ot 88 fl/,
9fl ft {s " acooo 'x mlle
I~
-i:;- .s2eofi

'f,. 2'40 fl/g

60

mph .

vr -n
B

An outomobile. rno-iinq ot o constant velocity of 45 n por


.i;ec paGGos o gaooltne -station . Two soc.onds l~ter., onothor oufomobile leoves the goGOline station ~ occo\orotes at fhe oonsfonf
rofo of 6 por soc~. How -soon will the .second outomobile over-

1&16.)

take tho first

Given : V1

+.s fi/~

o'" c:;H/st
~e.q'd :

t;mo

Q:iln :

V= d/.t - +st =d -
d~ Y2 .o (t -:z) 2 -

4!'t 'V':z(~)(t 0 - +t.t +)

1st
19t
0

t -

t..z-.,.1; -r+

+-+
19t ++ O ~ ~uodrot ic
=

-1 &.79 soc

t'= 16.19 -2 " 10.79

sea .

The motion of o pariiole jg g ivon b y the equation .s .. ~t+ - t a


.
co
2
t 2t whore s is in foot ~ t inGOOOndG . Compute. tho values of v ~ o

1019.)

when t

'" QGOO.

t 1c;ec
Re;;il::l :

V~Q

-(~.)
r

V = 8(2) - 3

.st!_ = 0 - ~+-i:.i; - t l
db
2

a 2-t(e)

2
-

+ 4 ('2) "' 70 f"f/G

.. 4

~ t + " ~8 ft/stZ

1~0~ A partiolo mo-vas; <n a stroi9h+ h'nc ac.cordin9 to the lbw


3
s=t -"f<>t where 1hcs ic;; in0!J.....tinsccond6 .(o~ Whon t=6$0C.S,

oompu~o fhv volroty .(b ) find fhc avo. veloo'ty during the +th
G'Oconds . (p) 'W'rut the padiole agoIn comes to rest, what is 'its acc:cloration ?
Rcq'd : a ~ ve-1. b.) ovo. vel. '5<...
c1 occcte rehon .

colh: .di =v ,.
dl

3lt -

-+<>

-a (s ) 1 -+o .. 3.S ft/s

b .)

c.)

.Jtl.. ,. 0 "' 6t " 6(.?..6s1)"' :21.91 ff/,!;oc ~


dt

v " :" V - ~o
O ~::.ta--1<'.l
}

..+():3t~

sec

= 3. 6.S1

10~2.) Che.ct<. tho anGwor-s to fllus. Prob- 1018 by t he followin.,3 moihod: Wrr~e on c"prcssion rolotinq 'I. ,.,.__ y. ,.,._ by ouccossivc
d ;fferonfiation 'how that va= 'XY.-/Jhaty.>J \... Oe - 'XOl'./-Jh'~-t>J t
h 2 YA/~ . Compute va 'i<... oe from these relations.
Ton~

"'-Vo/vA

since Ve is downworo

Vs ... -v,.. tanexll +h 11 .::: z.!i. i L=~

h-y@

di fferonf1a~e i
o ~ Y:.z (x<l'th 2 ) -'1l1(12x d x) - ~

~="a

j dV0

1
...

at

'ljr+n"

oe

dt.

.. v....

dt

dl

dY = Va
o~

Ye : "l.V11. ( x"'th"rY
dVa = Oe
olt

=~ Y/\(ic.<l1h 2 t"2 + )( cJVt> (x2 th 2YY2


dt

y~ xVA.(xi+h<l.) 1/:($.1-)

Git

dt.

Qe : v~ a

-t )( OA

~ {)(+~2.

as-=-~+

~-tn~

- x 'l VA 'J.

(.[x-1 h2)3
htVA 2
~(xZ+h'}~

Vs 3 [9(10~,.f9'-t11t~ "

OB

G"ri,4

[9(4o/J~~+~ +f1e 11 (1ot~~9%if2 2)a

c:

(;.t;1

n/~ 2

1ota.) The roc;tilin_oor IT'Dtion of o particlo Is g1'Yon bys v -si


whero S j.S in fC,ct ~ Y in foe f- por GCCOnd. 'v\.11cn t 0, S 0 ~ v '3

ft per sc.o Find

the

.s -t, y - t , \..., o - t

Givon" S-= V~-9


t~o;s~o;vca

~q~: s-t,v-t,~a-t

r el()ti or.S.

Gol'n . s +9~v"" - v =~" ~


dt
~.,. dt ~ BGe<:.,...e~ e-

-l6

~ 9t9ton~

1
'

.S= afon~

di; ..

a60C.~-e

oe-

scce-

Y._S-+9
-511

e .1: .. ~.-~.-j)-=-1t~-

-{:t<--- 9)11

t _v_11
___...,_

Tho yolocify of o portidG ~'19 ol0t1t'.j lho x oiiis is c:bby'f=Jts- ,._,.... GX where vis an fcof porsccond "-)( i.s in
fed- Canpu-fo fbo YDlu.cof thooooo~ ~ lt=~fl 61...m : v.,,,x-a--n...nx
s,1;.: 'l ...1~ - 4f.q6(1)
'Jl=L
'1 = ...R/lieO

fO:U.:)

faix.d

Rotfd :

>ldV = odx:

= Vd'#AJJt

dYd~ "'3'J.11 -81(. Hi

d'fd"I -=~

a-.

8 ft/ 6eelr
102.5:) 1hti ~ of o porlicio is dGti1J11Gd by thG rolatiion
where o Is n ft per le2 "-.. f. "1 .socondG. U is ~ -lllhot s ..1ff
i.,. v-= i'f'U" .soc ~ t -1 GCC- l>GlcnrinG ihc reDotioos bclwcc;n
0-= .+C2 ) =

""'-L si..t 1" \....4>fiWcn :

o~.-.t

S f

V-

t-1

Reqa : v- t, i;-t, v-s

sotn : d't'.fcn..= l
v = ~ tC ~ .d11 f'C.
. 2

c=o; 1f

t.~1

'f.e :Zt .. -t : ~

do/dt = dz

.s ..

~l 3 -tC

a-

= 2(1) 0 -t c

=2
c '

+ ac

1/a

..s ""

2t3

+ i ....
.a

.35.., 12till-t 1

3S = ~(%->3 t 11
::ss - x(rv~ ,,. 1
~-IT

35

~~

t1 ' .

(3s-1)

"' "'~Ji

10f6.) Tho mOtion of o particle~ governed

by tho c;quoticn o -~ .

whcro o is in foot per .soc Ii-, s la in foot Whon t 1 .se.c. s +fl lk.._
2

v" ~ft por ecc. P o temiino 1ho rolotion botwcon v ~t, sft..,t, v'-s .
6ive-n : a - 6/siz
v "';{ dG

rs <tt""

t;1,G+,Vc<t
~eq'd

: v - t , s-t, v-!; ,

4dt %ds

solh :

ads "lldV
vdV -% 2 ds

-tt 2s"l<l

v/2/ !l.'

1
-as-1

tC

v 1' s - 1 tc

,f S t 'ff.... V2

.: C "0

y ..

16/s

s -

1GA

tC

+ ... o/3 (+)ate. ~c


c = -+/;!S

"f't ~/3 .S a;.r


112t

t (-

4./3)

11.!t - 26?,i"<t _

1? (i ~e)3f2 - "f

112t =

128 - 4

yr

ico lnO
find tt-iO

1027.) The rnot;on of _o pariiclo 'u;; givon by o Gv'l.r; where Q

pcr&cc.r 'vi~ in fl por s;oc. When t Is


rclo1 roos be.tween v ~ t . s '*. t, v ~ s.
G1V&l"l :

d%t

o 6vYci

to, g .. (;, V O

~ 9t 2
c

9~ 8 t C

s-= ~t-a ..

d%l 6v'/<

d'0;.cv .. dt

l;"-6 ~ v O.

ifsGl....t3-co .. C '" 6

Roqo : v-t,s- t,v- s


Soln:

%Cr0,

G =

3t3 +6

,'

dV(v)Ys 6dl /
2vVe 6t t y
'
t . 0 "'v. 0 . c "'o
2
2v" 6t

s , ~(.J%) 13
.5 .. ~y~~
~

(;

( 9)

y'llt -.3t

v" 9lt
t = ~h

"t

o parf1olc iG 9ovor-ncd by fho relation o "+t,


0 ~~ in f'l/s 2
'1s in GGC. When t i~ zero, V 2 ft/B ....... S + foof .
find the voluc10 of v s.._s whon t = 2 sec.
G.Von : a +i L
2 .. ~ t c; ._ C" 12. V .. +ta t ~
3
3
3
t o, v 2,s .. +
v +h (<Z) H "' 1~.{;7 N/s
Reqa : v ir...s when t .. ~ dS/dt c+t,% t 2
s . "ff ~f t ~t t4
&>1'1 1 d%t +t<
.s ,. +t.}4~ +~t t c ~ = .,..(2)/'f iz t 2(e) ++
V ... +t%+c -,ftOls..,S ctjC= 4
S""13.33ft .
1028.) Tlic motion of

where

'toif.j From the vt curvo in Fig. P-1033, deter.mine the distonco


trovelcd in 4 sec\..., olso in 6scc. Also skekh the o l ~ s-i
.cur-.tero opprox1rnolely to ..scale.
if~')

V 'Y1(t 4)
Yl

- 1C(t 6()

S2. - 10L/L

s.
. t C~ )

'

60t tClt

t+

6, <IO

"f-0 ~ -S(.+) ~ t 60(.+) ! C%

Ci -1:to

ol 'f SoC

.G2

-st 2 t 60l-t2.0

G Yt(+)(to) 41<>n
ol 6SU
~ . %1(,)(10) c

60{1.

n
"
'TI
v. - ~t

Si .st:,4 t C1
G, 0 "". t O :.c, O

St .Sl /t
103+,)

The molion of o porfic.le stortinq 'from rost i~ governod


f1q. P-1034. .Sl<Oich thG v-t ~ st

by the o-l curve shoYm in

o..rvc.cl. Do1crminu the du;plocemont at

t g G".

a. 3

o(ff/r,)

+/:J ( t -o }
O. . +,(,l tfO

Ct ~

60

'

Y2 = - ll/H~ t wt o/.s (g) t t ~o{.o )

vz - ~n/s
Ot :it
V1. 2/H" t <11

iFV1O Jl..to .'. Ci 0


V1 t f.
36 ft/~

"t

a. =- ~13t 'J.O

-60

A.o

~ ~r~'L_ ; ~-
-~
,., ~

_ _.___

I
I

.. . t
I

~+c"~~---'~~...:...'t~)

v, - - +/,t' t !lot tU.

v,. t

if v.... - v. .. ~ !!.., t 6
3 6 ~ - +/, (,)1. t W{f.) t (4

s, t!/s tC1; lt s,o 1fc. t o :. ei -o


!;1 ..

f.

t~3 = (61/!J .. 12.

n.

Yt

t~t?t-l>O

- 1/.at2

&1. = -2/gt 3 + 2ly'12t - Mt

n. - -.

C2

Cr.

t.

~
6
2/9 () 3 t 10 ( " / -

SL-G, - 72

M>f..~) t C~

t2.0

SL :.-o/0 t t t 10t"- b0t t 120


~ " - 1(g (9) 3 f 10 (9)~ - '(") t 120

Gt. 2,:2.6 fl
104{).) /\n object attains o vc.loc-1ly of 16 fl pot" g;c. by movi0t3
in a s trolgh\ l\ne with On OCGClero\iOO 'f'hiGh vOOGG vniifonnly
from zero to 8 f\ pol' GCG2 it'I 6SCC- Compufo i\; '101\ial 'IClocity ~

the chango In dlc;;placomcnt dunnq the 6 GCG intCN"OI - Solve


0y uGln9 mohon c uN<A; "-., chcc.._ by calculus; -

,Af; ..

Af;

= 16 .... t

=6
2

16 = 6/iti. {f,) t C
c = -8
v= &/12tt. - 0

if

..
I

ii,., i=O

\I \lo

~ -9('- o) t(8l~)fll](~-')
=: <>

'

v"!.i2t" tC.

v(t2t.) t Aorco..-., - t,.

- 0fl/i;
'\<M1. Tho oc.cto ~ In 6 !;CG
Vo

'

or on ob'jed
ot w/v

dcm:-.oGC& unformly fl't)l'll 90 ~ .s}

time a, velocity 110 10 fl per~- RJ nl(; in-

itiol vc.lro-ty ti..._ lhc. chango in d:sploc.ornc.nt d~ng th; 6 ~


info.rvol . fulvc by ~!f109 mo\ ion CU"-'CG " .clicd. by calcolw; _
0

.. eJ6(t-6)

8
t

o - - 4f.H t e
if

V-Vo

llJ....,

cO

Vo. ,.._fl!s
A f; .. V(t.c- t ,)

'/,. - 2/$t~ t

if

0t I (j

v-10 to_t 6

thorc:;fore , C - 1~

~"-o)
6S 1Zf1 -

+ lv'Oa ll...-t h
t 8k (6)(1:/a ~)

1o+-4-.) /\n clcvot()('" wcighinq 3220 lb storls from res1 ~ ocquire.s


an upward veloli1ty of 600 rt per min in o d istance of QOf'l. ff 1ho
occelcrohon i~ oonslont. Whof i(; lhc. \en51on in ~he o lcvofor roble?

soln:

QI

W;3220

(609'60) 2

v ~ 600 fl/min
5 wfl.
Rc.q(:l:

T- W

"'

12a(eo)

a 12.s fl/s4
~ w1g (a)

T - .3:Z:ZO i: .!>2::U)~2.!2 (. :z.a)

T "' .3+10 lb
1~) /\ mo" wcighi"g 1~1 lb is in on ctovotor 'rnoving upward w/
ro occolorotiOn of 80 ~.sec . l_a)Whot prossuro docs hooicerl
on tho floor o( H'G olovalor ?{P) Whol wilt iho prossuro bo ,r \ho
clovolor ''" de..sconding w'1lh 1hc sornc accclorotiOf'l?
Gi-..on:

.SOlh:

w..~ 161 lb
o Bfl/.s~

o) 'T- w

Reqo : a ) proGSuro ho ol<Crl


b~ prO!;sure 1 f"elevoior

is dcsccndin~f w/ lhe .somo

""/g (a)

-=

1"Y~.'2 (a)
T :zo1 lb.
b-) T - w - - %(0)
T -%1 " -(1~.ra~.v(0)
T-

ocooloroiion

1~1

T "" 121 lb.

rooohoG a vobOly of 40 n per GOO in


1000 ,.siorilnq fr0mrc.Gf. Conipu le tro ooofficient of'~1'notlo frioi1on
bclweon tne blooK "'i., the ground .

'I04i8) Tho bloci<. in fig .

P-104';

Given:
V 4oft/soc;

~
P DOlb

Soln:
7

G 100 fl.

Rcqti : C.OO!f1Ci'1ent

of t1.in&t1c

frioi1on

,,/>3%

' F 'l"'tl
ye ~ '2as - -10 ~ = 2 0(100)
a~ s ff/6 i
p - f " IJ1GI - (,0 - 161(f') "-1'~2.2(8)

;t .. o.fH:n
Delorrri1no . thoforce P thi:l~ will give 1ho body P, fig- P - 1047 on
aocolcrof1on
6'fi porooc.~. The coofF0'1oni of Nrioi1'c frio/ion 1so2.
G;ven : o >< c:.n/i;e t-0.2 sol'n : f(~ - Py)o,2

1047.)

RcqCI :

or

forec P

.c (322 - %PJ o.~


+feP -(~~ -3..<isP)o.t. "'~2%.2.~ (b)

+/!!>P - 6+.-f- t

O~fo P =

<;O

P-= 1~,2z lb .

/I magnetic porticlc v-.o.ighinq a.c; grams Ii; pu llod through o


.solol"Oid wifu an acoc\eraf1on
G rnoiOl'G \)Cf' Groe. Compu-to tho fur'O'
in pour-de aolln<3 on tho podiclc. Nole: 1 lb45t9romc; \...1in. 2,6tetn

10<ffl .)

or

6iv0fl :

W .:s"9
a

7:92%;ir. 10-3

l>

~.!IG.~ irlj5 - 19,,9 (ifs<

6m/s4

Goin :
F ~

Rcqo : Forco

7.'M9.Sll10 "(19,(>9)

.3!2.~

F o.oo+es lb
c

1051,) T'(Y'o blookc A ~B arc rolcosod

frow rvet on o ao inc.lino w


thoy oc-ol!Klfl op::irf Thccoof\loicnt of fr-iclion under the uppor 1:>100<.ti
'1s 0.2 9' that ul'ldcr the lower bloek.. B iG O'I- . Compute iho olopscd
\ime unlil tho bloc~ ~oud'l .

GrV.en :

-f}- 30

)l,.. o. 'l.

>,.__.f-a -; o. +

Roq'd : Time to e lapsocl unt'1 I


+hob~ \oLJch
.solLJtion : 8Gin ~ - 0.4BC.OS30" "'
as ~ <f-,% fi/s Q.

%2.'l. as

,Asinao - o-'l A ca.PJtJ f./a~e a~


QA

10,.5~ n;~ll

' ::S - ~ Clet 2 e ~(41.gs)ill


.f
~18 Yz a,..t,ll - ~ (10.!i<lill)
.50 t 2. n~ t 2 .!5.2"5 t

t'

t 4.23 o!>Cc

1os2.) Deter-mine 1fhc. occcbrot1on

tho fi'i<cd drum '1s'.smooth

f1

''

of the bodios fn Fig. p - 1o.sz

,r

I\ Is heavier than 13 .
.sub$t. '2 lo 1
W>. - We -Wll("A~ ) =WA ("'/g)
w"- T;w"'(a/g) -
(WA-We)g - a
1-we We(a/g)-@
T "'We t Wa (~/,)

WA+We

"'

1053) Rof'emng to Fig. P- 1~. assume /\ woighs 1'1ex> lb ~a wei9hs

-too lb. Pei - +he accclcn'.)tior"I of the bodie.s ,r the c.ooffioiont of ~notic
f r1ction ic 0.10 bo1. the CG1blo k ihc fli<od drum.
W"~OOlb ) 0.10
liifr~ efe- 0 .,(l)
we " 100 Ill
1~/re c
..so1n :
T,4 = 1.~110 - - @
2Cl0 - TA = 200;G.a(a)

- <9

Te - 100 - 1oofa.e.12(0~ - @

.subetiiufo .:i lo 1
~00 - 1 ,:~qle .. 2oq.3Q.!Z

from

2,

J~

= 100 t

a - +

1oohr.>.2 a

Gub6fduto. a Io
~oo - 1.37(100 t .3.11 o) ~ 6.21 a

a r G.21 Q
~;~~
>
1o.s+.) Two bodies /\ ~ 13 In Fig. P- 10s+ ore separated b y a i;;pn'ng .
lhc,lr motion clown tho '1nolino is rcsiGiod by a
f'' ~lb- The
cooff1c1cnt
kinetic friction ic; 0.::10 under/\~ 0.10 under 6 . Oo rlOO - 131 -

4. ~lb1

0 io. 012

rorco

or

tcrrn'inc the

force In '\he Gpr'1n<3.

G,:..on:
P2<X>lb

1" o.ao
Se 0.10
0 L1\ , -100 %(-+00) -

4h(lO)(o.3)ts :4<%ua

- s'u; :012.4-20

oi 8, 't3/e(6CO) - s +1~(600)(0.1) ~ 600fii2.eC1

-s ~ 18,,!Jq ~
from 1, s -= 12.+za 'f .s6
.312

,subGirlutv

toe I'

31!Z. - 12.4~0 - ,sf; " 10GZIGI


Q.SIO

= ~1-05 Cl

a = e. Q-t- f1/.9 2
s 12.412 (8.24) t .56 = 1.58 .+ lb
1os1.) 1hc coofficiont or \'-Jnoho frdio11 uroor block./\ in fig. ,P-1os7
'1s o.-so ~under blo'*'- S ii; 0:20. f1ncl ~ho occolcroliori of tho syslom
c

'..... '\he

lcns'1on it\ covh chord .

o\_c, 300 - 12 = act'ht>.<l

Cl -

11~ .f>, 1.z -T1 - l!OO.g1nao-

(}.(JJCl)8.i3()

<!)
0

12or'~12 Cl
1~z-l1 - 1a+.,4 '-1210 -

(0.!J.J

""

ai /I, l 1- 1ros1nao -100Q:l6eo(o.s) 1~.<.Z a

1.,
11

-7898: 3 .110 -
:7.s.9s~~.11a

.SUbSI. 11 to@,, 1~-(r.!.99 ~~.1101)- 131" '+: ~.121 Q


12 -210.6~ :g,aea - ..

T~ 21a'2 I

9.:3~ o

Gubs.tiluio 1o 1
300 - 10.,e 1 9.,g~a = .300~, a a

a9.:3G = 1a6+a
a .. +.a nh~

T1 "

75,96 t 3,11 ( _..e)

go.g1

lb.

112 - ~10.,1 i 9 .3~ (4.&) : ~"'5.3,10 .

Chopter 11

ClX'Vilinec:r TronGloiion

l \,
\ ;I

stone is thrown from o hill ol on ongle of 60' to the


hor-i:tanlal wi\h an inif1ol velocity of 100f\ per.sec . Afior hiHlng
level ground at tho base of the h ill \he s lono hos oovorod a
hori:tanfol clistpnvo of .500 rt. How high i" fho h ill?
1102.) ;\

.sofn :
'f,.eVo~t

.900:; 100 c.o.s 60 t

~ 1060C

y = Vo G1n&t. - Y2 gt. 'l


-'-'--"-=-3 00
- "'"r'"'"
f ---a. . !l" "
~ '\COSf1~o"C10') - Y2(3~.z)(1o)!l

,
1

Y 743.97
\

fl . or

1103.) /\ shell leaves a mortar

w/ o

743.g1

fl .

muzzle v eloc'i!y of

.sooff/oc

d;rc.c,-lcd upword ol 60 w~fh the ho";.:z'.O,,+ol. Dclcrni1no tho pos:4.1'on


of tho shell 'ifr; resvUorrl volooty Wsoc af1or f)r'inq. Hovv hi.gh
will '1f r so?
.sot'n : x a Yo cose-i
X = .!iOO c.os Eia' ( :20)
J( = 5000 fl .
y ~vosil'l6t - ~gt 2

V.,;.soo f't/s
-& =6()
t20SOC

'{ ~.soos1nM" (20) - Y!<(a2.i)(20Y


_

Y-Yo

"gl

= e~~o . ~..!S .fl .

- v.,, -.soocos60 o - v.,, esort/s.

. /I

Vy -.soosncoo -3:2.2 C10)


Vy

h = V~s1n~~
29

= 210, 99 fl/s .

.soefln 2 6()

..

2911.5

ft

2.(.31.Z.!l)

110-t.) A projoctilc is fired "Niih on iniiiol vclocily of 19.3.2 fl/SOC


UfM'Ord ot o n ong1e

or 30.

to the hori::z0nt_o I from o. po"1nt 25 ' 6

ft .

above a lovcl plain. Whot hor1l:ontol di1>tonco will ,1 covor bofcro


,t Girlcos iho lovol plain f'
~ivon:
Solh : R = ~vovox~-e-(+on~ t Ian-&)
,g Cbs p
vo 193.1.Z fl/s
-e-aao

toflp ,,,.syx

Co6 !

R_ :js,IZt )<2

x/ rz

-< = !l (1Qa.2)2 Cos <! ao


J~tx~
32.~( )(
--)

~.s:.6

ton 30

{<';P<6'f-i ><"

~ 1003-r'(

,
)(

11

n.

1338.s::i
- z.57.6 c 193.~ S1n3o't - !lz (3.e) ! 2
- 2 5 7., ~ 96.6 t - 1~.H 11 t ~ 6.soc.
J(

/er/

+ 44791 4_q1)<~

- 1ooe.9>< - -q."'1-7.914,9

;. =Vo oose-t = 19a~ c.oGao(s)

1105) Rcpoot Prob. 11o+

if the

1338.53 fl .

project'ilc ,s !1'"d downward al .so

to tho hori :r.onto 1-

Solo : y = Voi:;ine t

oo

257. 6

~57,,

257.6 - ~"''t

"
'---

V11 Si II

=103,2 sin sot


t

16 .1

~ (3:t.-2) l ~

ay -tXuoclroi\o fOl"mulo
t ~

)(=Vocose-l

f;OC.

19~.cos.:30"(.)

x ~ 334.63 fl .
1106.). A projociilo is firod w1~h on ,nitiol volocity of \b ft.por
sec.. upward at on angle of -& -...:dh tho horiwntol . Find tho
hori:wntal dlstonco c.ovcrcd before the projectile returns to
,1c; cnginal level. Also dcicrrninc the m())\1mum height otioin~ by tl'"e projectile .

(ic11:,n)

h e vos1n1H -

,' .. -1--..
./

I
(o,o)

Y9tr

Voal59-t 1

)<

-..... o t(x,O) j h =0

L~qtl
Yo .sine- c V2gt ---+- t.
x "'\locosa-(zvos1ne-) - v0 es,11 2eo1 (Xie , n) ; t l.f~
g
9
.
1'1<t.

h~

'

1'-/~

vo9T"t6

[!1..V~~;O(> - Yt.~~vo;nerJ

h ,. Vo 2 s 1n-&-

- Vo.is1n~

29

h = _VoL sin
'f.9

0-YoG1ng.,._

(x,o)

e-

2Vo sin~

g .

The <Pf"' shOwn In f:g. P-1107 Is jud to ckor


fillod gop Find the to\<.o -off velocity Vo .

1107.)

./"

tno

wotor-

h ~ Vo61n&t - Y~gl'-zf,2

~vosnJSO't - Y~(;;stt.-.flH~--

~ - Vo

17-3

oosaot

=Yocos&>~t

2<l/vo

.sub6f i tu to t'. fo 1 ,

%.Gm30(2o/:i4S') -1,.1(~oh0 )e

- u.1l -

t .
~t.~

Yo

6440

~'\,,<i.
Yo " 14114- ft/60C.
11~~ /\ boll is thrO'M' so thot 'it.just c\oan; a 1of1 fc;rico Ei"Ofl
o'woy . lf il loft tho han.d -'fl oboVo the ground """of 0 origle
60' fo tho hori%on1ol, what was the iriiiiol veloe1ty of the ball?

or

Given: h 'qofl.

~ 60ff, -19- z(J()' /

r -~ff -

. ffoqC:t: Vo.
solh: x

60

.
Vo oos&l

__%: -

I'
'

------- ---~ "',

.!!, '

I
1

Yo GOSG,0 4 t

t 1~0/v6
h Yo ~M-6-t - Y~ gt~
10-.!3 - Vo &1nb1>(12"/y0 )

ga.g v/

1,,1.(it0,.Y0 )"'

~ ~51&10

Yo = 4e.4 ff/soc.

w/

11og.) Determine the distance s a1 whioh o boll thrown


o
1
velocity V? of' 11'.X) ft. par soa ot on onqlc {}-, ton- :J/+ will strike
ihc 1ridlnc ShoV"l'l

In rig . p - 1109 .

,.'jh --, ',

~e-><--

y ,

"
'

-e-

tan - ::s/+

-f7 = '313, a1

Rcq'cl : distonco s

.&olh:

t~-t . - ts.%,t' .. ~,1,t -16.1 t~

-y .. vo&n&l - 1/tgt.
~

Y/x

:sy 1( -
")( Voc.os &t.

s.51 Goe.

= ~9 . ff.
y a.g.s(G.61) ,. 14.gs fl .

')C

77. 8' (.S.81)

- y .. 100 sin 3e.&1 t. - 1~-1 t l1

-y ~<.7G l . - 1~1t 2 -
'f.. "' 100 c.a1.3&-&7 t
11 .. 'n 86 t - @
.s .. J(4t9)" -t(14t. ~o) 2
1"-..~ - tiy. n&H i y 25.9.St - @
s +.s2.< n .
1110-) In f{g . P-1109, o ball thrown d~r'\ tho lnolino stn~s ,tat
0 di&f()'l~ .9 ~.s-f--.S fl ff the ball rlGCb io 0 rooximurn hc1qh\
h - 6<f,4ff . obcNe the pcitnt of rclcoso, conpute 'its 1n iliol veloc:dy
Yo"' inolinaiion-&.
Given : h '4-.+f1.
&;

P.octd \

.sn.

~t.s+.

Vo ..,_

Sain : Sin~ " tt/s


~ "' o/f.S+.s - tf - go.s ft .
cas ~ :: x/s

:ako x4,e -

'lC "'

24'1 . ~4

fl.

- 80.s .,. VoS1,,&t -!'9l 2 - -

t-+1-+1 " Vo GGllO&t - - @

subGtitutc .3 It.., 1 1

-eo.s =64- -+t


0

-1~.1t~

t - .s 6C.C
:Z-4-1-+t = 'lo COS& (s)
Vo cos&- "IS .f.00
Yo61n~ = 1ari&
Vo c.os8

" "++/-+8t68

Yo i;1n.sa.1::17 b4'-4 -

1111.) Rofor to fi9 .P- 1111 ""-

find 0(. to

Vo "'ga,s

--& c,s3,131'

N/s

coug~ the projectile to h'd

point B In o'l'OClly 4 sec . 'Nhol is he dic:tonoe.

:11 ( _

1111Z.) Boot /\ moves with o con6tont volocity of w

fl pcrsoc, .storthe po$dion shown in Flg. P-11H . f ind -& in order for
the projectile to hil the boot ..s sec ofter .s'\ortinq. under ho
condition qiven. How h'1gh i<:: the hill obo"e tl)e woter?
h~

from

' f,6fl/~e

Q Vt -100 200;)
~ ~

t--111~.)

100 t 301 .,, -1-01

- . . ., .............
~~ Ye Co.s'9-t c 401 ~ 96600& &(6)
a::nstont '-,
.. \ ~y
220fl/5iG ',, ,
'
-e- - 3.3). ~ '
"- _
-: .. ~ h Vo sin &t - 'f'L (;:,2.<1. t
1 .... '"" . - h
.G1naa.ea (.s)301

"1 (5)~

9'"

f1 .
It is desil"Cd to pitc>i o golf boil aoross a trop to o grocn
owoy. Wf'lat is the best club to use ii the initial velocity
h~

- 13.3.25

1oo f1
~he ball t" Go ft per sec.' Assume tho! tho boll stops dcod after .strikin9 tho grocn, which iG on tho.some label o.s thi> poin
fi-om which tho boll r<0 struck . Assume the c lubs hove .slopo.s
9roduaicd ot inter-vols of 6 .so thot o No.1 iron hos a foce ,n chncd o.l oo to Jhd' ground, the No. 2 iron ot 7+ , clc ., doY'(Y"l too

or

Ho.!1 iron inclined ot


@ivcn -. Vo 60 ft/s' '

.32 .

.Solh :

rt;

-x " 100
RcqCI ; Ho. of Iron

'1' = vo"s1n2&
g

-e- .. 31.72

Use No. 9

il"'Of\

111+) /\ stono has on initial velooi~y of 100 fl por.sc.c up to the


right at .ao w rth ihe horizontal . The components of' occolcrof
Of"O C{)r\Sfont al Oll - 4 fl per sec' ~ Oy " - 20 f1 Per"SOOt. Com -

puto

the

hor .171:.ontol c;l\Gtoncc covered until the -stone rcochc&:

o point 6 0 fl bclCJv.< 'r\G or'1qi nol elovotion .


.
(.?ivcn: Vo 100 fl /G
-e- ~ ao , O;oc - "1- (Tfs!I
x -vocos~t t Y~att
.
oy =- - ~o ft/s~, y ~ 6 0fl .
'f. ; 1oocosao(6') t Vz(-4)(~)!1
'

Rcqa : horimnt91 di.stance


:Sblh : y .. Vosrne-t - 'hgt 2
- W 1CXhsm.30t - 'h. ( 20)t !I

~to .sot - 10t e


By Qu0dr-otlc you get the
voluc of'

~ 6

sec .

1. .. 4-47.bi

n.

111.s.) /\ pod'aclc hoco such

a curv; linear rnohen that ih; .x coor-

dinate iG defined by x =.st.a - 1ost where x '1c; in inches 'It,.._ t In.se cond~ When l 2 Gcc, the ioto l occclcrohon Is 75 In per sec ~.
lf t ho y component of' accclcrotion '1s constant \..._ the particle
.starts from rest at the or'1gin when t ~o, dotcrrri1no the toiql
velocity when t - 4 sCC.

6ivcn : x ~-st-a-10.st
$0111 : x =.st3 - 1o.s t
Yx : 1si~-10.S
t e:z.soc, a - 7.S in/s ~
t~o."'-.

t=+soc -

t - 2. ;

0)(

30t

CIJC

::30 (<t) ::. 60

Roq'd : V

a =Jo,. 4 +ay,,;
Oy "' 1'5

t 41-(;cc.;

v~

"'7:?1

12

= ~o,ztOy,z

in/sec ,z

=1st. ~-1os

v-,. = ts(+)~ -1 0.s 1asin/G


Vy " Qyt ~ 45(-4) = 180 in/G

V = v.,. i t Vy'"
v = 4f3S<Zt1&o,z
V "' ~25

i'n/s oc .

1110.) The normal accolorotion of o porticlc on tho rim ofo pulley

in diameter '1s constant ot B<XX> n pcr.scc 2. Octcrm'1nc the.speed of the pulley i'n rpm .
Given : d =1oft
On = 8000 O/siZ
ReqCI : S pccx;I in rprn .
&:,1n :

1oft

On ... :::C..
r

- aooo = X:
s

v :zooft/G
V

lfOH

200 =1(1o)N
H ., 6-+ rps (60)

.382 rprn

At the boHom of' a loop the: .speod of O'loirplonc is 4-00 mph


This causes a nOrYTIOI occclcrotion of g ~ 11 pcr$X2 . Determrno the
radius of the loop.
.so1r. : a(') = v1./r
Gi_;en: V e 4KO rnph
.9(:32.1.)r- /4oox.s2SO.>< 1/ s6oo1il.
On =9gfl/G1.

1119,)

Roc(d: Radlvs

r -

1187.64

rt.

A particle movo6 on o

1120.)

"its arc dl~tonco

+P -1ot where ~

of 20 f1 radius c;o that


from o fi;iod po'1nt on ihc path i 9iven by .s ..
,~ in tho foot 1,.-, t in ~dG. Cbmputc -tho total
c if"COlor poth

or

occelcrafion at tho end


2soc.
61~on : r2on
Solh: d .. 12tQ-10 .. v
.S

~(:j

(W V "' 12 (~) 2 -10 :98 fl/GOC

+t 9 -1ot

t eGcc: 0

_5ff_ .,_ 24t .. at


dt Of. " H(tt.) .. 40 fl/s:z

an Vo/r (30))/w

an .. 72.~ fl/s 2
a 0t<ant

.. (46) 2 t (n.. 2)

86. 7

fi/Gect

wz1.) A porf ide . is movinq olonq a curvod path. f\i a oortain


instont whon tbc .slope
the poih Is o.1s. a,. =6 ft per sect ~ay
10 A per ~oo<. Compute Hie vo luos of o~ ..._Ori oi this instant ~
St<.Ofoh how tho pa~h curves.
61'v en: Ox = 6 fi/6t.
Soln : On .. a.,eose- - a,. sin-&

or

Oy 1ofl/5~
slopo - o.7.s
. :

tr - t an-1 o."J.S 3 6:l.87 0

Req'd :

Ot ~On

On ~ 1ocos3~.e7
Qn ,.

0"~a 11 ~+ay'1 ~<;,110-

a ..

'

-+4 fl/G

- c; sn a6.1n

=11."zrl/s

alai On2

(11,c;~2)

at2

t (+.+)t.

at ::. 10.5 ft/s"


A stono is thrown with on initial velocity of 100 rt f>C"60C .
1

1122.)

upward o.t w to the horiwntal. Compuic the radius of cur-.o~ vre


of its Poth al the poini where 'it is soft horiwntally from 1t' iniHol posilion .
Given : Yo; 1oofi/G
Vy - 100-Sln60 -31M~(1 )

!-60. x .son.
Req'.d : t<od1u~ ~f
OJr..ot'vrc
b'olh : Oy : 9 e az.~ ft/s~
J\ : Vo COS,8

..SO

100 COS Go

t 1.scc.
Vw. -100 COS60 0

v.,. .:So ft/s

Y!J : .Sh4 fi/s


v"' ~soc +~-t.)l ..

73. eg

ft/s .

ton&"' Vy/v'J( " 5+,4/.so 1.088


On e Oy Cose - a,. Si~
On 3~.zQ:>s+1.41 "'121,79

an vYr
r -(13.e9)X1.19

r - 2.so.66 fl .

fl/st

on initiol velocity of ~oofl per .liOC up to


1ho ri9ht ot o s lopo of +to 3 . The c.onponont~ of occelerotion
oro constont a\ a,. -1~ fi P.or-scc""" Oy '" -:zo ft pc:r:--soc~ Compute the rooius of C1.>Nature of fhc storl" ~ ol the top of fho
t1P.) /\stone I 1os

7*"T

path .
61vcn : l/o q,<X}fl/s
oi..

" .- ,_"1! ~
~
~,
/
~ \

- - 1;i.fl/~;", O,.. -ao f1/s'

rn -+/s
Reqo : rodiu9 of wrvuivro

-soin :

H vo sin ,11 (UXJ) l+l~c

la_}''

rI~
t;...O fl

"

~ (20)

'20y

H Yo .sin ~l - Ya.t>1P W40 ~(tjs) t - ~(eo)t f


t't - 1ol~ - iot'" - 1'ot t , ... o~o
t - 1,t t64 - o - (t-&)(t - &)co
.. t8~ .

'"

v... 200(~/.s)
v~ 9.4 ft/'

- 1~()

Vy wo(-4-/ri) -to(8)

Yy O ; V- 2Jtft/,

On Oy (X)!;fr - a,. s1w

ao (/)GO -

H!GlnO

o,, !lofV.s c
011

vy,.. - r

24.Y~o

('~& . &fl .

Ms.) /\ porticlo moves on o circlo in ()GOOt"'donco w'1th tho oquotion s t - et whcro sis tho difiplocemcnt in foot moo~urod
olon<_3 the ciroulor path ~ t i~ in .seconds. Two soa:iods of
for Giarli1"i3 frorn f'OGt the totol occelorotion
tho parfiolo is
48 JT
per ooc' . Compute tho rodiuG of lhv circle .
Given : s - t+- et , l f ~ .

or

a "48BH/s~
ReqC! : rodo"uc; of HIO
C1i-clo

ai1h:

s-t-at

Vt "tl.ll-6

al

1~ ~ *

Ol = 1(at "' 461Ys


11

a'~at tan 12

(%~)" (+6) tan<


2

Ori 48

fl/6<

11117.)

f;olvo Illus. Prob. 11~ . uG1ng the

fl per soc ;

ff : dota :

W .. 100 lb i 'I = f:...ll3

L 'IS in .

6ivon : L ~ 18 in ~ 1.3 fl
w - 1001b ,

"e.oa l'l/s

.so1n=

- v"/gr-

iOl"l8'

Sin&

v.i1gLs1nb-

CO!P&

.Gu1~ "'V'2C.OS&/_gL
4
1
1-(.()5 & -V CQS-&-

=0

.,9L
C,c!. ~e

Q)6e.t e.~~C06& - 1
,3!l.'l

-o

~ " CO.ft l!T


11

- 1

JJL

--Sy~uodrolc.

o
cos&- = 0..5948

l1.!i)

W .. Tee&~

r ,, Ls1n&

-<>-

~ .$7. 1'.7 .

100 100SS7.67

T =187 lb
= 1.s (.sin &7. ,7)

t :zr J 1/9ton&

"

1 2'7s ff

2rj MZ~~1l tansH?'

o. 9<13 sec

.+O. lonq.' rotates in o horizoniol plo1rio obo1Jt o vorliwl


oitic throuqh .,., cc.ntor Id oooh end of HlO rod i6 f OGtenod O
oord .3 fl. lon9 . Eoch' cord suppodc o wai9ht W . Cornputo tho 6 Pocd of rotof i()(\ n , j(I rprt'I to 1noline &>Ch coro at .90' w1'fh the
t1!XI.) /\ rod

vcrticol .
Givon : l .. 3f1 .

-e- = 30
Re.q(j : .Gpood

Sotn :
.Sin ;w' 'f./3
)( " 1.5

r" 1.s u

ft .
~3. s

ft .

Tof\& " v L/9r


tofl 3() m v i'3e.!Z. (3.s)
Y a.01

y - 2Trn

O/s
2T (.3,s)

n "'

= 807 (60)
rz2

rpm .

wig.) A weigh~ c.onc.ontroicd al ~he otid of o card fol"rn o con;col pc<>d1>lum to" which the porlod ic: 1 &CC- Doicrni1nc tho
vclooty Y of the weight if the cor'Cl r otates ~nc linod ai 3ow/

the vertical .
6ivon . t , 1 sec -

ton& v~r
ton.:io' ~ vy32.2(0:4-,1)

iT ~30

Re.qa : veioc.i ty
Golh :

t =-

V 11

2vJ f/6tanfd

1 21f

6.7 S6.Z1

..

Y " ~. gG

ft/s

"" 0.901

m/s

J%.2 +o11-ao'

r "' 0.471

n.

11.ao.) In fl9 P-1130, the ~O-lb ball Is fcrccd to irotote orcund the
St"'\C)()ih il"\Side ..{;vrfocc of a con1rol .shell ai lho role of ono rovolu11
h'On in l/+ 60C . Asi;urn\~ th:rl ~ = 4!l ft/Gcx 1 find fhc fons'1on ;n tho
cord Yi!..., ihc foroo on tho conical Gholl . At ""ho+ . ,speed in rpm w, 11

tho forco

Ot'I

tho ~holl become i::oro?


Givo11 : w-2olb
t cll/-tscc, j 9=::12fi/s~
Recid : tcnGion ~ fo<'CO

--- - --I --soln : r- Lsin e-

" -t sin.30~ =

S..... .spc.c,d 11'-(hon Iii 0

2n

N S0.~8 lb
T "' 2o t o.s(s!M9) - .57. .3 lb.

n 1 reY,;.-, -tit rps

0.86G

" .. !?lf'n =- 2ir(2)(4/t)


V~

for r1 :.o,. o. s,c; T - ft-~f{< 2 o

1G{\/s;

,.l!fv -o

T a.3.1 lb.

Tcos'30 - rtsinso-w
~F11

&0-

o.sT

~ s QV /2(a~)

o.s(2a.1) ,,.

T61nso t !1GOG30-wv/9 r
...... o.Bl;GT - Ostf.: 20

o--

~. 1

f.:t.2- o.ST .-o.06't1

" 20(16)/3g(R)

from 1, T (eo t o.s rt)/o.g'"


Q.lb&titu to to 2
a

o.&(~O t0.611) + 0.96b ti " 80

o.e"
11.si; t o. 299 rt t o,e" ti

'"

so

0.3125 Y It

ya 6.079

ft /s

v :1'111rn
6.079 ['211'(~)nlJ'o

n 29 rprtl .
c

of ""eigh\ W rosls on the smooth 1nclinod sUl"foce of the fromo shown in Fi9. P-1131 _ A poq o Hoched to

f131.) /\ OOdy

ihe fromo forces the body ~o rototc w/ , obout tho vcdlcol a xis-. Oolerm1no the speed in rpm o f which iho tension in tho
cot'd 'u; oquol o tho weigh~ of ~ho bGdy.
~l'v ~o; T~n:sc1 r1sin ~o

r=c.o.s3d(5)

~f14 o; Tca.s30 - rte.or.Go


.

.
.
wsmao t r1s1n60 .. w

T:w

=- W

.. wv!:. (!)
gr

11 ""' w-o.sw .. o ..s11+ w


0866

subGtiMo to 1 .

'Ifcos3D' - o,57744ylcos6o
v 8.97..2 f1/s

V-= ~lfrn "' 8972

0
.,

)lt"vY32.~ (s) cosao

2Tr (~C<lS3o') P/c;o

n .. t9.79rpm

20rpm .

Ot 01'\ impact t~ing moohino wei9hs t:+Alb.


1n f<q P -1132 , 11 i.s attached to the oncl of o l~ht rod
whidl rs pNotod +o o horizontal 011 is ot A . (o) What Is tho

t13~) The hammer

Iv> showri
.._ 0 lo1""9

t:mr<riq ('()O(;iioti Of?' ~h(; piOt on in.s1onl after oo:nq roleosod from
the 91~ p001iiori :'~~ What is tho beorif'lq roocti()(l j u.Gt boforo imp:wt ot e ii tho volooiiy of tho hammer is then ..s.9 fl por6CG?
,t

6iv~ :

'

Wa(;+,+lb
l.4fl .

Tcosiso

Roq(:I : (a) lt():)ofion

le ... .. ' al.so al (bl


,._

Sein : a ) Tc.asao w
(;-t.'t

T 7+.36 lb R
b~ T w twvYgr
, .... -1- +
(.s.9):J/.g1.Z.!Z(-4)

r,. . .

T ,. 81,8 lb ... R .
1133.) . To choc~ fho rod;us of

o roi lrood cvrve, ~ho offocl of a go


lb wo19M is observed to be M.7 on o .sprin9 .svolo svspondod from
t~ l'OOf of on 0><pcrimen1ol car round;n9 the cu.....,c e1t "!Omph .
What i.s tho rod1us of the curve f'
Givori : v. 4-0 11117h - 68.(;"
1 " zo.11b ; w ~010

R9qa : radius
&oin :

z'Fy=O
Tcas~ "' w

n;,

CO&(')-: ~0/gM

-o- M .9"l~
~F,. =o i T~on& " W1T~r .. ~o.1s1n 1 .ot.q't3
[~('4-0x88/'"o) ~/a:i.~r = Qo,7.sm1 .Q1-3'
r .. 400,s ff .

t1:SS.) Whot counforwciglit W will mointoin tho Corl1s!. onqi


governol"' ;n the pos;ition show in F19. P-1135 at o rolionol spood
n-120 rpm . Eoch plyboll we'1ghs 16.1 lb . NG91ec-t ihc wciqht or
lho olhor link.s .

.SOl'n : s1nl!O'
"5i1

wy'
,gr

d~~"

dAo -

-~ - o.s'-

'V'21rn2r(o.e~
\fG.28 fl/s

.1,.1 2&
.?12.2

o.s

.39 .~lb

~-o
(i>)A8.S1n7c. 0

30.-tS.s1nc;o'(10) -16 .1 &ing06~

-~
/\e - +.S.1 lb.
ALJAa l\8G0$4'1a t l\e<:OiO - W

[El

'tS1 co.s+5' (2)

-w

w - 63..781 lb.

of tho onqine in fig . P-:11:% is SA lonq ""-woighs


1oolb. lhc cronl<.S /\0 ,._,J~C aro of longhi r.,.1s in. "'rot<:Ato oiaoo
rpm. Oetonn1i'le tlie moxlmum bonding momem M In ihe rod if M
Wl/S, where V{ jg tho total di~tr1D..itod lood 1t-, L iG the lenghf
1136.) Tho side rod

or

the rod .

r~")j1===+c~:;:;::i~
,...... _,,,./#

'

_,~

6iven : W iOO lb

n aaoorpm
r-.1ain 1....,L~efl

&o\n:
MgWL .. ~00(8)]/8

~~ : M
M" 1oolb-ft.
1137~ The .503monl of rood poa;s~ ovor the cre9f of o hill ic: 0efinod
tl)' tho poratx>lic cu~ y '" 18- - ~ /\car ~i~ a200 lb *"'"'so . 1oog tho road ot o consfont -peed of 30 fl per~. Whal i' the prc;.ssuro on tho whoolc; of the ror when H iG at tho crcsi of tho hill whG'C y ""'fl ? Ai the whof Gpeo::I will lho rood e~ro I: zero?
/finl : the. rodius of ovrvolvro by Y,P (d't/cfx~V[1 t(d~"t]_.,
Givcri : r+ft. ; V ac ff/&
X c~O

Wc"32~01b

Reqa : pressure R

'/ ... - .fl_ ~ 4Ao ~


-lo

100

L(!tb)(o

1. - ~00
!{o ~ Q;'1ao -- f'" sofL
ll+ wvygr =w
Golh ! -t ~ - L
10 100
R :a!.IM -[3~20 (.aa)iJ/32-2 (.!Jo)
x --w>< t .+oo - o. (~-2o)(x-fU)) .tJ
R M~Olb.
~VI'

le

'.l.Ol"O

&

if R-o

j(;~ - ~

-.. y'l gr ; ::i2.~ (so) "1610

_gr

40.12

ft/s
0 nat

A boy running 0 foof rocc rounds


Cl.JT'VO of .sort. rod.
1fho runs al the role. of1s rnph, of what 009le Vil the voriicol will
he inolino h'is bocly ?
G.Ven : f'.sofl
Soln : tan& ~v%r =l_1s)(.s2&>,h6oa]/::12.~(1Jo)

1141-)

v 15 mph
fon& = c::1o 3006
RC9l:i : -tr~ =1,.13
11'41Z.) /\ daredevil drives o rnotorcyolc around
c1ivulor vo,..fiool woll 100 (\In oiamofoi:- Tho f bet . tire~~ wall Is M;O. Whoi 19
tho m"1n"1murn .Gpcc.d that will pryvont pl<; Glid lnq down +ho wall ?
At what anqto 'ill ihc motor cyolc bo inohnod to tho hori~ontolr

What f~ tho offeot of irovclir1<3 at o greaior speed?


G 1~CI! . d 100ff .
Goin!
f o.6 ,
-e- .. tan-1 0.0 =a 1
RoqCI: Vm"1n ~-&
tan-& - g r/v 2 - 32.~(so) .. o., v'
v-= .s1.0 fl/s ".s1.& [.=1c:oo/s2eOJ c 35.a mph .
11-4a.)

The

~uperelcvotion

or a roilrood frock is the number cl'

inchos; ihot tho Ot'.lt-'1de roi I (Grx:used to prevcr>t eido ihrost on tho
whc.cl flongCG of c;Qrs; round1'n9 +he curve at rote .9pcod. Dcforrri1no
iho 6upor olo"at1on c f(y- o traol<.. having a qouqo o{' 4 ft 8 Y-2 ,r, .of'
2000
radius S..,o rote .speed of 60 mph . Whot iG tho f1an9c proGGUrc P on tho whoolG of o 100,000 lb. cor that rounds the curve

n.

ot so rnph.7

Given : r = ~ooo fl.

Soln : ton fT

...

v 60mph .

2
v 2/gr - [60 x s2WaGOOI
32, a ( '2000 )

iontr = o.1Q02
-tfl 8.S ii') .sG.9 iii
-0- ~ G.S,
Reqa : ihc svpcrolcvotion
e. for o truck.
fon"fT ~eh
0.1~0~ - ej.s~.s

.
e=6,79in
11+1') An oirplano make o turn In o hori.zontol plono w"1thout
s"1do!.lip ot 4aD mph . At whot ongl<? rnusi the plane bo banked iI'thc
rad;uG of the turn is 1 m'1\c,? If-tho pi lot weighs 1-00 lb, whai prossurc dOGG ho oxort on his

-scot f'

Given Wp 1.!SO lb

Sotn

tot'& ...,.~,.

v 480mph
im;le
prcssuro

fteoxMIJ0/3600]"
3!l,2 (S!180)

r~

Req~ :

.:!J.':L : ~. 92(111<>) .ot~ b .

ton& fl.92

~r

i'T 71 07

N ~ 1soi +4!!S l

.,. 4b2.g7 lb

WIS.) I\ oor Y"C1gh;n9 3!120 lb rO\.lnOS 0 OU,.......O of 200 ff rodr'us banllod of on angle or" find 11-o frlc/ion fon;;c oofing on tho tires
w hen the car i' frovo\ling oi 60 mph. lhc cooffioicnt of fri(;ft(xl
bolwocn the f ir'C'' ihc rood i6 0 .90.
0 iven : W3220 tb
Solh:
2
r ~oon ; ~ .. 30
~ .. 31120[60 x 68fao] 3872 lb
32.~

.9r

/A

" 60 mph
o. '30
ReqC::I : fnC.~ion Forco

( 200)

tonf -o.g

9. ~

..Sin1 f /R. ~Sin'4-2 .. f /.s"5.95


f "3368.37 lb.
bonk'.inq fOf' a hi9hwoy curve
rodius de&i9nod io aOXlmOdotc co~ fravoling of 100 mph,

or aoon.

u..._) Nnd the angle of

W fhe

ooo!lio1.oit of friction between tho t ire6 "' the rood 16 o.,o.


Wrot IG the rat Gd ' p<r,d of fho curvo f
G1vcn :

r300ft. ..0.60
v 100 ,,,ph .
RcqCJ : rofod ~pood

Boin :
ton9S ~

o.,

:ao.9c
ton (1 te-) V~ r

Ion (!!><>9' t ~) Foo"$.leo~oaJ


a2.~ (.~)

:3Q.0' t

'f>'

"-5. e2

-& a.+ .e"

tone= 'lr~r

fon34U'c Yr~12,!Z(300}
'Ir e. .!!1.1.2(.300) +on H -.&>

v,.. .., B~.o~+fl/s


Vr e~.~"l' x 60,,io M mph

Tho roted speed of u hi9hwoy curvo of ~oo rodius is


30 mph. If the cooffici ont of fricf ion bof~c.on t ho iires ~ tho
rood :,G o.w. whot Is tno maximum spocd at wh'1oh o C<:lrcon

1147-)

round tho curve wi ~hout ~k.i'ddin13 '!


Given :

r~2a>fl . p..06

.soin :

to,,-e- vo/9r

Vr40mph
Rcq~ :

ton-&

maximum ~

[:30.x~~ao/aaj/3~.i'(2o0)

ton 6' " 0.301


-(} = 1,.73

fofl

1' o.,

~ = ao.gG

fofl(~ t&)

: vo/9r

fan ( ao.q~ t 1''~') v02.2 ('.loo)


Y EM.11 fl/s
V 8+.11 ( '10/eo)

.s1. 35

mph .

11~) The c.oofTicicnt or ff1cti0n

bo't . t~ r0CJJ ""' the tires of the


err &hown in fi'9 P - 11-4-e is o~o . Tho oor wofrjis 3~~o lb . Ji Is rou rdir\g the CUl""IO of .SOOrt rod1vs at ma11imum spood . What i.& tho
"oluo of the fr1chof' f~reo octin1.3 undcP coch whool ~ how higli
ot:vio tho rood must, tho ccnton of qravity be to limi~ fhi' mo 'lltinurn $pe by t~ei tOndcr.icy to ovortu"n .'
GtVen ~
rc.sooft

.'

+61(19.46 'lflSO(ot."1-1'1) t 5799,48


ax.3tJ(3) - 3220 &mao (g)

w-a1u1olb

111 fe1,~- lb

I"- "0 6

F1 fAN1 o.c;(49G3) : ~977.g lb


at the oufe,... .,.,heel<;

RcqCJ : Frnot10n ~

row hlB'h

~e ~ 0 1 rlL(+.6333) ~ 3~*' <:.os; 30"(2.+n)

6'1n:

t ~799 <f86!030(217) t 32'10 51t'HJ0(3)

ion 4o.6

- ,s7g9.+s cos aols)

t/J =.30.Q6.

ion(~ f~) v~/~r


ton (ao.96 ~ ao) Y2/s~.~(.soo)
&

v - 110/1.9 fl I .s

~r

3Q~0(170.,eq) "'G799.4'81b
3!.l.Q

(sea)

~..,,.c

N 1 (.t.e333)e3q2c~a</(!Z.4'11)

72"-4' lb
fz. "f Hz. (1fl 4'J 435,96 lb
11~

''

at fhc oufCf' wtioolG

ta" .sc.gG" !.l.41'"'Vh


h. 4,0~ 78 ft .

,r tho rood

11"49.) Repeaf Pf'Qb. 11..a


OG 'hown ;,, f{g. P- 11+&.
6 ivCY1 ~ j/. o.,

is lbonkd of oo 1nfood of30

f'.SOOf1

'h

w - a.i20

f<cq(j : fffld;Qrl
f;olh '.

ton 4 o.6

q. 30.06

to11('-t~) v'"/9r
toll (309" t 20) ::. V~2~(.so0)
vf.. 19asa ....g n~,

..

wv/9r ~1120(19&59.49)
3e,t(s60)

a!H0.7 fb

~M,,D/ N1 (+.Q33.3) "' "'~~0~20(~,411) t a010.7.s1020(2.11;)


t a070,7 az20(3) - e1~~0 ~1n:to (g)
ti~ - 362+.,1 lb
F1 Q.6 (382-t.61) Z2g4.& fb GI~ oufor wheels
Mso
f1r(4.e3S3) 3~2o~w(~.'f17) t3910.1s1ri2o(t. "t17)

t 3:12os1n~o(s)

- 39?0,7 C<l6 ~(3)

.ss9.81 tb
f1t : .s.s,.e1(0.') -

H~

to,,

33.5. ~2 lb ,,,~ whco l 5

30,96 g,417jh

h. 4 .0f78 f}.

C hoptcr 12

Rototion

,.

1202) /\. flywheel 6 fl . in diometc.r occclorotcs from l"C:Sl oi the

of "'! rprn per sec. Compuk. the- normal ~ tonqcntiol componc0ts of ihc occclcrotion of' o particle on the rirn of'

cons\ont role

lhc {lywhcol.. oftor

10 GCC .

Rcql::l : the normal &.., tonqcnti'ol


61vcn : d ~ 6 f.i .
cornpancnts of tho oc.cdcrotion
0( 4 ~pfl'l/scc.
t ~ 1oscc.
Goin:
o< 4~-L
x 2l1'rod x 1P!ffl
/l}l'ln G<X-

o(

~ o.~

o( ..

JP<I'

60sec

rod/sect

urA

o.4f(10)

ur

ur =4,.Q rod/s
Qn ..

rur 2

'
2
= .3 (4.2) 2 ~ .s2.92 fl/s

Ot.

=("<>(

= :a(o.+2)

=1. 26 fl/s 2
The rim of c .so-it'l. wheel on o

Ot
120.3.)

brukeshoc testing machine

hos o GpccO of f/J mph whon lhe broke Is clropp:;d . rt c.orncs to


rest ofter the rim hos truvolc,d a r1ncor distonco of 6000. 'v\Aol
ore the conBta11~ ongulor occeloraflon ~ the number of revoluti
tr.c. whe.ol makes in corninq lo rost ?
61vcn : d " .SO in
Roq~ : o< is....*
v =60mph
.5, ~00 ft.

So\'n:
r= 2s/,a =2.083 ft .

.s =re600 .. (2.083) e~

.. 28Brod

tr e 2S0 roe! ;t 1 ro~n rod


-& "45.!34 rev.
V;rur.
(~x eefGoJ 2.oaB lP.
uf. +~.241' rod/s
d

" -w;,

-(~24):

t
c

2.&
.2o< (200)

o<. = -3.10 rod/soee

Whon the onqulor velocity of o 4f1 diomolor pulley ,~ 3 rod/s,


the totol occolorof1on of a ?'int on its rim 'is .30 fl per-soc~ Dotorminc the a~lar accolorot1on
the pulley ot this 'instant :

ml5.)

or

Rcq'd:<

Given: d=4fl .
UT'=3rod/S
a=3ofl/s 2
.solh:
On - rul 2

!l (s)11..

.an= 18 f'Vs 2
OQ

= Ot~t On 2

= otQi ti 2
2
at" 2t fl/s

30'

at-r.x.
24 = !l<><_ -

0<

c. 12 rod/Gt.

Ocfof'(l'l1no the hor1ioolol ~ vedical cornp~ts or the occolorotio" of pciot 8 on tho rim of the flywhoel '.Ghown in fi(;i. P-i2C6
Ai tho givon pa.Sdion,l)f 4rodpc.r-..soo ~0<.=1!l rod per.sec', both

1200.)

01001<..wioo.
Given: uf 4rad/G
o<= 12 racl/s

r.=3ft

!I
I

I
I

Roqa : horizontol ~
vorhrol c.ompononh; of

\he accdcrotion

.solh :

(~Q) ~ 36 Hfs.z'

Ol =rcx,, " .3

an

rur 2

...

~ 3(4)2

a = ~at~tan 2

"

a. 6on/s

48 fl/G 2

J~

4'6

12~.) A pulley hos a con.sk1nt onqulor occc\erotlOf\

or 3 rod porG0::.2.
When tho onqular velocity is 2 rod pof'GC.C, thot total occolorohon
2

of o pain~ on the !"tin of \he pulley fs 1ofl porGro Corn~tc tho


d1omotG<" of the pulloy.
Soln : On~ ref2 = r (~)'l
Ot r~ r(:3) =3r
Given: 0( = .:1rod/s 2
a 2 = oi + a'l 2
ur =2 rocl/s
2
10 2 " (.sr):2 t (4r)
. Q 10 fl/gZ
{.!

!<(Xfll : diomoler

100 "

_gr 2

r~= ~(1~

d -+ft .

1Gr
0

Q(f

.stop pulloys shown in fi~ . P-12og ore caflncctod by o


Cf'OG(;ed rolt . If tho ongulor occolorofion of c Is t2 rod per.so:=~
what time is required for A to trove! 190 f!frorn re.st ? Through

1209.) Tho

who\ distoncn w'11l D l'f\O'(v wnil~ /\ \G moving 240


6 lvon" o<c !lrad/s 2
.S01h:

.s

180

fl?

fl .

'9

f<cq'd : ~.... ?
when S,,!2~f1.

S <?

"/\.

b) -0,. . - ~"' 12orod

a) e- "/l! O{l,..(l
o<,-. ~ QOl.c
<Ac 2M <trod/s:i.

s,,.. -rfr...,

~180

~Y2d~ 9 120~ V1<.(4) i ~

-e-

t 7.75soc.

-Q&..,

--e-0 = Y2 9<c. t

-a-... 90 l'C1d
90 Y2 (<t)I;..~

1210~ Rcpcot PIOb 1209

if' ihc rodii

~ 18 in.
1
given : c::x.c 2 rod/t;

s"'1eo n.

.GA

('

f>"I

7'J. ~

o(

2.67 rad/!(;"'

.So

2 -(YI\

'72

1n.

2+0/(a()M)

fro ~ Y2~ct~ "Yf(~)(&.48)


&o = 71 . <11 rod

"'' 100 "" .3Skf

-ft>. :

-e-}\

t a.1a '5VC

Z<c
1.SCV.. ~ 2(~)
o(A "

of pvllcy B ore chonqcd to~

% 1/ (2.67) t 2

*ct"'
4

rod.

&A" @6rad
17..... c 12 0(..., i !J

.sotn :
a)

60

(') ~ 100 fi .

-e,, s.../r

b)

t.., .~so when &. 240'

.:3

1 (a.) (1,7s) 2

-&0
br.> ~ rtt0 ~

i:,. 6.71 &ec.

Reqel

fl

r-Bo

~o" .s(n.91)

0:ld .

.So= ~~.S . 7.S

t,..fl

fi .

'h. ( 2. G7) t 11._

t.... 7,34

sec.

dcf1.ncd by the rclofiori tt=2t.30i ~ 16, where-& fs rnoaGurod in radius ~ t in .scconck Compute
121a.) The rotaii'on

of o pulley

js

1hc '<'Oluc.s of onqulor voloo'dy '0119ulor occdero!i0n al ihe ins-

tant whon t " 4GCC .


G'1ven :
fT - Ql 4 - aoc~16

t .,

<t .SCC

Rcqa : ongulor vclovify ~


0'19ular acc.oloronon

so1h : oe-/dl "" ur sl~ -6ot

w=

8(4)3 -60(....) "212 rodjs

ur c st -(;ot
our/cit "' ~ 2~t~ - 60
3

o<.24(4)t-<Oo '=32+rod/s

Tho rotation of a rfywhcol is governed by the equoiion uf


4..f[ ; W'" iG
rad ;anS pcrGGcond ~ t
i11 seconds . -e ~n:;)cl whcr
t 1 sec. Compute ihe values of -& ~or, at tho insta nt when t :a sec.
6i...CO"
Solh )A(c +Jr

1!21-'t.)

rn

is

or "'"..rr

d&/dt ,.

~ = 2 rod
Rcqo :
0<

ur .. +rt

+tW dt
t> + l3/!J.(t/s) t c

de-

-&"'

if i1 a rod when t ~ 1 sec.


2 + (1) 312 (2/a) i c

c. - ().'67
~ 0/3 t 312

\!/9.

o."7 = Bfa (s 1

o."1

-9- 1a.2 rod

dio/<J t . " V2 ("")t _.,.,


.~

2l~r ''

o( -

<

1.1.S47 rod

/ .s 2.

ins.) t\ ba;ly rctotcs occoroif)9 lo thv rolohon 0(,. 2t , w~ 0(


is in rodian.G pc: sc.cOnd ~ t i in 1SccondS. u) " + ~d por .soc ~
t) is zero whon { 1r. zero . Cbmputc iho volucs of tJ( ~& ot tho in.s1ont u>hC11 t - 2 l>OC.
Goin : o<ti
61vcn : . -2 t
dl>l'/clt ~ 2. t
uT- 4 rod/.r.

Rcqa : urr... t}

dw ~t dt

ur ~tY!t i- c
of"+rod/.s u.>hon

If

t O

'!-(o> 2 tC.

c -t
uf" .: ttt lft !ZGCG.
ur 2 a i + .. s ruct/60C

oe-/c1t - vr t
dE1

-&

-,f~D

t. 1 + d't
2

t%1 t-tt tC
i!.._tO :. C.0

-e- ,. t% t .q.t if i
+(.&~
-0- .. 10'7 rod
f} = 'l.

/3

"z~cc.

1216.) Jolerminc the number of rovoluhons ~hrouq h w/c o pul \ey will rototc from rcsi
it~ ongulor occolcrof1on is incrro6cd
un;formly fl'l'.)n'l zero to 12 rad por sc.o' clwrrf"!q 4 gee 'it., then uniformly decreased lo 4 rod per G~.,_ during Jhe ne;.. t .3 ~cc .

ir

! ~-t. cc.a'"%-

-...--~- ---

,, rolls

, .,. ,.,,

.slope, + -. 1. - 61..,

'

o<-12

de-/dt ~+

- 6/s ( t -7)

a-< 3'-at t <Je

'3c< ...

fT, -~ i

-su'e
o( ,. :.fill!

.. ~i.. -

., r -B, o
dCtt/cit-

.3

"t .o .-. ct-

69p f -

8/6 z - 4:S.3S

cit - 69/& !: ~ - 9f1d t~ - 41-5.33t

W1f3t

:.:;t+ c

-e1=e-..

if

ir -.<-o \. t o ... c .. o
w. . .Sfa t2.
f'~ tj- &/3 t

...... t .....

-%(1)-s 68/6(4)'l.-8f1g(+)' - 'hS.:!13(4) +c

c " 60.4-4.3

"'z

-&~ = 6&/6 t - S/1&!."- 16.3-3{ t G0-4'1<3

"111.twt ""t4

-e 6B/ <>(7) -e/ta (1T- 4S.$b) t 60.443

~ '~ t - 8/d... t c

~ a '8/-' ('\)- 6/~ (+)


G ~ -1'.5 .~8
:'./-4)t

uTi -

-fu f'~"t7

-6-e -

H~.OIZ

rod

b8/3t - '8/6-l~ - -i.s.3~

1211.) The ongular ac.cclc-rotion

of a flywheel decreases uni-

formly from Brod per scc in 6scc at which lime its anqulor vc
loc,ity is +2 rod per sec. Compute ihc inif'1al ongulor vdoci~y ~
the nurnbar
revoluiion&' made d uring lhe 6 sec in~crvo l .

or

o(

Blopo

&

a;:

- - - J2t.
~--L..-.J..-...;t

dw/ot ' ~ a-t


w
IP ~ ;
42

&

St-tY~ t C

& f j t G

(6)'~
100 rod

-e- -

..(T" 1~0

t6
8(,)- b~ tG

et - t h

if'

d9'/dt ~ et - t 2/~ t 12
-ft . 4t. ~ - t 9/6 t1~t 1 c
tr ~O 1o.... t ~ o . . c O

-e- - +(<> 2 )

Cz1Z
ufu(. t O

w=-

-1

o<- & .. - 1 (t o)
,f"'<X. .. 8- t

t 12

w-12 rod/s

/(. }{if

1~(6)

Chopter

Wor.,.,

14-

Energy

1-406.) Whol foroo P will 9ivo the .sy,tcrn of bodies shown 1n fig P1406 0 vc\oGih of 30 ft por ~c oflcr rnoving ~o f1 n?rn resf ?

onn

v' ~ zos
a

>
1
3<J /z(w) .. ~z..sn/s'

~ f~1

P " 3991,3+-so(o.~) - so(u.~l


:o12.~

f.,., T1 -100(M) 600(~2'!1il/s~.t


T1

09,0~

p ,.

lb.

44-'t.20 lb

.if.11,'"0 ....

T, .. sg.&&. ~oo~.ots'(o.t.) +fOOliin~S

-[~(!Zt..si//at.t

Tc - 399.3+ lb.
1-107,) find Ilic velocity
10

of body A

in F'9 P- 1-wr oflor ,t hos l'TlO"'ed

n from rest . l\r.6umG ihc pulleys 1obc wciah11c.ss ~ frictionless '


1 ..... 2

300 t9.320A

2T2
4'

- -(!)

"3

2(W0 - 1!Z,'4~Q..)

"I-CO-

~4. 6'\-Q...

3(X)fg,320A

300 t 9.3ZO>.

100 " 3""' 1~5 DA

v"'~

a.. - 2.Q3
~as,..

n;,.i

.....----.

v,.. a~ f(.~u)(10)
YA ].(,S fl/s

1Ai

a:l? T2

Te 200 -

"(200/au)(a.,)

6 . ~1 Qa

Tt wo - 6,t1 (ta,.)

Tt -

2(J0 -1f,-'t0A

--@

fig. P-1408 mc:No 1n


velocity from 6 fl parsec to 12f'f perGC::C

14-0&) Through whol distonco w 'ill body A in


chonginq its

~ \....~

n1

a... - -

300 t+'~

~ ~1

t(t00 - (..9.10...,)
400 - 12 ,iof.i:!

=:

3C~'.H 4 .4){,Q"

a.._ aao t

o..

4 -~'

100. - 11.. oe a''

o,,' ,;. s.as n /s.12


%
vl>.2 - vA, - eas,...
~

~' T, ~:~:::~ -
1~

12 1

,..

r~ -~c~/_gz.~) os
Tt:=.3oot

140'1) Dcterrri1nc

= e(s,es).SA
g.23

./1
~

<J,32(0,../.12)
a.., - -@

4.~

fl

'7j.

__LJ_j

T2 " 300 t Q.3t Oe

11 .9Cl?t

6~

~2 ~
\

tf'A-::>- Z

the :vclovify otlolned by block A in F\9

oftor movfrll3 o disfrak.e of H1f1 .stort1nq from res~ .


T1

"

~':'~+~ 1.,
'

0t - 106+DA - -@
1 "'2

111,. 30+ t

1+12 OJ\ - .. :: +

48,._3

~(1<>+-11,+a ...)

40& - 37. t6 a,..

"304 f H! ,+t o"


=

304- t 1 t.4-1

104 =~.7 CM

a... - 2~n1~
v,.,i

2ac

V.... ~ 1(L.a<J)(1e)

V"

Ta
Tc

as

2~ - 0.a2
f04- 0,3~ Ct. a..,)

>

1.08

f'i/.s

a,..

1"4-10) In whoi d1... tancc

city of 11

will blocK A of fi'g. P-1049 attoln a velo -

A por sec, Gior\i'nq frorn rest?


1 Z..t
~lt. aD+t11Z..<!-'10A

- -

... 8...3
1.{W4 -13.~0,.,) dO+t 11'.42 0A
~06 37.1Z60A 30-+ +1!2. 4!1

104

a,..

"111J,7Q,..

o..., 2.og O/s '


ta~,.,

V;... "
/l

1t

fw -1eo(o.t) -l t eooae
ML~)

Tt

~ - 9,a2a6

~ (fl,O!J) bA
EA "3'4A1[J .
c

Tt. ~"' 0 .ad1ZOA)

/6(111'

r, ~0<1--1e.'4aA _..-@
n%4 in the poG'hon .Ghown in fia P -H11.
Doi . 'i\s volocity oflor it hoG moved ~fl olorig the fnot;on loc' ~""
focc
8V11 ov,..

H11.) Bodr " s1orfG frorn

Y6 0

Gubstdu1e Ve to 1,
1400 .3.11 (o) ~ t 4,6(, VAz.

y,.,

Zo.jis '- ta
zr~

17.33

n/.s.

17

Io$&.. . e'

z ,..

e~~ '-

zz.a/ot ~.. d,./c::1t


f, Y&

- ;t.V,..

RW ~ u>;i~ (v 2-v,/)

too (11.

-8) ~ tOO~'t.4

vs" t aoo/6't+'(Az
- -0)
~11.) Find the veloi1ty of body I\ In fi'q . P - 1411 ofier , ~ hos
movod, 6tarl inq from rcst oi !ho givor) pa;dion, for 9 flcfonq the
fr1cfionles1; surface shown .
1"!-00 .3 . ~1 Vet. t .4.66VA 2

l'.1.t0"~t

f ll.d ,./ ot aoi/dt


)(V.-.. 2.Vo

.r.,

Jis

2t

- 17

e~

Z(' = ~, ~I 6

10

RW - ~ W/tg (v' -v,/-)


1
too(11-1c) "t00/1>+.4- Ye~ t ~/G-+.+Y,..

--

(;Y,. 1CVe

Ve 'V.../10
1

('"''l~o) - -- -@

v.'

11

from1, l"'t<lO a.11v~!t4.. ~'v,.,

subrotitule.

1,

2 to

1...00 3.11 ((j,v,../10) t ..+. '6 v,. f.

- s.1av;. :i.
" " c 15.56 f-1/6

1+m

connected by o l1qht, rigid linll. 10 (1 long, mov-e in


lhc frictionle&" quidos ~in f 19 . P - 1+13 . Determine -!he ....elocity of 5 whct'\ .x 6 fl ,f' /3 Giads from re%! when vertico lly belOV'( A
Asi:ume w,... -wa =10'.J lb Ii.. We 50 lb.

1+t3.) Two .slide!;,

SOO., 1 . ~~' t1.s1>Vs"" i077'V"


2

.soo 1,0i;v,...a t !Z.~vs

Slb:iiluto

- --

to ~

500 1 . 61>(.36Y11o/~'4-) +(.3:JVec

v,. 1f..S fl/.i;.


x t y 10
ty' 10

6'

rert
9)(dx/d t t tydf,H " 0
J<Vs
6Vs

-rv"'

av..,

'(,.. ( .. ( '""/~). -. -(j)


RW wh~ (v 2-v,,)
100 (;o -8) i .50(6)

"IOO/fr++v"' Hlo/''4'A-Yect6o/~..+Vcl!

Ve Ye

1414.)

Repeal Proh

r 10f- 6L

1413

v,., c e(?>Ve.i<,)

- y 6f1

.,,.. 1yt 10~

2~G1it/dt t

when )( 80

t.ydr /en =o

x Va - yv"

8Ve - 6 VA.

2
-

-@

RW ~ W/ 1e9 ( v'- v0 <)


1
~ 1oe/61-tY,.. t1W~-+Ytl ~;i

100(10-&) t-50(6)
Ye Ve,

ea> 1.ssv..,~t1 .ssvs" t

OH6Yee.

800

= 1.ssv..... 1 t .33Vs

9.1~ 1h.1 to 1

&oO

--

io ~,

-1.sfJ(Vt-V&c.13,). 2.33Ve~

Ye 1f6i n/s
1"117.) /'. wei<,ih ~ Ir,; droppod from a pa.;ition Ju~f abovo, b..il ~~ tovchin9, a ~pri'ng . .GhO\N the fnoJ<murn dclOrmoton prooucod will be

twlOc that if tho sorno ~oig ht '' ~i,oo lly lo~ uP"" the 'pril'CJ.
'It. K.b/ wlf,
6, tW/11..
w. "-oc.

t:,, -W/K.

1>1/b, (1.YJl(y(;4)
& !lbt
2

A block w~ighing Q6.6 lb is dropped from o hc1qhl of .otO.


upon o .sprinq whose modulus is 100 lb per in -Whal vclocify will

1+1&.)

tho blxk have ot ~ho ins ton I tho .spnnq Is doformed 4 In.?
6 1von:
so1n:
WQ6.6 lb

h -+ rtK-100 Wiri

.+in
: v'olocify

w(htb)- ~ K.bc. wv~q

96.6(4 1-+/1L) - Y2(1ooit12)(4A~)' Q6.6/"4.1v


Y 'JS,.32

1+19.) /\ 600 lb black .slid<A down

fijs.

on inclino

having o .slope of ..3

vcrlicol lo 4- horizoo lo/ It -' lort~ f rorn rcsi Ir.., aflvr moving i feel,

n-

Glril<o' 0 spring whO!iO modulus iG 100 lb per


If fhe coeffteGnf
of ~1netic fr1dion i' o,w, find ihc moicimum velocity of the bloc,k .
61vcn
SOI

n:

W"6001b

-'-

~t\

"'ope .3/4

1<.- 100 lb/(/

.. -

JN. o.-i

d 4n

RcQa :

mo)( vclodty

[w(~ls) -w(~)(o.i)](d 1.S) - Vz K6 6 = wv/f9


ooo{tt6)[3/s - '/s(o.i)] - 100/t b~ ~(600/6...t}v e
2G-to 1 1os6 -5006.. g.:;,zv"-

d'ffef"Cr'ltiole velocity wih re.Gpeet to 6 ,

.2~ 6

~ g.3~
2."+0 (for mo,,. velocity)

tOOb
&

600(1 t 2.w-t) [ a/.s- /s(o.z)]-SO(fl.6+) 1

9,32vr

v 12-28 ft/s .
d iGfoncc mu't tho 6<X) lb blocll. of Prot>. 1+1Q
slldo from ~t before louch.1n9 the sp~nq if is velocity i~ 10 ~
at ihe in,font fhe .sprin9 \c;; do formed 3 fl f Assurrie the sprrnq c.oostont i' changed fo .30 lb/in .
1420-) Tl1rou9h whot

Givan :
W 6001b
V t Oft/t;

..sorn:

[600(3/s)-600<'.4b)(o.t~(d~3)-[30(1t)(~)Vt (,00(10) 2J/li+.'4


t<i+ (dt~)

3fl

QS.s;1.7

d , G.67 1 .

"'- 3olb/,n
Rcq(:j : dislonco

/\weight of W lb i'" suspended frorTl o verticot spring (F<9 Plb per rf , Tho Wci9hf lG pvJlcci clOv'ln S
from ifi; cquili t:rium. P,O"if10n "" lhcn rclcosod . Determine ,~, velocity when H reiu ~As to ihc oquil ibr1um position .
1421.)

n.

11-1Z1.) Wh050 mOdUIUS iG K.

~K8t ..

WV'i'Rq
' v'=~..g

" - ~~ "'o/w

The rigid hori.:ontol .shown in f 19 P-1t22 i5 supporlcd br 2


Gpr;ne /\!\CO lb wci9hf iG i ho plocoGI upon iho barGlf fj, I\ ~ddon
1..."1!2.)

blOV'<' p~ccts 1ho weight toward/\ wilh an inifiol volocity of 6fl/'


W'hol '1 c; it' vel~1ty when ii rcacties /\ ? Hcglcci frict10n " tho wci
ght of t~e bor.
300 r 100 (6)(12) so(1~ )6

6o.sf1
(300/r;,+.tt)(v,' . ,t) ~ ~1f)fo.sY c -'Ofo(1z)(' o.s)"
~

v~

- 1(,i.q

Vt "' - ~1

fi /s

lfoglccf friction of \lte 40-lb collar ogoinst its vertical gu'1de


'""'C0ffiPIJW the volocily or the collor aflcr 'ii hob fallen 7 ft I stodinq frorn rosi in the position shown in Fig P-1423. 1ho unsfrekhed
length of I he spr:nq i<: 3 IL
I
L " ~ .3 2 ~ 3i -+ H.3 ft .
6, - 4, 2-.~-~ ~ i.243 fl.

1'423.)

"'IQ(a) t

V2(sx1:z)(M4;,) 2

"(..c;{;'t.4)Vll

v 16.365 n/s.
L-~3'M1:

6t. ~ .s-3

.!lft .
-~n .

r~ (s x.12)(2)~ (40/G+A)(vi-1~1 %5")


v~ 1 =- 332 .2 1 W'./G 2

4!0(4)-

V2 = 18. 23 flj.s .
1424.) R.epcat Prob. 14t3 if ihe unsfre!ched lcnght_ oflhO 'pr'mq
<Oivon :
Soln:
w401b
L~~3~i.3" 4.243ft.

RcqCI : Ve loo ty

b,
"'1(1(3)

is 2ft.

.+-M:3 - 2 -z.24:3.ft.

12(i;x12)(~~3)'1 (-!o/~.4)y 2

v 20.86$ ft/.s
02 ~~H2 3ff .
"io(+)- 'l2(s)\12')(3)z
Y1 2 ~ 2S..9.1
Vt. "161

=(....ofi;+.+)(v/- 20.ess~)

fi/s

1425.) The CDr in Fig. P - 142.5 ii; mov1'ng toward !he bumper spr'ing
~has a 1<'1nef1c energy of 1~cw in-lb. The mciin bumper &hield
(aa) i-;; conocc~ed to the rroin spring, which hos o modulvs of1000
lb per in . lho lwo ouxilie1ry bumper shiolds (b) arc 12 in. behind oa ~
oro attached to secondary .springs. coch o( which has o modulus of

fl por In . 'Who1 the c.ar is bl'VIJ9h l io rest, what will hove been
the grccit~ct ma1cmcnf aa .? Whal porconia9c of the energy has b:=on .
ob60rb::d by the rnain 'Pring ~
soo

Given :

Golh :

KE " iOO,oo::> in- lb


J(a 1000 lb/on
i<.b .. .!JOO lb/1ii

F<cqC! ;

.s ""' /.

'l'H1000)(1u6) 2

Y2 (1.)(soo)6 =1~000

soofrz'1246tb)t ECOb'- 100,000

1ooob t

12QX>6 - 2aDOO .. o

bf. t 12 b - 28 , 0

~ - 12 t

..f!2--t(1 ) (-2&)

2 1n .

.1(1)

S:.. 6 1 12 2t12 " 14 in .


KEs Y2K(12-tb )

/<.Es Y2 (1000)(11-t2)

/K ~

~ i<-Es

"' <JBOOO

= 98000/toaooo ., ga Y,-

1429.) A iroin wcighi'nq 100 Ions is boing pulled up to

21.

grade . Thv

iroin res~onc.c '1i;con5+ani oi 1o lb per ton . The Spe.od of Ille ft'Cl~n


Is ;l"Orco'od fram ro fl P8"' Gee to 4C ft por ~. 1n a di.stonco of 10X)fi .
find tho rnox1mum rcrsc power developed .
~In

61vori :
W 100 ~oni;

~ S =

wfzg (v 2-v.,a)

[F -100(10)-100(1000)(too)}x1000 100C'a:xx>X402
6 ++

R 10 lt>Aons

s 1000 n.

F 8, 72G . 71 lb
Hp"' FV {8,726.71 (405)/sso
I
Hp::. ~34. G7
1+~) Water FIONG through 0 nozzlo 1 ln . in d1amotor under a heocl
of 400 fl io dn-.C 0 ,furb'1ne. The furbiM j~ 90'/. c;rfic'1enf ~ jG conna;~cO lo o generator wjc is 941. efn cien~ . Vv'ha l is tho power oulpot
in 1<.l lowoHs ~
GiYOn : d 1;,, .
ReqH : i:bwcr l'.A.ilput
Rcqo : Powen in +Ip. .,

~oln:

l'.bwer; ~Qh f v

Fil ~ AY'1
F

",2. +~T(1M)'J4J[100] ::

1/!3 -"""~---

1w.sn/s

" =f2/:3u)C400)

hp "'

f V fr36.t3'

hp

136 .136

(160.sfl/ sso

= .3fJ.7.3

lb.

e output /1nput
a

0.9 = hp /39,73
hp(-rolor) 3S. 75 hp.
0.94 ~ hp/3-5.7.S

hp " 33-G1 hp
Powor Output "33.~1 (o.7+6 1<.w) , 2s.1 KW.
1431.) Water enters o hydraulic reacllon turb'1ne w/ a vckX::1ty of 1z
per GCC ~ loavo5 ,~ 3(t lower v-.:dh 0 velocity
4 fi pcrGcc. If
100,()(X) lb
wafer flow through ~ho turb"ine cooh .second, compute::
the ho~cpowor output . Ab5umo the turbine iG
efficient .

or

or

soi-

6ivcn:
"1 12 fl/s
ve~1n1~

ea 001.
Woter IP 1ooe 3

lb/s

h" 3fL
Hp Ovtput
fuln :
'4j49 = v~fag
Rc:q~ :

(1~)~(s2.2)

-1 b t

<t~(32.2) t 3

+1 ~ .... 9e16 n.
3
Hp,ul'.et~.. "" \))H 1oox103 (4.9S7~) = +90.76 x10 fl - lb
3
t1p = 498,], 'l( 10 fNQ_ )( 1 hp c 906. 04 hp.
sso ~-~
.

e : :. Pcutpuf /Pinpu+
o.a

9o6.e4 hp
Poutpuf = 725.S hp

-= Poutpvl /

Chopfer
lmpulGc
I/

15

Momentum

1so.3.) /\ 300 lb bloch is in conta1;t with o level plane wh


so coefficient of' k1i:ief 1c friction is 0.10 . If the block Is acted
upon by o hor'izontal : force of .50 lb, what t irnc will clopGo before the block. reaches o voloc"ity of 49.3 ft. per sec, star ting from roGt? If lhc .sO- lb force ,G then removed, how much
\ongor will tho block. con-t1nuo to rnovo?

Givon :
>
Goin :
w~ .3001b , ' fc.S(Jlb
~

p.."" 0-1/

Y "- 40.3

Ft-

fl/G

Rcqo: timo

~fic = O

[S0 - 300(0.1~ t ~ .3D0/32.2 (48.3)

t: ~..s sec .
.300(0.1) t .. :300/:a2.~ ( +a.3)

" 15 SVC -

1.504.) A hor1"zontol forco of 300 lb pu.Ghes o :200- lb block up

on inclino whose slopo is .3 ver!t'oal to horizonfo/. If f"-"'0.2. /


dolcrrninc the l ime required to incrooso -the vclocil y of the
block frorn 10 to so fl per sco . .
G1\lcn :
.Sol'n :
F
f .3001b,, W:!Za:Jlb-

m=~;)A..._ 0

0,Q

Y1 " '1Dft/i;, V:L ~.goft/6

Rc.qC:l : t "1mo

aA;(~~(4/s)

.
[.300 -1w -160(()~)] t " 2~fti~~ (so-10)
146t .. :240.%

t " 16B soc .


ccntrol 1mpoct occurs bot. a 100- lb body moving
to tho righ l Of .5 fl per Gcx:i "" 0 body of wo1ghf W moving to
the loP ot .3 ft por .Gee- The cooff1'c'1ont of rcstdufion e ==o.e.
A flcr frnpocf the 100 lb body rebounds to the lofi ai 2. f-1/soc.
Peicrni1~ the weight W of the of her body .
Given:
Sol'n:
1543.) Dln:x;f

w,."" 100 lb , e o.s

WA.YI'\ t We Ve

v,.. =..5H/s , y,; > 2 HIs

1oo(s)- 3W = 100(-2)

We =W, Ve

csfl/G

Rcc(d: Wc(ght

<

WAV..; i Ws

Ve
1

t WV0

700 ~ -aW+Wve' -.- -G)

e = va' -Vt\/v"' -V1l


o .5 " Vs'

Yi=

HZ./!5 - (,-a)
2(i/..s

.substltufo v~ io equation
700 =3W ~ w(a.)

1 ,,

W " 1+0 lb

1s+4.) I\ g~lr boll is clroppocl- from o hoi9ht of 20. ff upon


a hordonod. Gic.ol plato . Tho coo(Ti'cienf
rosMut'o n is 0.8
find tho hc1ghf to which the boll rebounds on t ho fir.st, ~nd,
?.., third bounCCG.
>
61vcn;
(o.sg4) 1 (h1) , h2

or

h zort , e=O.S.94
ReqCJ: h1 , h2 &..._ha
Solh:
e =~ hih] tiiA,1 lt3fti2
#

he1f.70fl.
(o.094) 2 (he)

h:3 10.~1 fl

h3

0,094 .. ~h1ho ._ h1"15,gg.fl .


154.S.) The ballG A &., B in Ft'g P-154.5 ore atfoched to sli(f rods

of n~ligible woight , Be1ll A

'16 roloaGod from roi;t ~ a1lowocl


to sfril<.O a. Ir tho cooffto1ont of rv~fitution t<i o,6, cicionn1'nc
ihc on9lo fT thr-oc,,gh which boll B will .swing If tho 1mpoct

la.sf.s
.

for

0,01 GO<fr a160 find tho ovo-a.90 imfX)ct fon:;c


,1
~
'
.s;o1 n :

61ven :

e,co,6 , t:o,Ol.GCC
JZcqd ; fT
I
;

'
j

A..

- -- _ _ .__
-' 'b,
w .!)olt> ,':

"-,

'

h .. ,sf) . ; b 10-5 =.sfl


. '
y,.. af2.9h " J2(32.2)(5 ~ 17d;J<f-fl/s

vs' "

G:>.&60 =h 10

/\

'

W,..V" ~ wAvA' tWeYs'


30(17, g4) " 30V" ' t MVs'

.s aa.2 = 3ov,..' i ~ov6 ' - . -@


O., fo{ - VA').-17.g+- O)
t(),77 = Ve' -v,.. ' - -@
from 2, v,.. 1 v8 ' subsitufc fo 1,
.S36, 2 ~

10.77

30(ve'-10.77) i IZO Va '


aove - a.23 ,1 t -iovs'
8GP3 =.:.ova' --.. vs'~ 11, i26 fUs

r,,39,12 ~

17, 2!26

--.

'

,(Q9"0 ; h s-c
~ :(_32,fJ)C

C R 4,61

h.

0 - 4.~1 ~ .;,,59

ft-

CosiT .. a.ag/s
fr = GS 0

o- 4~37B

15+6.) The system shown in Fig. P - 1s4.s is used to defcr-

mine the ~ffforon l of rest i lution. If ball A is released from


rest ~ ball B sw:ngs through -9-"' 53.1 o0ef' being $i ruck., deformino e,, .
Soln:

eos s~.1 " h/e

eos6o" h/10

h' =-+.en

h~-5ft.

b 10-.s
=

v,..

c~ 0-4.~ =

>.!;fl '

f29h - ~1-2-(3-2.2-:-)(.,..-s

:3. 2fl.

Ve' " ~(22:2)(:3. 2) -= 14.35 ft/s

M
w,.,v" =wliv:. t WaVr/ .
c

11.94

30(17.94) s3c:)v,..

20(14.35)

v,.: = 8 .37 n/s


f}

=fo/a'-V...,')/0/A -Va ) frt.35 - (3,37)/(11.94-0)

e .., o, 333

A ball is thrown at on ongle fr with the normal ~o o smooth wall, os shown in Fig P- HS7. It rebound6' ot on 0119!0
&'with the nof'mol. Shovv thof the coclTlc1enf 0 rc.siifufion is

1547.)

expressed by

0,,

Iii

tan &/tan-e-'.

.solh :

,D

e "' (va '- v1')/<v1-w.)


mV1 CO.sf7 rnV/co.s&'
V1 sin9' aV, S1n9- - --
e = -v~'co.se-/v1 cose-

"-,*,....,,

,,,,, "''"~:,,,,,,h,,,,
from 1, V1'

- V1.S1nfr/s1rie-'

e ~ (-.ll(s1n-e-) ( -cosi7
s1n1v

o "

1
)

= tone cot-& '

Y{co.Ge

tone/tan&'

ts.+e.) /\ ball is thrown w'dh a vclocify of so fl per sec d1rcxtcd of 60' with lhc horizonfol cigoinst o smooth vcrtical woll . '
1hc ball is rcloa~cd from o poG"dion . 4<JA lhc woll oncl 6flobovo
0

the level ground ~ trovels in a verheol plonc . The coefficient


of rcstilut1on behvcen ball ~ woll is o,6. How for rrom !he woll
d~G the ball sfrike the ground ?
Given :
ReqCJ: di5~onco from lhc woll
Vo = sofl/s
when fhc boll .sfriko ~he 9ro1Jnd .
R .. 40(l

Solh:

_ _ ,,,o' --~

16.1!4 t

=so c.os 6() t

t 1.6 sec .
yVoGn&l - YJrol"
y a .so.c1nG0(1 .E.) - Ya (.3!L'l)(1. 6)"-

Vy 6.f fl/~
Vs1 ~ J2s"1e. 2 .. '" .26.a1 fJ/s
o< ton 1 &.2/2s -18.16'
,,

----D

--] -

.,, ,.d.18,1'.
.. ~

~,,....,,,,,, .

e ~ v'e.a J( /vs 1J'.


2

Y~ x/26.31 ca;1e.1"

Ysz7.- 15

f1/s

2,.~1.s1n 16.1h

'le?.y

.. Ve2y

a. e 11/s

Vs~ -./ 16 .. H!.:12.


Ve2 11.1+ rt/s

fon _, 5,1~s

12

~&. 66

8.'2t

t:i

-a "l'07 O

+J (S.l) ' - +(1, 11)(-3+.01)


2 (1,.1 )

><

Yx 250/t;
Vyc - Voy 20yS
2
Yy 2 -{so,1n60} ~(32.'2)('2S.01)

o.<>

~9t'

1.222

~cc. '

n.

-(.50COS60) 1 O

,.,,.

- 6.t

t.

6 tzS.01 .3+.01

Y,!-Vo-,. 2a~s

y./

t.

~0. 07 (l.

y -

H-

34.07 11.1s1nte."t1161

flaVoa>5~l

'40

--- ><

Yosi~t

- Yo<:oslt
- 17.1 Q).s

28. 66 (1. 222)

'f. - 18.3 ff -

h shown in fiQ . P-1&-4-9, o 4'0- lb boU movinq horiumtolly


to iho right with 0 veloOity of 6 ?"' s;cc, collide~ obliquoly w/
o ao -lb boll moving up to tt:ia lcO at 3D' to tho hon:umtol al 1l1
fl per sec. If tho coofficicnt of resiitutioo tG o.o, aotermt00 the
amouni \,.. d1rochon of thG "cloc"1~y of eoch ball dlroctly oflcr
~.)

impoot .
w,..olt> ~llOlb

~~
v,a.1on1'
I

m.V1" t f'l'lt'4ll ,. m, Y1 +m:t Vb


.fQ(&) - ~(10 cent!)
v1 ~ +311'1;.,.

"'

,0, 2

"t<>V1~ t 31JVJ,,

--6)

e "(v,,;. -v1'vv.. -vu

'' .~ -v,-i'}/

& t f()(".0&30.
10 '- Vr.~ -V1/ - -@

v;., t m. "2 y

m."41 -1 m.."2y m,
MS (10 srn 3{f) c

...c v.}- -v,,i ,. .s n/,

v.; 1otv,;

from~ -

su~t .

to

"1.t ....cv;. t .50.(-1o~v1;)

'''

x
V1.,,' -3.4S n/,

'4<>V, t 3G10 faOV1 ~

-239.e 1ov1~

v. ' V40

3.43

0/~ lo tho Jofl


10~ (-3-.+.!) - 6,57(l/$

from 2 , v,; ..
v,' J'4.:' Vir1
Y1' V30

~''1t~

8,t~ nfs

-9 ton-1 V//v.;. fan 1 .s,4,57

-e 37, f.7

Y30 a.~'

n/, up to right

3 1, 21

oi

w/ fhe h:lrizonra l

~ee~
lv1.V1 "Mt'h M,V~' tMtV/
KE1 'ti M.V1' -1 1/r M2vi'

kl:, ~ Vt M
,v;' t 1/t M,v,,,''

e V1.' -v1 '


V; -V2

PROBLEMS

Chapter 2:
1.) /\ r '1gid bar C"BO supported os shown "' Fig . B is ocicd
VpOr'l by two equal hortwn~ol fore.es p Opp\iec;I o \ C \-.0. Cal -

Cu\o\e the reoct ionG that wi\I be in duced a t the poiniG


pporl. f\Gfl.utne l --tft 0 a3fl b 2ff

1l=, =:-Jr

of..su-

M Rb -!lo 0.2SP

2.) Owing 1o the weight W of t~ locomo \ivc c;hOv-(n

in Frg t=. lhe


reoc t 1onG at ihe two pain ts of support /\ 8 wi II eoch be e quol to W/ 1. . When the locomc1 1ve ,~ pull ing o 1ro111 !!.,,
drowbo.- poll P ico jubi equal to the total fr ici ion ot the pt&-.
of contact A'-.. B, determine ihe mogn'itudes of +he vertical

"*'

rea;tions Ro~ Rb .

tt:e

Ans. Ra" :!:!... - Pb Rb= w + Pb


2.

"PO" ,

'2ci'"

......) For the pot"ticulor position shown in Fig .C the cornecting rod BA
cronk.p"1n

of an engine e)ler-\s o

force p .. .!XlO lb on the

ot A . R~Gplve 1his force inio iwo rectonqulor com-

ponent~ Pn ""-"R,/~~ting

horiz.onf(:dly 'b... vert ioolly, ot /\.


/\f1S. Ph ~""!:OB lb
Pv 177 lb.

~.) A vel"I 1co\ lood P i'D 1>upp()l"'ted by o t rionqulor \,ir'OcK.ct


os 6hown 1n Flg .e Find tne tOf"COS +ronGmitled to the bolt6

A !l.. B . /\ssume thol lhe bolt 0 n\s loosely in o verlicol slot


/\ns. Ro 1.2s P : Rb = Q.75 P
in lhe plole .

Chapier :a:
1.) /\ woiqhtle~ bot"' AB
0

n.nge- ot /\ ~ 0 tie bor

colly.

,c; Gupported

the oy.101 fon:e

oc,

OS

"in

o vertlcol plone ~

ehc--m. Oeter-rv\1ne , q-ophibt trn, ociion of

S induced in \ho \'ie bor

vediool lood P opphed ot B.


l\nG.

= 2P

(ten&10n)

i~ /\ roller of r-odiv r 11 ,n . ~ w~i9ht ~ .. .500 lb u; to be pul\ecl Cl"er a cvrb of he'19h\ h 6 in. by o hcr1;iont o\ force Pap plied to the end of o strinq wound orouncl the circvmf'erence
of the roller . Find the mo9ni+ucle of P rcq,.i,~c:;I to Gior t
roller (7olef" the curb
f..n.;. P 1.88 l b .

+he

,,,&;:~
I

0 s~h horiwn1al plane ond


hos oHoched io 1\i ccn~or t .....o Gtringc; AB~ AC ,,..,h;c h poss o ~

3 .) A boil of wc19h+ w re,+ vpon

fri otion\e" pul\eyG 0 t 0 ,bi-.. C ~ corry loods p ~\Sl, l"ec:pecii-.cIf the s tring AB ,s hori :co.-.tol, f.f'd the a .,9Je o<.
lhot the sfr1nq /\C, ' rfjlO~S with the hori~oniol when the ball is

Yer

ly,

06 Ghown

in a po~1 t1on of e(iui l;br1urn . Al60


. '
the bal\ &- the plane .

find the pressure R between

/>vis . coso<. P/Q..; R .. w -/~" -p 2

the o)liol f~ , k.. S!i Induced in the borG


lhe Fi<aut'O due to the oc;hon of Hle hori x.antol Of>'

.4.) Delermne

/\C ~

ec

'in

plied lood ot C Tho bor& ore hinqod togother o l C $.._ fo the


founclo\tOr- ot /\ a.... B
;v... ~" 1e2 lb,(tc:irWon) ; St. ~lb,(comp .)

.5.) /\n electric-l"iQht fotvre of weight~ --w lb i~ supporleci o s

Determine the tcn~11e forces' Si !!.., ~ in H'le


BC , f their ongle.s of inclnotion ore OG Ghown .

ioh()wn .

/\ns. $ 1 20.3 lb ; S2 ..

wires

20.7 lb .

BA

St.._

Chopter

'4-:

Oe1errnine the rcoc tion ol I\ "-. the fOl"'Ce S in lhe bar OEdue
to the oction o f the load s P ti...~ opp lied to the crone. os shown.
Neglect the weigh t
the crone &._ ossume ideol hinges ol A. D.
'*._ E . /\ssume tho\ P ".!lOOlb, ~ 300lb, o e fl

1.)

or

1-o 4-o -l ,.-.n6.

Ro "' 1140 lb; 5 ~ 1~!S lb c;ompre~ion

f;::::::::;;i;:;:::::::::?c

1o...6 of the plane


Ans. S, - P ; s, tP;

2.) Determine the oxiol force& ,no bor"S 1. 2,3,4,

trv" liupported 'it... looded o; ;tiown.


Sa - o.&P; S,. ..

O~P ;

Ss - o.335P.

a .) Deie.-mine the for-ce S in the bor AB of the plone truGG loodod "" supported o& shown .
c;

Anr;; .

s o...,.3 P .

~.) A plone figure - four frome /\BDf: i" Gtlpp0ried on on inclined


plone 'I!.- loaded os ghown . Colculo~e the axial force induced in
the member BO.
Ans . Sbd ~ 106.7 lb, tens;ion

Two beoms AB~ BC. joined toqeiher by o hinqe B . ore supported by four barG, hinged al ihBir ends. Oeterm'me ~he force produced ,n each of the~e bore due to fhe achon or lhe load p . $C0 lb. The
dimenc:io,.., 6\lhe G\ructure ore os shown. Ans. So 160 lb.Comp . ;
e.)

Sd Se 2Gs tb. Compreee'10,..., ~

Sc ..

62 tb . tens.ion .

Chopler 5:
1.)

Who! ~ the neccssory coeITic1enl of friction belwc:;en Vires 51<..

roodwoy to enable the four wheel dr ive oviomob'rle to climb o 30 "/.

9rode?

2) Two block. hoving weiQht~

w,

1*. Wt ore a:innected by o "t~ng !a_

~Gi on h01"'i~nto\ plonec; OC shown

fr \he on9le

Ot rridiOf/

f,

r()('

eoch bloc!'- is
Find the mo9r:i"dude &.., d1rection or !he leost
force P opplied to the upper blocK +hat will indvce slt.d1n9.

/\ns. Prrii~

- ( W1 t W2)

sonf.

or

~.) A 'sr'i'looth c'1rculot ey\;oX!ef'


Wdigh' ~ ~radius r is supported bY Mo Gemicirculor eylinder ooch of -the same rodiu1> r &.. weight 1S(J2. os Ghown If the coeffiCient of' stotic fridion be+ween tre
flot facer; of the semicircular" cylinders ~the hor-i :z:ontol plone on
which Ihey ,-.,st ic . = 1/2. \...., frii;:;tion between the cylinders therri GelveG ,c; neglected, deformine the rnox imurn dis-t')noe b -betv-(eon
the c;en\ers 9 ii.. C fur which equi Iibr'1um will be poc;sible w/O<J I
the. rniddle cylinder, touching the horizoniol plone .
:f I '
/\ns. bmot. = 2 .0.31
,

"

'

Re ferring Lolhe figure . the coeff."cienh; olfrlclion o reos f'ol low: 0.'25 o l. +be floor:-, o.3 ot ihc woll, ~ 0 2 between blocks. Find

+ .)

the minim1.Jm value of'a hori~niol forceP opplied io+ho lower


ihe GYSfem in cqui_l ibriurn . /\ns. Pmin 81.z lb

block. thot will 'hold

e) A solid ri9ht circulo" cone of altitude h?. 12. in. ~ racl1'us d.'
bose r:ain . has its cen~erof grovily Con ,ts geometric o><i\; ot ihe
d1~tance h/+ ".:3 in . above the base. This cone rests on on inclined
plone AB, which rnoKes on ~ngle of 30 with the horiz.ontol &,. for- 1
. which. th~c.eefficlen~ of' fridion is p.=o.fl . A hori~ntol force P
i~ opp\ied to the vertex 0 of the cone ~obi in \he verticol plane
of the f1gur-e os shown. find the mo><imum ~ CT)inimurn volues of 1
p consistent with equ'dibr'1urT) of the co'ne if' the weight w ~
10 lb .
Ans. Pmo,. ,. h61 ltJ ; Pmn" 0 .590 lb.
~--P

Chapter 61
1.) ln lhe C<lSe of the tripod shown . there is no friction between

1he ends of the leqs ~the (1oor on which they rest . To pn:.verit
slipping of the legs their ends ore connected by stringG along
the lines AB, BC, ~ .AC . Determine then the iern;ile force S in .
eoch ol \hese strings if each leg makes :30 wHh the verf1col ~
pis o vertiool load
An$. S = 1/g P
p
0

2.) Determine the forces produced in the bars 1 to 6, inclusive,


hvss GhOWn, 0"'f;l1g to the ac tion<: four verlicof
loods P applied os shown . ACBO ~ A'C'El'D' a~ two squares
w i lh porolle l sides ol lengih~ a~ .20 . respecfivelv. !'i<.. 1he

or ihe Spoce

d istance between hori zontal plones ACBO &...A'B 'C'O'is 2a .


Ans. S1=S2::-0.2SP ; S3 -= S.,.-1.ooP ; Ss=S, .. O.

"
a'

in

3.) F1iid the tension T


ea.ch
crone looded o s s hO>Nn.

of the guy wfr~s 0 0 l'N. Be or fhe


Ans;. T::: .+.:10 tons, e ach .

c
p

+) A sirul AB otloched to lhe focc of o ve....+icol vvol l oi A by


o spharirol hinge !>lands perpond1eulor to the woll ~ ,s sup ported by two guy wires os shoWTl . f\t B. 1'n o plo ne porollel to
'the wol I,two forY:'es P &.:..~ os GhOwn, '(i? being hor1:onto 1~ P;vertical. Uing the meihod of moment~. find the oxi ol forces prOduced in the mernOOi-s , f P~.soo lb&._ ~ 400lb.

Ans. S, = 103 lb; S2-= 1625 lb ; G3 " -1,600 lb.

~.) /\ pulley A of radius o if> Gl.lppoded from the foce of o


...erhcal woll by 1wo braces l\Bllt..,AC iogclhor wi~h a tie bot-AD,
as sho....m . A fle,.ible cord EAF ,r;, fast~ned 'to the wall at E,pos99s over. -!he pul~y. ~ car.r1es ot its end f o load~ find the
teris.1te foroo 8 proclu~d 111 the tie bor- AD
~ 1oolb, a -"6 in.

,r-(;/

b -+ fl ,

c I' 7112 fl .

Ans .

s =!l3 lb.

Chapter 7:

1~ A homogeneous slender w ire 12 in . long is ben\ in iwo "right angles os Ghown . Oeiermine the ccordinotes of 1\s cen Ans. i.c " 2.0 in ; Ye "'Zc o.67 in.

ter of gru"" ty .
y

"'

+"
0

+"

shon

2.) I\ steel
of orculo r cnx;i; section noi; a circular steel
hub pressed onto it OS shown . for the dimen.siQl'lG ~hown in t he
f!qure; cle\ermine tho d istance 'f..c from the 1en end or lhe short
~o the cen~er of grovdy c or the compos ite body .
Ans. . Xe r 6Z8 in .

or

homogeneous body consis~s


0 c'1rculor cylindrical portion of rodius r oHoched too l')emisphericol portion of rodius
r OS shown. Oeitermine \he height h
\he cylindricol pod ion i t
3.)

"

or

H-e center of grovity of lhe cornpos'd e body lies o1 lhe cenier


C of ~~he
circulor ~.lope fOce of the hemisphero. Ans . h r/ff

'

4 .) A homogeneous body cos1sls of O righr circular conicol portion ot\oched to 0 herni spher'r col porf1on
rodiu~ r 06 sh()'.'VI'"\ . Determine tho oltihx:le h of \he cone
t he cenler of
grovi ly of tho composi te bod y coinc1des w ith Hle <X:nter C of
the o rculor base of he c one .
Ans. h ..J3 r .

or

,r

!!) Determine

ihe coordinate Ye of the center of 9rovity of

a Gt<'el rive! having the dimenGions shown in Fig. H. Assvme


the heod of \he r ivs f lo be heMi~phericol .
Ans. Ye 1.1s '1n .

Ch opler 0:
1.) F ,nd tho polor momen\ of inerlio of on isosceles t"ionqle
roving bOSO lJ 'b..,, ottilvde h with respe<", \ lo i\s opc7' A.
/\ns. Ja = bh 3 t hbY+6

/<t

2.) Find the polar mol'l'\Cnt cf' inedio of iN! s hodcd orco
shown 1n figure vl.\h re.sped t o point O.
Ans . .J0 = o.27.+ r

'

the polar moment or inertia of lhe o r co or a circulor


sector of rodiuG r 'II.... centrol angle ex. w "dh rospecl to its

3.) F1i-icl

/\ns . Jo "'o<r-4-/ 4 .

center.

-t.) Colculote the moment or incrt io


fi<J. e with respect to the x-axis.

of the

shoded Arco in
26.83 In.""-

Ans. l. "'

&~ Colculote tne morr.en\ c inertia of \he orco of the ongle


soeciion hoving ihc dimonsions shown in fig./\ with reGpoct
\o o centroidol OlCG. porollel to ihe "/. 011iG
/\nr;. I,. " .s.~ in~
y 1t

>

()

I ..
--"'~
-+.

Chop\er- 10:

to move verticolly1.1pword und~r the in ~


f\uet'Ce of grovay with on 1'nitiol velocity
2orps . Find
(o) 'the m()'J(imum helqh\ to which 'd w1t1 riwe 'a.. (b) the time
requiroc:I for ,t to return to ,t1; initial polP'tion. Tol'.e the .lilfof\'
fm9 poin l os tt'C origin GO 1hot )(o O '*., neglect air resi.. '_ .....
Ans . (o.) ~JI 6.2 n:(b) t i .24 sec.
1.) A body &torts

" "

2.) I\ train is moving down


30

mph .

slope of o.ooe with o velocity

of

/\i o certoin inslont tho engineer opplies the brokes

~produces

toto~

resistance to rnof1on equol to one - tenth

of the weigM of the tro1n . Whoi d istonc.e x w "1\I the troin


\ravel before stopping?
,An~. '/. ... 3.27 fl .

the' cro~er or the tool Volcano wos colauloted


in the following rriohner : From a helicopter flying ver-ticolly upword ol 6 m/s, a 1srnall bomb ..-.os ~leosed ot lhe instant H'le
helicopter was 2om. G>bove lhe crater surlo~. lhe oouncl of'
I

eicploGion wos heor-d 9 seconds later. If !he' speed of sound


is 3.35 m/s, ~hot , ~ the depth
lhe cro~er?
.3.) The depth of

or

Ans .

~pth

" 240.2 m.

4 .) The irain trovel i~ from siotion A io .station B wh ich ,s


11<-m opod in o min'1 mum time of one minute . If the froin
starts from rc Gt cit i.+otion A , &.... occelerotc s ot 2.s m/sec~
2
continues ot conston~ speed ~ clecelerotes ot 2..s m/s until ,t siop ot slo1 ion B, find the ma')(;mum .speed in Km/hr.

How lonq did

, t irove l oi

this top .speed .

Ans. $ ~G0.7G, Krn/hr .

J5.) :The m~tion of o portiole ,i:; 9iven by the equation


_: 3t 2 t sl + 0 where S ,c; in rt . ?.. t i~ in sec Compute

7.639S.

.G=

4t

the

volueG of' V ~ o when t 1 sec .

Anr;.v 11f1/s; 0 = 18f1/s 2

Chopter 11 :
1.) /\ motorcycle ?M. rider of totol weight W = .500 lb tr'O'JCI
,n o vedicol plone w ilh cor.stont speed v= +5 mph olonq
the cir-culor curve /\B of ro<f1us r 1000 f1. oG shown . Find
thC reoction R exerted on the motorcycle by the trocK. os
,t posGes the crest C of the curve.
/\ns. R "' 432 lb.
2

~
'-I

~I ~

I
. ~I

t.I

'

/ .
I

or

fr1~iion between wet O!i>pholt po,,.... flC~.1


"' 1he tires of on ouiornobile ic; found to hOve the value ..
i
o.w. At ~t constant speed v c.on the automobile travel oround o curve of rodius r- eoofl . w/out skiclding ,r
-the rood ,s level ?
Ans. Y'niO, "' 49 mph .
. 2.) The

coefficieAt

::.l.) In f iqore below. o hommer of weIQh ~ W 2 tb s1or+s from


~Gt of /\ !lo.._ slideG down 0
for which the coeffi"clen~

roor

of' friction ig
where

,t

p. ""0.2 . r ind 1he di.ston~ x to

h1ts ihe ground .

/\N:> . )( =

fh~
14-.4

p<:)ln! D

ft.

the rnoxirnum rongc: of o pn:Uec the initiol velocity iG inoroased by


Ans. 21 per ccn t .

-4) In whot proportion will

tile be increased
10 per C'lt ?

, f

.e~ In \he figure . ihe pilot of on a irplane flyan9 hon"zontolly .,.,:, ~h constant .Gpeecl v-= 300 mph of on elevation h = 20000

obove

lc:vel ploin wisheG to bomb 0 torgef 0 on the grol,)nd . /\-t who~ ongle t7- below t~ horiu::intol should he see
tnc target ot the insiont of releasing the bomb in order
to score o hit? Noglect air resistorce .
Ans. -c7 - 22 12'
O

Choptcr 12 !
1.) 1hc ormoturc

of on eloctric molor hos on ongular


speed n " 1800 rpm ot tho 1nstont when the powor is cut
off. (o) If i1 cornos to rest in 6 sec, colculote the ongulor
dccolcrohon o< assuming -trot ,t ,s constont . (I'.>) How mony comploto rovol1JHons docs the ormoture moke during
Ans (o) ex= 1oir soe 2 ; (b) go rev.

tn1s por1od?

2.) Considorinq the system in the f1g ure , dctorm'1no ~ho


value
& for which the negative ongulor occolorotion
fr of the bor OA hos Hs rnoximum value.
/\ns. fr = 30

or

1--&

--P-t
+-'::n_B'7e.?--+--~
'lo

rigid semicircular w ire of ro<Jius r iG


t;Uppoded in its own verticol plono by o h-1ngo o1 O &.. o
smooth peg /\ os shown . If tho peg Gtorts from O ~ rnoves
w i th c;onstont s peed Vo olonq the hcr1'z.ontol 1- O)(iS, find
the ongvlor velooty .G or tho w ire o~ the -,nstont when <r
.. eo.
Ans.
Vo/r .
3-) /\ slender bu\

-e. ;

4~ For the. figure .shown w,..,,. 8 rod/s ; o<A" -1.s rod/s ; t "
2 socs. f ind fhe veloc'1\y ~ occolor"Ofioi" of blocll.. C
Ans. Ve .. 12s rnm/s ; Oc::. 37. 5 mm/ 5 2

1oomm

.!1) For- the f ig1.1f'O shown , o<. = 6 0 rod/s

the block. C is rising .

/\ns.

S-= 6 m . How to&l


Ve" 1-+.7 m/s .

1!l0mm

Chopter 1.+: {
bloc.,_ of 'l"eight W ,s in given on ini t'1ol vcl<;>city Vo
olong o rough t'lorizontol
piano'*,
,s ' brought- to rost by fric.
.
tion in o distonco x. Dotermine the coefficient of friction,
o&Guminq thql 'it ,s inclopcodent of voloe1\y .
!\n-;;. p.. - 'lo"/29x
1.) A

.'

Oetcrm'1ne tho dynomicol deflection o thot will bo produced o\ the center of o simply supportod boom by olloa.)

w ing o ~ lb wci9ht to drop onto ,t frorn o hoight of


4in . Whon qroduolly opplied, ihe same lood produces o
static deflection of 0 .1 in . t'1C91e.ct the mosoG'
tho beorn .

or

Ans.

o 1.00 in .

3.) When o ball or w<i1ght W roGt Ono spring of constant K,


,+ produces o "to tic doncction of 1 ,n. How much will iho
some boll com~ss tho ,spring 'f ,ti~ droppecl fromo height
h=1

f1? Hogled tho ma"s of tho spl"ing

Anr;.

<S = 6 in.

4.) /\ grrull blocK of weight W -co1b is given on ini~iol veloYo 10 fps down tho incli11yd p\ono shown in Figur-o. !f
tho eocfficiont of friction batwocn the plonc ~ iho block.'
,s ,M. = o.3, find tho volocity v of the blocK at 0 oflcr ,t hos
\rovclcd o di-stoflcoa ")(=soft.
Arv;. v"' 29.s f ps .

city

~.

tr tho s;yG\orn in Flguro is roleosed from ro't 1n tho c.onflguro+.on .shOwn by solid line!>, frid ~h<; moic imum distance
h 1hot the weight P will foll . N09\ect friction "' assume
thot the pulley.G A 1*.. B aro very small .
A~ . h 4Pqt/(4t\>~-P2 ).

!J)

~'

Choptor

'l

1.!5:

locomotive wbighing 60 tons hos a volocity of 10 mph~


bocks into o froi<Jht corweighing io tons thot,1s ot rest on o
level troc;K.. After c;ouphng iG mode, vvith what voloe1ty v
will the enf1ro s~.Gtem ccntinuo to move ? Neglect oil friction .
1.) /\

Ans v"' 8.57 mph.

2.) /\':Hood block. weighing -'lb rost.s on o srnooth horizon tol .surfocc . /\ rcvolvor Oultct weighing 1/2 oz. i~ ..shot nor' u m\olly into tho .Gido of tho block . If the block. oHoins Cl
velooity of 10 (pi; whoi wos the mu.zz.lc vcloc'1fy v of the

bullet?

Ans. v

~1010 fp6.

mon weighing 1so lb runs~ jumpG from o p1er into o


bool wi~h o horizontal velocity v,: 10 rpG' Assuming thot the
impoct i~ . entirely plastic, find the velocity with which. the
rnon ~ boot will move away f rorn the pier , f the boot wcig
200 lb.
Ans . v =4.3 fps .

3.) A

4) A golf ball dropped from rest onto o corncnt Gidowolk.


rebounds cight-tonths of the height through which ,+ foll :
Neglcchng o'1r rosistonce, dotorrninc ihe coofT1cicnt of
roGtitution.
Ans. e = o.9 .

.5.) In the figvro

.shown, o Grnall cor of weigh~ w stads

frorn rest o-t /\ "'rollG' without friction olong on inclined


plone to B whcro ,t is s1r.1k.cs; o block. ol~o of wo1ght w
'tt..., initially al rest . f\ssvminq o ploshc irnpoct ~ B, the
cor 1... blocK wi ll move frorn B toe aG one porticlc. If the
c.oofT1cient
friction between the bloc!<..~ plane ,c; p.
1/2 , colcvlole the d i~fonce x to pc>1nt C where the bOdlcG
como to rost
Ans. x = 14.2 fl .

or

~
11'

Das könnte Ihnen auch gefallen